You are on page 1of 237

Math reviewer

Probability Questions with Solutions


Tutorial on finding the

probability of an event. In what follows, S is the sample space of the experiment in question and E is the event of
interest. n(S) is the number of elements in the sample space S and n(E) is the number of elements in the event E.

Questions and their Solutions


Question 1
A die is rolled, find the probability that an even number is obtained.

Solution Let us first write the sample space S of the experiment.


S = {1,2,3,4,5,6}
Let E be the event "an even number is obtained" and write it down.
E = {2,4,6}
We now use the formula of the classical probability.
P(E) = n(E) / n(S) = 3 / 6 = 1 / 2

Question 2
Two coins are tossed, find the probability that two heads are obtained. Note: Each coin has
two possible outcomes H (heads) and T (Tails).
Solution
The sample space S is given by.
S = {(H,T),(H,H),(T,H),(T,T)}
Let E be the event "two heads are obtained".
E = {(H,H)}
We use the formula of the classical probability.
P(E) = n(E) / n(S) = 1 / 4

Question 3
Which of these numbers cannot be a probability? a) -0.00001
b) 0.5
c) 1.001
d) 0
e) 1
f) 20%
Solution
A probability is always greater than or equal to 0 and less than or equal to 1, hence
only a) and c) above cannot represent probabilities: -0.00010 is less than 0 and 1.001 is
greater than 1.

Question 4
Two dice are rolled, find the probability that the sum is a) equal to 1
b) equal to 4
c) less than 13
Solution
a) The sample space S of two dice is shown below.
S = { (1,1),(1,2),(1,3),(1,4),(1,5),(1,6)
(2,1),(2,2),(2,3),(2,4),(2,5),(2,6)
(3,1),(3,2),(3,3),(3,4),(3,5),(3,6)
(4,1),(4,2),(4,3),(4,4),(4,5),(4,6)
(5,1),(5,2),(5,3),(5,4),(5,5),(5,6)
(6,1),(6,2),(6,3),(6,4),(6,5),(6,6) }
Let E be the event "sum equal to 1". There are no outcomes which correspond to a sum
equal to 1, hence
P(E) = n(E) / n(S) = 0 / 36 = 0
b) Three possible outcomes give a sum equal to 4: E = {(1,3),(2,2),(3,1)}, hence.
P(E) = n(E) / n(S) = 3 / 36 = 1 / 12
c) All possible outcomes, E = S, give a sum less than 13, hence.
P(E) = n(E) / n(S) = 36 / 36 = 1

Question 5
A die is rolled and a coin is tossed, find the probability that the die shows an odd number and
the coin shows a head. Solution
The sample space S of the experiment described in question 5 is as follows
S = { (1,H),(2,H),(3,H),(4,H),(5,H),(6,H)
(1,T),(2,T),(3,T),(4,T),(5,T),(6,T)}
Let E be the event "the die shows an odd number and the coin shows a head". Event E may
be described as follows
E={(1,H),(3,H),(5,H)}
The probability P(E) is given by
P(E) = n(E) / n(S) = 3 / 12 = 1 / 4

Sponsored Content

Better Than Botox? Mother 57, Looks 35 Without SurgeryHealth News Online


Who Needs Air Conditioning When You Have This Smart DeviceCool Air Technology

Recommended by

Question 6
A card is drawn at random from a deck of cards. Find the probability of getting the 3 of
diamond. Solution
The sample space S of the experiment in question 6 is shwon below

Let E be the event "getting the 3 of diamond". An examination of the sample space shows
that there is one "3 of diamond" so that n(E) = 1 and n(S) = 52. Hence the probability of
event E occurring is given by
P(E) = 1 / 52

Question 7
A card is drawn at random from a deck of cards. Find the probability of getting a
queen. Solution
The sample space S of the experiment in question 7 is shwon above (see question 6)
Let E be the event "getting a Queen". An examination of the sample space shows that there
are 4 "Queens" so that n(E) = 4 and n(S) = 52. Hence the probability of event E occurring is
given by
P(E) = 4 / 52 = 1 / 13

Question 8
A jar contains 3 red marbles, 7 green marbles and 10 white marbles. If a marble is drawn
from the jar at random, what is the probability that this marble is white? Solution
We first construct a table of frequencies that gives the marbles color distributions as follows

color frequency
red 3
green 7
white 10
We now use the empirical formula of the probability
P(E) = Frequency for white color / Total frequencies in the above table
= 10 / 20 = 1 / 2

Question 9
The blood groups of 200 people is distributed as follows: 50 have type A blood, 65
have B blood type, 70 have Oblood type and 15 have type AB blood. If a person from this
group is selected at random, what is the probability that this person has O blood
type? Solution
We construct a table of frequencies for the the blood groups as follows
group frequency
a 50
B 65
O 70
AB 15

We use the empirical formula of the probability


P(E) = Frequency for O blood / Total frequencies
= 70 / 200 = 0.35

Exercises
a) A die is rolled, find the probability that the number obtained is greater than 4.
b) Two coins are tossed, find the probability that one head only is obtained.
c) Two dice are rolled, find the probability that the sum is equal to 5.
d) A card is drawn at random from a deck of cards. Find the probability of getting the King of
heart.

Answers to above exercises


a) 2 / 6 = 1 / 3
b) 2 / 4 = 1 / 2
c) 4 / 36 = 1 / 9
d) 1 / 52

2017 Grade 8 Math Challenge Elimination Round (Questions 1-


25)
Posted on September 8, 2018 | 1 Comment
This is the 2017 MTAP Grade 7 Math Challenge questions 1 to
25.Questions 26-30 including the pdf will be posted very soon.
Questions in the previous years can be accessed on the Past Tests page and
all questions can be found in the All Posts page.
1.) What is the sum of the reciprocals of all the positive divisors of 8?

2.) Factor completely: .

3.) Expand and simplify: .

4.) If ,what is the value of ?

5.) Factor completely:

6.) What is the area of a square whose perimeter is ?


7.) Factor completely: .

8.) What is the product of three consecutive odd integers if the middle
integer is ?

9.) Factor completely: .

10.) Write the fraction in lowest terms and without negative exponents:
.

11.) Simplify using scientific notation:

12.) Expand:

13.) Simplify:

14.) The length, width and height of a box are in the ratio of 5 : 4 : 7. If the
perimeter of the base is 54 cm, find the volume of the box.

15.) Solve for .

16.) The difference between the squares of two consecutive positive


multiples of 5 is 1225. What is the sum of the two multiples of 5?
17.) The ratio of an angle to its complement is 5 : 13. Find the measure of its
complement.

18.) Solve for .

19.) If , for which values of is equal to ?

20.) The midpoint of the segment joining the


points and is . What are the coordinates of ?
21.) Factor completely: .

22.) How many digits does have when expanded?


23.) Give an equation in slope-intercept form of the line passing through the
origin and perpendicular to .

24.) Perform the indicated operation and simplify:

25.) If you want to determine the favorite subject of your class, which
measure of central tendency should you use?

26.) Perform the indicated operations and simplify:

27.) What is the difference between the mean and the median of 2, 3,
3, 4, 6, 7, 8, 15?

28.) The surface area of a cube is 50% more than its volume. Find the
total length of the edges of the cube.

29.) What is the slope of the line with y-intercept 6 and x-intercept
?

30.) If the standard deviation of a set of 20 numbers is 0, what is its


range?

31.) A date is randomly chosen from the month of February 2017.


What is the probability that the date chosen is a prime number
greater than 10?

32.) Solve for .


33.) Donna’s age is 1/5 her grandfather’s age. In 35 years, the
grandfather will be 100 years old. In how many years’ time will
Donna be 1/3 as old as her grandfather?
34.) Find the greatest possible perimeter of an isosceles triangle with
sides 10 cm and 8 cm.

35.) If line l passes through the origin and the point , what is the
equation of l in slope- intercept form?

36.) In △ACE, ∠A = 42◦ and ∠C = 88◦. Arrange the sides of △ACE


from the shortest to the longest.

37.) If Liza is half as old as Paul and Paul will be x years old in five
years, what is Liza’s age 10 years ago?

38.) If two dice are rolled, what is the probability that the sum of the
numbers showing is 7 or 11?

39.) For what values of a and b will the system be inconsistent:

40.) Mark and Marry left home together and drove to their mother’s
house which was 84 km away. Mary drove at 60 kph and Mark drove
20 kph faster than Marry. After travelling 2/7 of the journey, Mark
stopped and wait for Marry. How long did Mark wait?

41.) Solve for and : .

42.) James and John can finish a job in 51 hours. If James can finish
the job in 3 hours less time 7 than John, how long will each one finish
the job alone?

43.) The number of girls in a school is 160 more than 1/3 of the total
enrolment in the school. The number of boys is 280 more than 1/7 of
the total enrolment in the school. How many pupils in the school are
girls?
44.) The graph of two linear functions and are
perpendicular. If , what are the possible values of ?

45.) If the sum of two numbers is 7 and their product is 8. What is the
sum of their cubes?

46.) The mean of 6 numbers is 28 while the mean of another set of 10


numbers is 36. What is the mean of all the numbers combined?

47.) Find all integers x such that

is an integer.

48.) In parallelogram ABCD, ∠A = (8x − 5)◦ and ∠C = (61 − 3x)◦. Find


the measure of ∠B and ∠D.

49.) In quadrilateral ABCD, AB = 8, BC = 12, CD = 16 and DA = 21.


Find the range of possible lengths of diagonal AC.

50.) If a, b, and c are positive numbers such that and

what is the value of

Solved Probability Problems


Solved probability problems and solutions are given here for
a concept with clear understanding.
Students can get a fair idea on the probability questions
which are provided with the detailed step-by-step answers
to every question.
Solved probability problems with solutions:
1.

The graphic above shows a container with 4 blue triangles, 5


green squares and 7 red circles. A single object is drawn at
random from the container.

Match the following events with the corresponding


probabilities:

(i) The objects is not a circle (a) 5/16


(ii) The objects is a triangle (b) 4/16
(c) 7/16
(iii) The objects is not a (d) 9/16
triangle
(e) 12/16
(iv) The objects is not a
square (f) 11/16

(v) The objects is a circle


(vi) The objects is a square

Solution:
Number of blue triangles in a container = 4
Number of green squares = 5
Number of red circles = 7
Total number of objects = 4 + 5 + 7 = 16
(i) The objects is not a circle:
P(the object is a circle)
= Number of circles/Total number of objects
= 7/16
P(the object is not a circle)
= 1 - P(the object is a circle)
= 1 - 7/16
= (16 - 7)/16
= 9/16

(ii) The objects is a triangle:


P(the object is a triangle)
= Number of triangle/Total number of objects
= 4/16

(iii) The objects is not a triangle:


P(the object is a triangle)
= Number of triangles/Total number of objects
= 4/16
P(the object is not a triangle)
= 1 - P(the object is a triangle)
= 1 - 4/16
= (16 - 4)/16
= 12/16

(iv) The objects is not a square:


P(the object is a square)
= Number of squares/Total number of objects
= 5/16
P(the object is not a square)
= 1 - P(the object is a square)
= 1 - 5/16
= (16 - 5)/16
= 11/16

(v) The objects is a circle:


P(the object is a circle)
= Number of circles/Total number of objects
= 7/16

(vi) The objects is a square:


P(the object is a square)
= Number of squares/Total number of objects
= 5/16
Match the following events with the corresponding
probabilities are shown below:

(i) The objects is not a circle (d) 9/16


(ii) The objects is a triangle (b) 4/16
(iii) The objects is not a (e) 12/16
triangle
(f) 11/16
(iv) The objects is not a
square (c) 7/16

(v) The objects is a circle (a) 5/16

(vi) The objects is a square


2. A single card is drawn at random from a standard deck of
52 playing cards.
Match each event with its probability.
Note: fractional probabilities have been reduced to lowest
terms. Consider the ace as the highest card.

(i) The card is a diamond (a) 1/2


(ii) The card is a red king (b) 1/13
(iii) The card is a king or queen (c) 1/26
(iv) The card is either a red or an (d) 12/13
ace
(e) 2/13
(v) The card is not a king
(f) 1/4
(vi) The card is a five or lower
(g) 4/13
(vii) The card is a king
(h) 7/13
(viii) The card is black

Solution:
Total number of playing cards = 52
(i) The card is a diamond:
Number of diamonds in a deck of 52 cards = 13
P(the card is a diamond)
= Number of diamonds/Total number of playing cards
= 13/52
= 1/4

(ii) The card is a red king:


Number of red king in a deck of 52 cards = 2
P(the card is a red king)
= Number of red kings/Total number of playing cards
= 2/52
= 1/26

(iii) The card is a king or queen:


Number of kings in a deck of 52 cards = 4
Number of queens in a deck of 52 cards = 4
Total number of king or queen in a deck of 52 cards = 4 + 4
=8
P(the card is a king or queen)
= Number of king or queen/Total number of playing cards
= 8/52
= 2/13

(iv) The card is either a red card or an ace:


Total number of red card or an ace in a deck of 52 cards =
28
P(the card is either a red card or an ace)
= Number of cards which is either a red card or an ace/Total
number of playing cards
= 28/52
= 7/13

(v) The card is not a king:


Number of kings in a deck of 52 cards = 4
P(the card is a king)
= Number of kings/Total number of playing cards
= 4/52
= 1/13
P(the card is not a king)
= 1 - P(the card is a king)
= 1 - 1/13
= (13 - 1)/13
= 12/13

(vi) The card is a five or lower:


Number of cards is a five or lower = 16
P(the card is a five or lower)
= Number of card is a five or lower/Total number of playing
cards
= 16/52
= 4/13

(vii) The card is a king:


Number of kings in a deck of 52 cards = 4
P(the card is a king)
= Number of kings/Total number of playing cards
= 4/52
= 1/13

(viii) The card is black:


Number of black cards in a deck of 52 cards = 26
P(the card is black)
= Number of black cards/Total number of playing cards
= 26/52
= 1/2
Match the following events with the corresponding
probabilities are shown below:

(i) The card is a diamond (f) 1/4


(ii) The card is a red king (c) 1/26
(iii) The card is a king or queen (e) 2/13
(iv) The card is either a red or an (h) 7/13
ace
(d) 12/13
(v) The card is not a king
(g) 4/13
(vi) The card is a five or lower
(b) 1/13
(vii) The card is a king
(a) 1/2
(viii) The card is black

Probability | Theory, solved examples and


practice questions
By MBA Crystal Ball on July 3, 2015
Categories
Tags
When MS and MBA applicants ask us – ‘What are my chances of getting into Harvard?‘ or
‘What’s my probability of getting scholarships from Oxford?‘ we get tongue-tied. There are so
many variables at play, it’s difficult to give an accurate answer.
But when you get probability questions in your GRE and GMAT exam syllabus, you don’t
have to get flummoxed. Understanding the basic rules and formulas of probability will help
you score high in the entrance exams.

Meaning and definition of Probability


As the Oxford dictionary states it, Probability means ‘The extent to which something is
probable; the likelihood of something happening or being the case’.
In mathematics too, probability indicates the same – the likelihood of the occurrence of an
event.
Examples of events can be :
 Tossing a coin with the head up
 Drawing a red pen from a pack of different coloured pens
 Drawing a card from a deck of 52 cards etc.
Either an event will occur for sure, or not occur at all. Or there are possibilities to different
degrees the event may occur.
An event that occurs for sure is called a Certain event and its probability is 1.
An event that doesn’t occur at all is called an impossible event and its probability is 0.
This means that all other possibilities of an event occurrence lie between 0 and 1.
This is depicted as follows:
0 <= P(A) <= 1
where A is an event and P(A) is the probability of the occurrence of the event.
This also means that a probability value can never be negative.
Every event will have a set of possible outcomes. It is called the ‘sample space’.
Consider the example of tossing a coin.
When a coin is tossed, the possible outcomes are Head and Tail. So, the sample space is
represented as {H, T}.
Similarly when two coins are tossed, the sample space is {(H,H), (H,T), (T,H), (T,T)}.
The probability of head each time you toss the coin is 1/2. So is the probability of tail.

Basic formula of probability


As you might know from the list of GMAT maths formulas, the Probability of the occurrence
of an event A is defined as:
P(A) = (No. of ways A can occur)/(Total no. of possible outcomes)

Another example is the rolling of dice. When a single die is rolled, the sample space is
{1,2,3,4,5,6}.
What is the probability of rolling a 5 when a die is rolled?
No. of ways it can occur = 1
Total no. of possible outcomes = 6
So the probability of rolling a particular number when a die is rolled = 1/6.

Compound probability
Compound probability is when the problem statement asks for the likelihood of the
occurrence of more than one outcome.

Formula for compound probability


 P(A or B) = P(A) + P(B) – P(A and B)
where A and B are any two events.
P(A or B) is the probability of the occurrence of atleast one of the events.
P(A and B) is the probability of the occurrence of both A and B at the same time.

Mutually exclusive events:


Mutually exclusive events are those where the occurrence of one indicates the non-
occurrence of the other
OR
When two events cannot occur at the same time, they are considered mutually exclusive.
Note: For a mutually exclusive event, P(A and B) = 0.

Example 1: What is the probability of getting a 2 or a 5 when a die is rolled?


Solution:
Taking the individual probabilities of each number, getting a 2 is 1/6 and so is getting a 5.
Applying the formula of compound probability,
Probability of getting a 2 or a 5,
P(2 or 5) = P(2) + P(5) – P(2 and 5)
==> 1/6 + 1/6 – 0
==> 2/6 = 1/3.

Example 2: Consider the example of finding the probability of selecting a black card or a 6
from a deck of 52 cards.
Solution:
We need to find out P(B or 6)
Probability of selecting a black card = 26/52
Probability of selecting a 6 = 4/52
Probability of selecting both a black card and a 6 = 2/52
P(B or 6) = P(B) + P(6) – P(B and 6)
= 26/52 + 4/52 – 2/52
= 28/52
= 7/13.

Independent and Dependent Events


Independent Event
When multiple events occur, if the outcome of one event DOES NOT affect the outcome of
the other events, they are called independent events.
Say, a die is rolled twice. The outcome of the first roll doesn’t affect the second outcome.
These two are independent events.

Example 1: Say, a coin is tossed twice. What is the probability of getting two consecutive tails
?
Probability of getting a tail in one toss = 1/2
The coin is tossed twice. So 1/2 * 1/2 = 1/4 is the answer.
Here’s the verification of the above answer with the help of sample space.
When a coin is tossed twice, the sample space is {(H,H), (H,T), (T,H), (T,T)}.
Our desired event is (T,T) whose occurrence is only once out of four possible outcomes and
hence, our answer is 1/4.

Example 2: Consider another example where a pack contains 4 blue, 2 red and 3 black pens.
If a pen is drawn at random from the pack, replaced and the process repeated 2 more times,
What is the probability of drawing 2 blue pens and 1 black pen?
Solution
Here, total number of pens = 9
Probability of drawing 1 blue pen = 4/9
Probability of drawing another blue pen = 4/9
Probability of drawing 1 black pen = 3/9
Probability of drawing 2 blue pens and 1 black pen = 4/9 * 4/9 * 3/9 = 48/729 = 16/243

Dependent Events
When two events occur, if the outcome of one event affects the outcome of the other, they are
called dependent events.
Consider the aforementioned example of drawing a pen from a pack, with a slight difference.
Example 1: A pack contains 4 blue, 2 red and 3 black pens. If 2 pens are drawn at random
from the pack, NOTreplaced and then another pen is drawn. What is the probability of
drawing 2 blue pens and 1 black pen?
Solution:
Probability of drawing 1 blue pen = 4/9
Probability of drawing another blue pen = 3/8
Probability of drawing 1 black pen = 3/7
Probability of drawing 2 blue pens and 1 black pen = 4/9 * 3/8 * 3/7 = 1/14

Let’s consider another example:


Example 2: What is the probability of drawing a king and a queen consecutively from a deck
of 52 cards, withoutreplacement.
Probability of drawing a king = 4/52 = 1/13
After drawing one card, the number of cards are 51.
Probability of drawing a queen = 4/51.
Now, the probability of drawing a king and queen consecutively is 1/13 * 4/51 = 4/663

Conditional probability
Conditional probability is calculating the probability of an event given that another event has
already occured .
The formula for conditional probability P(A|B), read as P(A given B) is
P(A|B) = P (A and B) / P(B)

Consider the following example:


Example: In a class, 40% of the students study math and science. 60% of the students study
math. What is the probability of a student studying science given he/she is already studying
math?
Solution
P(M and S) = 0.40
P(M) = 0.60
P(S|M) = P(M and S)/P(S) = 0.40/0.60 = 2/3 = 0.67

Complement of an event
A complement of an event A can be stated as that which does NOT contain the occurrence of
A.
A complement of an event is denoted as P(Ac) or P(A’).
P(Ac) = 1 – P(A)
or it can be stated, P(A)+P(Ac) = 1
For example,
if A is the event of getting a head in coin toss, Ac is not getting a head i.e., getting a tail.
if A is the event of getting an even number in a die roll, Ac is the event of NOT getting an even
number i.e., getting an odd number.
if A is the event of randomly choosing a number in the range of -3 to 3, Ac is the event of
choosing every number that is NOT negative i.e., 0,1,2 & 3 (0 is neither positive or negative).

Consider the following example:


Example: A single coin is tossed 5 times. What is the probability of getting at least one head?
Solution:
Consider solving this using complement.
Probability of getting no head = P(all tails) = 1/32
P(at least one head) = 1 – P(all tails) = 1 – 1/32 = 31/32.

Sample Probability questions with solutions


Probability Example 1
What is the probability of the occurrence of a number that is odd or less than 5 when a fair
die is rolled.
Solution
Let the event of the occurrence of a number that is odd be ‘A’ and the event of the occurrence
of a number that is less than 5 be ‘B’. We need to find P(A or B).
P(A) = 3/6 (odd numbers = 1,3 and 5)
P(B) = 4/6 (numbers less than 5 = 1,2,3 and 4)
P(A and B) = 2/6 (numbers that are both odd and less than 5 = 1 and 3)
Now, P(A or B) = P(A) + P(B) – P(A or B)
= 3/6 + 4/6 – 2/6
P(A or B) = 5/6.

Probability Example 2
A box contains 4 chocobars and 4 ice creams. Tom eats 3 of them, by randomly choosing.
What is the probability of choosing 2 chocobars and 1 icecream?
Solution
Probability of choosing 1 chocobar = 4/8 = 1/2
After taking out 1 chocobar, the total number is 7.
Probability of choosing 2nd chocobar = 3/7
Probability of choosing 1 icecream out of a total of 6 = 4/6 = 2/3
So the final probability of choosing 2 chocobars and 1 icecream = 1/2 * 3/7 * 2/3 = 1/7

Probability Example 3
When two dice are rolled, find the probability of getting a greater number on the first die
than the one on the second, given that the sum should equal 8.
Solution
Let the event of getting a greater number on the first die be G.
There are 5 ways to get a sum of 8 when two dice are rolled = {(2,6),(3,5),(4,4), (5,3),(6,2)}.
And there are two ways where the number on the first die is greater than the one on the
second given that the sum should equal 8, G = {(5,3), (6,2)}.
Therefore, P(Sum equals 8) = 5/36 and P(G) = 2/36.
Now, P(G|sum equals 8) = P(G and sum equals 8)/P(sum equals 8)
= (2/36)/(5/36)
= 2/5

Probability Quiz: Sample probability questions for


practice
Probability Problem 1
A bag contains blue and red balls. Two balls are drawn randomly without replacement. The
probability of selecting a blue and then a red ball is 0.2. The probability of selecting a blue
ball in the first draw is 0.5. What is the probability of drawing a red ball, given that the first
ball drawn was blue?
a) 0.4
b) 0.2
c) 0.1
d) 0.5
Answer 1
Problem 2
Answer 2

Learn how to solve:


– Simple and compound interest problems
– Speed, distance and time problems
– Ratio and proportion
– List of Maths Formulas

Free Trial: ApplicantLab | GMAT Prep | MBA Abroad Videos


Serious about higher education? Join us on social media for regular updates.

MBA Crystal Ball provides professional Admissions Consulting services. Hire us to improve
your chances of getting into the top international universities. Email: info [at] mbacrystalball
[dot] com

First time here? Learn more.


Get a reality check | Knowledgebase | GMAT | GRE | Scholarships | Business
Schools | Career Options | Our Services

//
MBA Crystal Ball
Our counsellors can help you with career counselling and admissions consulting. Check out
our free resources for: GMAT Preparation & GMAT Syllabus | MBA Subjects | MBA
Scholarships | And much more
Got queries unrelated to this article? Post them on our General Queries Page.

37 Comments
1. algia logan says:
November 20, 2016 at 3:40 am
a number of people gave a hat check girl one hat. suppose all the tickets got misplaced, so all
the hat were given back randomly.
a) if its 2 people determine the probability at least one person got their hat returned.
b) if its 3 people determine the probability at least one person got their hat returned.
c) if its 4 people determine the probability at least one person got their hat returned.
d) if its 5 people determine the probability at least one person got their hat returned.
Hi I’m Algia and I need help in solving this problem, can you help me please.
Reply

o Sam Sam says:


November 12, 2017 at 6:02 pm
a) 1-(0.5)=0.5
b) 1-(0.667*0.5)=0.667
c) 1-(0.75*0.667*0.5)=0.75
d) 1-(0.8*0.75*0.667*0.5)=0.8
Really nice sequence.
P(h’)=1-P(h) etc. At least one hat is correctly returned is compliment that
no hat is returned correctly.
(n-1)/n
Reply

 Random says:
February 3, 2018 at 1:58 am
The math here is totally wrong
Reply

 Sam Sam says:


March 18, 2018 at 9:48 pm
Feel free to enlighten!
Reply

2. abdullah says:
December 5, 2016 at 9:58 pm
Hi
Last question must be 212/216 right ?
Reply

o Santhosh says:
December 12, 2016 at 7:48 pm
Tell me the way u did that sol. Plzz…
Reply

 shaily says:
April 25, 2017 at 1:05 pm
yes its must be 212/216
Reply

 Sangin Pandey says:


November 1, 2017 at 12:21 pm
I think it should be 212/216
Bcz we have 4 number of event to getting a number of sum less
than 5
{(1,1,1),(1,1,2),(1,2,1),(2,1,1)}
It means p(e) = 4/216
Nd getting a number of sum at least 5
Is
1-4/216=212/216
Reply

3. John victor says:


February 23, 2017 at 11:03 am
A woman bought 5basket of tomatos each costing 1250naira,in her discovery she observe that
90% of the tomatos where damage resulting to a loss of 510naira.(a)what is the probability of
obtaining an average of 50 if the cost per bag is 50 above the cost?(b)what will be the actual
price for selling the tomato at cost plus(+) 25%?
Reply

4. ramanan says:
April 26, 2017 at 4:22 pm
the personal director of a company wishes to select applicant for advanced training without
regard to sex. let “W” denotes women and “M” denotes men and the pattern of arrival be M
WWW MMM WW M WWW MMMM W M W MM WWW MM W MMMM WW M WW
MMMM WW M WWWW MM WW M W WW. will you conclude that the applicants have
arrived in a random fashion?
Reply

5. haval says:
May 9, 2017 at 12:24 am
pavement.before any 250 m length of a pavement is accepted by the state highway
department,the thickness of a30 m s mointored by an altrasonic to verify compliance to
specification .each section is rejected if a measurment thickness less than 10cm;otherwise the
all section is accepted .from past experment ,the stat highway engineer know the 85%of all
section constructed by the contructor comply with specification . however the relability of
altrusonic thickness testing is only 75 ,so that there is a 25 percent chane of errorneous
concolusion based on the determenation of thickness with ultrasonic . what is the probablity
that a poorly constructed section is accepted on the base of ultrasonic test?
Reply

6. diriba says:
May 26, 2017 at 5:09 pm
solution
the possible out come of rolling die is =6 here in this case since it is rolled 3 our sample space
is 6×6×6=216
we have asked to solve the probability of sum which will be atleast 5 this means 5 and more is
possible. so that we have to search the possibilities of less than five to easy our work this will
be like[111][112][121] = 3 out comes onlywso p(s`)=3/216 when p(s`) is probability of sum
less than five or probability of sum greater than equal to five.
since the sum of p(s) and p(s`)=1
p(s)=1-p(s~)
1-3/216=213/216
Reply

o Sunny says:
October 14, 2017 at 11:18 pm
what about 2,1,1?
Reply

7. Meychou says:
July 10, 2017 at 8:06 am
Two cards are drawn at random from an ordinary deck of 52 card. Find the probability P that
(a) Both are spade
(b) One is a spade and one is heart
Reply

o Arjuna says:
August 19, 2017 at 6:52 am
Ans:
(a) Probability of getting spade 1st time is 13/52 and Probability of getting
spade 2nd time is is 12/51
Total Probability is 13*12/(52*51) = 156/2652
(b) Probability of getting spade is 13/52 and Probability of getting Heart is
12/51
Total probability is 13*13/(52*51) = 169/2652
Reply

 Michael says:
July 27, 2018 at 11:43 pm
Question ‘b’ says that one is a spade and one is a heart. Therefore the
possibilities are ‘heart-spade’ and ‘spade-heart’.
The answer should be:
((13*13/52*51) + (13*13/52*51)) = 169/2652 + 169/2652 =338/2652 =
13/102
Reply

8. Olasunkanmi Mayowa says:


August 10, 2017 at 11:58 pm
copying the solution offerred by @ diriba
solution
the possible out come of rolling die is =6 here in this case since it is rolled 3 our sample space
is 6×6×6=216
we have asked to solve the probability of sum which will be atleast 5 this means 5 and more is
possible. so that we have to search the possibilities of less than five to easy our work this will
be like[111][112][121] = 3 out comes onlywso p(s`)=3/216 when p(s`) is probability of sum
less than five or probability of sum greater than equal to five.
since the sum of p(s) and p(s`)=1
p(s)=1-p(s~)
1-3/216=213/216
The above solution is good but a little faulty because it considered only the possibility of
obtaining a ‘1’ on the first die, it omitted the possibility of getting a ‘2’ on the first die i.e
(using the same notation) [211], this is the fourth possible outcome.
Hence P(s)= 1- P(s’)
= 1-4/216
=212/216
=53/54
Reply

9. Hema says:
August 15, 2017 at 5:02 pm
A bag contains blue and red balls. Two balls are drawn randomly without replacement. The
probability of selecting a blue and then a red ball is 0.2. The probability of selecting a blue
ball in the first
draw is 0.5. What is the probability of drawing a red ball, given that the first ball drawn was
blue? Solution please
Reply

o Sunny says:
October 14, 2017 at 11:23 pm
Lets assume probability of picking a red ball is X.
The probability of selecting a blue ball and then a red ball,
P(B)*P(R)=.2
.5*X=.2
x=.5/.2
x=.4
Reply

o Pururaba swain says:


January 11, 2018 at 3:58 pm
P(R|B)=P(R and B)/P(B)
=0.2/0.5=0.4
Reply

10. Bharath says:


September 5, 2017 at 7:45 am
The personal director of a company wishes to select applicant for advanced
training without regard to sex. Let ‘W’ denote Women and ‘M’ Denotes men and
the pattern of arrival be M WWW MMM WW M WWW MMMM W M W MM WWW
MM W MMMM WW M WW MMMM WW M WWWW MM WW M W M WW. Will
you conclude that the applicants have arrived in a random fashion?
Reply

11. babar says:


September 6, 2017 at 10:53 pm
The probability of snow tomorrow is 0.6. And the probability that it will bi colder is 0.7. The
probability that it will not snow and not bi colder is 0.1 .What is probability that it will not
snow if it is colder tomorrow?
please solve it … and tell me answer.. thanks …
Reply

12. kaushik says:


September 18, 2017 at 12:17 pm
in a class 10 boys and 5 girls .three students are selected random one after the other.find the
probability that
1)first two are boys and third is girl
2)first and third is of same gender and third is of opposite gender
please help me in solving this
Reply

o Findme says:
June 9, 2018 at 12:00 pm
A) 10/15*9/14*5/13
B) 1st case:
1st & 2nd are boys & 3rd is girl
10/15*9/14*5/13
2nd case:
1st & 2nd are girls & 3rd is boy
5/15*4/14*10/13
Reply

13. Lci says:


October 11, 2017 at 11:28 pm
There are three boxes, one of which contains a prize. A contestant is given two chances, such
that if he chooses the wrong box in the first round, that box is removed from the selection
and he then chooses between the two remaining boxes.
1. What is the probability that the contestant wins?
2. Does the contestant’s probability of winning increases on the second round?
Reply

o quark says:
October 24, 2017 at 11:23 pm
Hi Lei,
It’s a Monty Hall problem. You can google it.
As for your question,
As the first box chosen if found empty is removed and you HAVE/Switch
to pick from other two, the P(W) = 2/3.
Above answer can be explained as Prob. of winning on first box + Prob. of
choosing wrong * Prob. of Choosing right between the two => 1/3+2/3*1/2
=> 2/3
The answer to the second: Yes probability increases as its a 50% chance to
win as 1 wrong box is eliminated.
Reply

o Jesse says:
December 1, 2017 at 2:11 pm
1) 10C2*5C1/15C3?
2) (10C1*5C1*9C1/15C3) + (5C1*10C1*4C1/15C3)?
Reply

14. Rizwan shah says:


November 10, 2017 at 9:28 pm
Plz solve it
XYZ company wants to start a food outlet in pakistan. There is a 40% and 60% chance of
stating in hyderabad and karachi respectively. If he start the outlet in hyderabad there is 30%
chance that it will be in saddar and 70% chance that it will be in defence area. If they start the
outlet in karachi there is 50% chance that it will be in defence, 30% in clifton and 20% in
pechs. Determine probability of starting the outlet in: (a) saddar (b) defence area of any city
(c) clifton given that the outlet is started in karachi
Reply

o Sam Sam says:


November 12, 2017 at 9:02 pm
a) P(H,S) = 40% x 30% =0.4 x 0.3 = 0.12 = 12%
b) P(H, D) + P(K, D) = 40% x 70% + 60% x 50% = 0.4 x 0.7 + 0.6 x 0.5 =
0.28 + 0.3 = 0.58 = 58%
c) P(C|K) = 30%
Reply

15. solomon says:


November 30, 2017 at 7:48 pm
please solve these questions. 1. The probability that a randomly chosen sales prospect will
make a purchase is 0.20. if a sales man calls an 6 prospects, what is the probability that he
will make ……….. a) exactly 4 sales b) 4 or more sales c) no sales
Reply

o Sam Sam says:


April 8, 2018 at 2:48 pm
a) 15*(0.2)^4*(0.8)^2 = 0.01536
b) 0.01536 + 6*(0.2)^5*(0.8)^1 + 1*(0.2)^6*(0.8)^0 = 0.01696
c) (0.8)^6 = 0.262144
Reply

16. abu almostafa says:


December 15, 2017 at 11:16 pm
please solve this problem : Suppose 100 new born in a maternity clinic , 55 were females and
45 males . What is the probability of the next three deliveries are females ?
Reply

o Pururaba swain says:


January 11, 2018 at 4:12 pm
In maternity clinic the probability of new born was females is 55%=0.55
So,the probabilitt of the next three deliveries are females is
0.55×0.55×0.55=0.166 or 16.6%
Reply

17. Jaime says:


January 26, 2018 at 8:23 am
Pls. Answer. Thanks. Five hundred raffle tickets are sold at P25 each for 3 pieces of P4,000,
P250 and P1,000. After each price drawing, the winner is then returned to the collection of
tickets. What is the expected value if the person purchases four (4) tickets?
Reply

18. mubashir azeem says:


January 30, 2018 at 10:35 pm
a major urban hospital has gathered data on the number of heart attack victims seen.the
given table indicates the probabilities of different numbers of heart attack victims being
treated in the emergency room on a typical day number of victims treated (n)fever than 5
,6,7,more than 7 p(n) 0.08,0.16,0.30,0.26,0.20
Reply

19. Geoffrey says:


February 23, 2018 at 3:35 pm
I have a question it goes a manufacturing firm produces units of products in 4 plants A,B,C
and D from the first records of proportion of defectives produced at each brands following
conditional probabilities are set A=0.5 B=1.0,C=0.15, D=0.02 the first plant produces 30% of
the units of the output the 2nd plant produces 25% the 3rd produces 40% and the 4th 5% .a
unit of the products meant at one of this plants is tested and its found to be defective. What’s
the probability that the units was produced in plant C.
Reply

20. Sheila M says:


February 26, 2018 at 12:18 am
I am kindly asking for help with the below question
There are three routes from a person,s home to her place of work.there are four parking lots
where she works, three entrances into her building, two elevators to her floor and one route
from each elevator to her office door.
1. How many ways can she go from her home to her office?
2. If she makes her various choices at random,what is the probability that she will take
mornungside drive,park in lot A,use the south entrance and take elevator 1.
3. As she starts her car one morning, she recalls parking lot A and B are closed for
repair.what is the probability that she will take industrial avenue, park in lot D, use the north
entrance and take elevator 2.

Math Review of Triangle Inequality


 JANUARY 13, 2015
Overview
Students can connect algebra to geometry by expressing geometric
inequalities in compound inequality form. Since the sum of the lengths of
any two sides of a triangle is greater than the length of the third side, there
are three inequality statements. All three inequality statements must be
true for three line segments to form a triangle.

The Triangle Inequality


The perimeter of any polygon is the sum of the measurements of its sides.
Therefore, the perimeter of a triangle is the sum of the measurements of its
three sides. Suppose a triangle has 3 sides with measurements a, b, and c.
In Euclidean geometry, there are 3 basic inequality statements for any
triangle: a +b > c; b +c > a; a +c > b.

Right Triangles
In order for a triangle to be a right triangle, one angle must be a 90 degree
angle. The sides of the triangle are in special relationship with one
another, expressed by the Pythagorean Theorem, such that a 2+ b2 =
c2. Right triangles are a special case of the Triangle Inequality, such that
the measure of the hypotenuse c is greater than the measure of either leg a
or leg b. However, the measure of c must also be less than the sum of both
legs. Suppose an isosceles triangle ABC is constructed so that AB and BC
are equal. By definition, the base angles B and C are also equal. If an
altitude CD is constructed, it will divide ABC into two right triangles,
ACD and CBD. Euclid showed that both AD and CD are shorter than the
hypotenuse AC. According to the Triangle Inequality, the measure of AD
+ CD must be greater than AC.

Non-Triangles
A figure made up of three line segments is not a triangle if any of the
inequalities are untrue. Therefore, all three conditions must be satisfied
such that a + b >c; a + c >b, and b + c =a. If any of the elements are
untrue, the entire statement is untrue. Students can use the Triangle
Inequality as a test for geometric figures, if given the lengths of any three
line segments.

Special Triangles and the Triangle Inequality


One of the most famous of the special triangles is the 3-4-5 triangle. It got
its name because one side measures 3 units, one side measures 4 units, and
one side measures 5 units. It is also a right triangle, and demonstrates the
Pythagorean Theorem, because 3 2+ 42= 52. It also follows the Triangle
Inequality because 3 +4 & gt;5, 3 + 5 >4, and 4 + 5 >3. The sides of the 3-
4-5 triangle follow an arithmetic progression. In algebraic terms, sides
follow a pattern of a, a + d, and a + 2d. Let a be 3, then 3 + 1 is 4, and 3 +
(2·1) = 5. It can be shown that any triangle that follows an arithmetic
progression, and fulfills all three inequalities of the Triangle Inequality is
similar to the 3-4-5 triangle. Suppose that three line segments follow the
arithmetic progression 6, 7, 8. They will form a triangle, because 6 + 7 >8;
7 + 8 >6; 6 + 8 >7. They are also similar to 3, 4, 5 because 6-3 is 3, 7-3 is
4 and 8-3 is 5.

1. Given that Side Z is longer than Side S, and Side Q is congruent to


Side X, what additional piece of information would you need to prove
that Angle L is larger than Angle M?
 Side Y is congruent to Side R.
 Side Y is longer than Side R.
 Side Y is shorter than Side R.
 The angles in Triangle A all add to 180 degrees.
 You do not need any extra information.
2. Angle Y equals Angle X, and Angle A equals Angle B. Side YZ is
congruent to Side BC. Side YX is shorter than Side BA. Which of the
following must be true?

 Angle Z is larger than Angle C.


 Angle Z is smaller than Angle C.
 Angles A, B, and C all equal 60 degrees.
 Angle Z = Angle C.
 Angle X = Angle A.
Relations and Functions
In these lessons, we will look at ordered-pair numbers, relations and an introduction to
functions.

Related Topics: More Algebra Lessons

Ordered-Pair Numbers

An ordered-pair number is a pair of numbers that go together. The numbers are written within
a set of parentheses and separated by a comma.

For example, (4, 7) is an ordered-pair number; the order is designated by the first element 4
and the second element 7. The pair (7, 4) is not the same as (4, 7) because of the different
ordering. Sets of ordered-pair numbers can represent relations or functions.

Relation

A relation is any set of ordered-pair numbers.

Suppose the weights of four students are shown in the following table.

Student 1 2 3 4
Weight 120 100 150 130

The pairing of the student number and his corresponding weight is a relation and can be
written as a set of ordered-pair numbers.

W = {(1, 120), (2, 100), (3, 150), (4, 130)}

The set of all first elements is called the domain of the relation.
The domain of W = {1, 2, 3, 4}

The set of second elements is called the range of the relation.


The range of W = {120, 100, 150, 130}
Function

A function is a relation in which no two ordered pairs have the same first element.

A function associates each element in its domain with one and only one element in its range.

Example:

Determine whether the following are functions

a) A = {(1, 2), (2, 3), (3, 4), (4, 5)}


b) B = {(1, 3), (0, 3), (2, 1), (4, 2)}
c) C = {(1, 6), (2, 5), (1, 9), (4, 3)}

Solution:

a) A = {(1, 2), (2, 3), (3, 4), (4, 5)} is a function because all the first elements are different.

b) B = {(1, 3), (0, 3), (2, 1), (4, 2)} is a function because all the first elements are different. (The
second element does not need to be unique)

c) C = {(1, 6), (2, 5), (1, 9), (4, 3)} is not a function because the first element, 1, is repeated.

A function can be identified from a graph. If any vertical line drawn through the graph cuts the
graph at more than one point, then the relation is not a function. This is called the vertical line
test.

Determining Whether a Relation is a Function


Understanding relations (defined as a set of inputs and corresponding outputs) is an important
step to learning what makes a function. A function is a specific relation, and determining
whether a relation is a function is a skill necessary for knowing what we can graph.
Determining whether a relation is a function involves making sure that for every input there is
only one output.

How to Determine if a Relation is a Function?


A function is a correspondence between a first set, called the domain, and a second set, called
the range, such that each member of the domain corresponds to exactly one member of the
range.
The graph of a function f is a drawing hat represents all the input-output pairs, (x, f(x)). In
cases where the function is given by an equation, the graph of a function is the graph of the
equation y = f(x).
The vertical line test - a graph represents a function if it is impossible to draw a vertical line
that intersects the graph more than once.

 Show Step-by-step Solutions


How to Determine if a Relation is a Function?
This Algebra 1 level math video tutorial
- defines a relation as a set of ordered pairs and a function as a relation with one to one
correspondence
- models how to determine if a relation is a function with two different methods
- shows how to use a mapping and the vertical line test.
- discusses how to work with function notation. It is defined as replacing y in an equation that
is a function.

Examples:
1. Using a mapping diagram, determine whether each relation is a function.
2. Using a vertical line test, determine whether the relation is a function.
3. Make a table for f(t) = 0.5x + 1. Use 1, 2, 3, and 4 as domain values.
4. Evaluate the function rule f(g) = -2g + 4 to find the range for the domain (-1, 3, 5).
 Show Step-by-step Solutions
Determine if a Relation is a Function
This video explains the concepts behind mapping a relation and the vertical line test.
 Show Step-by-step Solutions
Relations and Functions
 Show Step-by-step Solutions

Easy
1.) What is the slope of the line with equation ?

2.) Arnold is 7 years older than her sister Rica. If Arnold will turn 36 seven years from now, how old is Rica now?

3.) Solve for in the equation .

4.) Find the area of the triangle bounded by the line and the coordinate axes.
5.) Factor completely: .

6.) If the base angles of an isosceles triangle measure and degrees, find the measure of the vertex angle.

7.) If , what is ?

8.) If the sides of a triangle have integral lengths, and its perimeter is 13 cm, what is the largest possible length of one
side?

9.) What is the probability of getting a sum of 5 in rolling a pair of dice?

10.) If and , what is ?

11.) Find the distance between the points (2, 1) and (-3, 4).

Average
1.) Find the measure of the acute angle between the hour and the minute hands of 12:15 PM.

2.) If , what is ?

3.) Christine can paint a whole table in 50 minutes, while Vicky can do it twice as fast as Christine. If they continuously
work together, how ling will it take them to finish 15 such tables?

4.) If and , what is ?

5.) One leg of an isosceles right triangle measures 12 cm. What is the area of the circle that passes through the vertices
of the triangle?

6.) Which integer values of satisfy ?

Difficult
1.) If is the largest negative integer that satisfies , what is >

2.) What is the equation (in the form ) of the perpendicular bisector of the segment joining the points (4, -
2) and (-1, 4)?

3.) James jogs every morning, while Dina cycles on the same route. If Dina’s speed is 3.5 times that of James, and Dina
starts 2 hours after James, how many minutes does Dina cycle before she overtake James?

4.) With what polynomial must be divided to get the quotient and a remainder
of ?

5.) Let ABCD be a square, and let E, F, G and H be the midpoint of sides CD, AD, AB and BC, respectively. The segments
AE, BF, CG, and DH create a smaller square inside ABCD. If the area of this smaller square is 1.5 sq. units, what is the
area of ABCD?
6.) The sides of a right triangle are , , , where and are positive numbers. What is the ratio of to ?

Clincher
1.) What is the equation of the line (in the form ) that passes through and the origin?

2.) Two consecutive interior angles of a parallelogram measure and degrees. What is ?

3.) A chemist has two alcohol solutions of the same kind but of different strengths, one with 35% alcohol and another
with 50% alcohol. How many liters of each solution must be mixed to produce 60 liters of solution with 40% alcohol?

Do or Die Question

1.) Find all positive integers , , and d that satisfy the equation , , and .

Answers:

Easy
1.)
2.) 22 years old
3.) -1
4.) 4 sq. units
5.) xy(x – y)(x + y)
6.) 100°
7.) 6
8.) 6 cm
9.)
10.) 74
11.) units

Average
1.) 82.5°
2.) x^2 + 3x + 5
3.) 250 min
4.) 462
5.) 72π sq. cm.
6.) 7, 8, 9

Difficult
1.) 29
2.)
3.) 48
4.)
5.) 7.5 sq. units
6.) 1 : 3

Clincher
1.)
2.) 28
3.) 40 L of 35% alcohol, 20 L of 50% alcohol
Do or Die
1.) , , ,

11.) If and , what is ?

Solution

Answer: -12
12.) Simplify .
Solution

Answer:

13.) If and are positive constants, simplify .

Solution
Note that , and

Now

This is already correct, but if you want your answer in radical form, the previous expression can be converted to

Answer: or
14.) What is the quotient when is divided by ?

Solution
Answer: remainder .
15.) In Item 14, what is the remainder?

Answer:
16.) If , what is ?

Solution

Answer:
17.) If and , what is ?

Solution
and

Substituting the given values above,

Answer: 238
18.) If the length, width, and height of an open-top rectangular box are cm, cm, and cm, what is its
surface area?

Solution
The formula for finding the surface area of a rectangular prism with length , width and
height is . Since the box is open, we subtract lw, which is the top face. So, the surface area of
the open box is .

Substituting, we have
Answer:
19.) A man walked km for 2.5 hrs, then jogged km for 3.5 hrs, and finally walked again km for 4
hrs. If his average speed for the entire exercise was 4 kph, what is x?

Solution

Answer: 3/5 or 0.6


20.) Simplify .

Solution
We can group the expressions as sum and difference of two cubes.

Now, this is in the form of the difference of two squares .


So,

Answer:

21.) When is divided by , the quotient and remainder are both . What is ?
Solution
If we divide 5 by 3, then we get a quotient of 1 and a remainder of 2. If we generalize latex the dividend, as the
divisor, as the quotient and as the remainder, we can form the equation
(*).

In the given, we can see that , , and .

Substituting in *,

Now .

Answer: -4
22.) If two more than twice p is four less than twice q, express q in terms of p.
Solution

Answer:
23.) If x is nonnegative and 3x – 4 ≤ x, what is the least value of x?
Solution
3x – 4 ≤ x
3x – x ≤ 4
2x ≤ 4
x≤2
So the lowest non-negative value is x = 0.
Answer: x = 0
24.) In Item 23, what is the maximum value of x?
Answer: x = 2 (Obvious!)
25.) Solve for in the equation .
Solution

Answer: 56/55
26.) Solve for in the equation .
There are two values for :

Also,

Answer: -1 and 4
27.) If ∠B is the complement of ∠A, and the supplement of ∠A is 138°, what is ∠B – ∠A?
Solution
Complementary angles add up to 90° and supplementary angles add up to 180°. So,
∠A + 138 = 180
∠A = 42.
Now since ∠A and ∠B are complementary,
∠A + ∠B = 90
42 + ∠B = 90
∠B = 90 – 42
∠B = 48.
Now, ∠B – ∠A = 48 – 42 = 6

Answer: 6°
28.) If the diagonals of rectangle ABCD meet E and ∠AEB = 140°, what is ∠EBC?
Solution
∠A + ∠B + ∠E = 180°
∠A + ∠B + 140 = 180°
∠A + ∠B = 40°

Since ABE is an isosceles triangle, ∠A = ∠B. This means that ∠A = ∠B = 20°.


Clearly, ∠ABC = 90°. So,
∠ABE + ∠EBC = 90°
20° + ∠EBC = 90°
∠EBC = 90 – 20°
∠EBC = 70°

Answer: 70°
29.) If two exterior angles of a triangle measures 80° and 130°, what is its smallest interior angle?
Solution
The adjacent interior and exterior angles of a triangle are supplementary. Therefore, the largest exterior angle has the
smallest adjacent interior angle.

The sum of the exterior angle of any polygon is 360°so,

Among the three angles, 150° is the largest and its adjacent interior angle is 30°.

Answer: 30°
30.) In a grouped data, if 100 – 200 forms one class, what is the class interval of the data?
Answer:
Solution
The class interval is the difference between the upper class limit and the lower class limit of a class. Here, the
upper class limit is 200 and the lower class limit is 100. So, the class interval is 200 – 100 = 100.
Answer: 100

31.) Factor completely: .


Solution
Rearranging the terms, we obtain
.
Factoring by grouping, we have

Answer: (a + b)(a – b)(c + d)(c – d)

32.) Simplify .
Solution

Answer:

33.) Perform the indicated operations:


Solution

Answer: 1/x
34.) Perform the indicated operations.

Solution
We can factor the given completely into the following expressions and change division into multiplication by
multiplying the first two expressions with the reciprocal of the third expression. The result of these operations is shown
below.

After cancelling similar terms, we are left with

Answer: 3/2

35.) Simplify .
Solution
Simplifying the rational expressions, we have

.
This simplifies further to

Answer:

36.) Solve for x in the equation


Solution
Getting the least common denominator of the left hand side and combining the terms, we have

Now, we can only equation the numerator.

which is equivalent to $x^2 – 3x + 2 = 0$

Getting the solution, we have


Therefore, or . But x cannot be equal to 2, because the it will make the denominator of the original
expressions undefined. So, the solution is .

Answer: x = 1
37.) The points (0, 0), (2, 3), and (4, 0) form a triangle. What is its perimeter?
Solution
To find the perimeter of the triangle, we need to find the distance between these points and add them. Let’s name the
points A, B, and C respectively.

Using the distance formula, we can subtract the corresponding coordinates, square them, and get the square root.

Distance between A and B is


Distance between A and C is
Distnace between B and C is

So, the perimeter of ABC is

Answer: .
38.) Find the equation of the perpendicular bisector of the segment joining the points (-3, 2) and (5, 2)?
Solution
Let A be the point with coordinates (-3,2) and B be the point with coordinates (5,2). Notice that AB have the same y-
coordinates which mean that it is a horizontal segment. This means that the perpendicular bisector is a vertical line.

To get the perpendicular bisector, we get the midpoint M of AB and find the equation of the vertical line passing
through M.
.

So, the equation of the perpendicular bisector of AB is

Answer: x = 1
39.) A man agrees to invest part of his 1-million-peso inheritance at an annual interest rate of 5%, while the rest at 6%
interest. If, at the end of the year, he needs a total interest of Php 56, 200, how much should he invest at 5%?
Solution

Let x = amount invested at 5% and 1000 000 – x be invested at 6%.

Answer:Php380,000
40.) If 2x + 5y = 10 and x = 3y + 1, what is 11x + 11y?
Solution
Substituting the expression on the right hand side of the second equation to x in the left hand side of the first equation,
we have

(*)

Multiplying by 3, we get (#).

Transposing the second equation and multiplying it by 5, we have

(##)

Adding # and ##, we have

(**)

By * and **

We have 11x + 11y = 35 + 8 = 43$.

Answer: 43

41.) If , what is ?
Solution
, .

Answer: – 13
42.) What is the equation of the line that is parallel to and passes through ?
Solution
Parallel lines have the same slope, so we get the slope of the given line. That is,

So, the slope of the given line is .


By the slope-intercept form, we get the equation of the line parallel to it and passing through .

Multiplying both sides by , we obtain

Simplifying, we have

Answer:
43.) What is the domain of the function ?
Solution
We know that we cannot have a negative square root, so . By squaring both sides and simplifying, this
means that .Since we can substitute any value for except the mentioned restriction, the domain is therefore the
set of real numbers greater than or equal to 1.

Answer: the set of real numbers greater than or equal to 1


44.) Find the range of the function in Item 43.
Solution
The minimum value for and , therefore, the range of is the set of real numbers greater than
or equal to .

Answer: set of real numbers greater than or equal to -2.

45.) If , cm, and cm, what is when ?


Solution

Since corresponding sides of congruent triangles are congruent, . So,

Answer: or .
46.) Let be an isosceles right triangle with C as its hypotenuse, and let and be midpoints on and ,
respectively, such that . If and , what is the area of the trapezoid in terms
of and ?
Solution
The area of a trapezoid is where and are the bases, and is the height. From the given we can
see that , and .
Substituting we have .

This simplifies to

Answer:
47.) If and , what is ?
Solution
Assuming that . From the second equation . This is impossible because . So, y is
negative.

This means that the second equation becomes . Now, suppose is negative, then the first equation
becomes which is impossible because we have already shown that is negative. So, we are left with the systems of
linear equations.

This gives us and . Therefore, .

Answer: 2
48.) If and , what is ?

Solution: To be posted later.


Answer: 3
49.) Let be a square. Three parallel lines , , and pass through A, B, C, respectively. The distance between
l1 and l2 is 4cm, and the distance between l2 and l3 is 5cm. Find the area of the square.

Solution
Draw line perpendicular to and passing through .
Let be the intersection of and and be the intersection of and . Then, (Why?). If we
let be the side of the square,

which means that

So,

Answer: 41
50.) At least how many numbers should be selected from the set {1, 5, 9, 13, … , 125} to be assured that two of the
numbers selected have a sum of 146?

Solution: 20 .

15-second question (2 points each)


1.) What is the slope of the line that passes through the points (-1, 0) and (1, -6)?
Answer: -3

2.) Which one is the largest, 3.2, 10/3, π?


Answer: 10/3

3.) A furniture shop offers a 15% discount in all its terms, and a sala set was marked Php 14, 000. What was the selling
price of the sala set?
Answer: Php 11, 900
4.) Solve for x in the equation |1 – x| = 8.
Answer: 9, -7
5.) If , what is ?
Answer: 4
6.) Factor completely the algebraic expression .
Answer:

7.) Find the least integer that satisfies the inequality .


Answer: 0

8.) Given an angle A, what is the supplement of the complement of A?


Answer: 90° + A

9.) The operation is defined as . What is ?


Answer: -6

10.) What is the quotient when is divided by ?


Answer:

11.) Ana can paint their bedroom in 4 hours, while her older sister, Carla, can paint it in 3 hours. If they work together,
what part of the bedroom can they paint in half an hour?
Answer: 7/24

30-second question (3 points each)


1.) A man started walking at a constant rate of 50 meters per minute. After 30 minutes, from the same point where the
first man started, another man started jogging at a constant rate of 120 meters per minute. After 20 minutes more, who
is ahead? By how many meters?
Answer: First man, by 100 meters.
2.) How far from the origin is the point midway between the points (1, 2) and (-3, 6)?
Answer: √17 units

3.) When is divided by , what is the remainder?


Answer:

4.) Find the equation of the line, in slope-intercept form, that passes through the points (-1, 0) and (1, -6).
Answer: y = -3x – 3
5.) Triangle ABC has sides 13, 13, and 10 cm long. What is the area of the triangle?
Answer: 60 sq. cm.

6.) Find all integers x that satisfy the inequality


Answer: -6, -5, -4, 6, 7, 8
1-minute question (5 points each)
1.) Find the point of intersection of the lines and .
Answer: (-1; 4)

2.) Simplify: .
Answer: 0

3.) What is the area of the region described by the inequalities x ≥ 0, y ≤ 2x, and 2y + 6 ≥ 7x?
Answer: 3 square units
4.) Find the domain of the function
Answer: {x| – 3/2 ≤ x < 0 or x > 0}
5.) State the inverse of the statement “If P(x) ≥ 0 for all x ∈ R, then x < 0 for some x ∈ R.”
Answer: If P(x) < 0 for some x ∈ R, then x ≥ 0 for all x ∈ R.

6.) The sides of a right triangle are x, x + y, and x + 2y, where x and y are positive numbers. What is the ratio of y to x?
Answer: 1:3

Clincher Question
1.) What is the sum of the x- and y-intercepts of line ?
Answer: 7

2.) In ∆ABC, half of ∠A is 30° less than ∠B, and ∠C is twice the complement of ∠A. What kind of triangle is ∆ABC?
Answer: Equilateral or equiangular.

3.) In tossing a pair of dice and a pair of coins simultaneously, what is the probability of getting a sum of 6, a tail, and a
head?
Answer: 5/72

Do or Die Question
In the equations and , where , what is the least possible integral value of ?
Answer: 2

Math series

Solving Linear Equations


Linear Equation:
a mathematical expression that has an equal sign and linear expressions
Variable:
a number that you don't know, often represented by "x" or "y"
but any letter will do!
Variable(s) in linear expressions

 Cannot have exponents (or powers)


For example, x squared or x2
 Cannot multiply or divide each other
For example: "x" times "y" or xy; "x" divided by "y" or x/y
 Cannot be found under a root sign or square root sign (sqrt)
For example: √x or the "square root x"; sqrt (x)
Linear Expression:
a mathematical statement that performs
functions of addition, subtraction, multiplication, and division
These are examples of linear expressions:

x+4 2x + 4 2x + 4y

These are not linear expressions:

x2 (no exponents on variables)


2xy + 4 (can't multiply two variables)
2x / 4y (can't divide two variables)
√x (no square root sign on variables)

Solve these linear equations by clicking and dragging


a number to the "other" side of the equal sign.
Remember that you are "isolating" the unknown "X" to solve the problem.
(More examples are provided below.)

More examples:
Linear equation, solving example #1:
Find x if: 2x + 4 = 10

1. Isolate "x" to one side of the equation 2x + 4 - 4 = 10 - 4


by subtracting 4 from both sides:
2x = 6
2. Divide both sides by 2: 2x / 2 = 6 / 2

x=3
3. Check your work with the original equation: 2x + 4 = 10
(2 * 3) + 4 = 10
6 + 4 = 10

Linear equation, solving example #2:


Find x if: 3x - 4 = -10
(using negatives)

1. Isolate "x" to one side of the equation 3x - 4 + 4 = -10 + 4


by adding 4 to both sides:
3x = -6
2. Divide both sides by 3: 3x / 3 = -6 / 3
x = -2
3. Check your work with the original equation: (3 * -2) - 4 = -10
-6 - 4 = -10

Linear equation, solving example #3:


Find x if: 4x - 4y = 8
(using more than one variable)

1. First step is to isolate "x" 4x - 4y + 4y = 8 + 4y


to one side of the equation by adding 4y to both sides: 4x = 8 + 4y
2. Second step is to divide both sides by 4: 4x / 4 = (8 + 4y) / 4
x=2+y
3. Check your work with the original equation: 4 * (2 + y) - 4y = 8
8 + 4y - 4y = 8
8=8

Linear equation, solving example #4:


Find x if: x + 32 = 12
Note: since the square is on the number and not on the variable,
the expression qualifies as a linear expression
1. First step is to square the number: x + 32 = 12
x + 9 = 12
x + 9 - 9 = 12 - 9
2. Second step is to subtract both sides by 9:
x=3
3 + 32 = 12
3. Check your work with the original equation: 3 + 9 = 12
12 = 12

Algebra: Age Word Problems


Age problems are algebra word problems that deal with the ages of people currently, in the
past or in the future.

How to solve Age Word Problems?

If the problem involves a single person, then it is similar to an Integer Problem. Read the
problem carefully to determine the relationship between the numbers. This is shown in
the examples involving a single person.

If the age problem involves the ages of two or more people then using a table would be a good
idea. A table will help you to organize the information and to write the equations. This is
shown in the examples involving more than one person.

Related Topics: More Algebra Word Problems

How to solve Age Problems Involving A Single Person?

Example:

Five years ago, John’s age was half of the age he will be in 8 years. How old is he now?

Solution:

Step 1: Let x be John’s age now. Look at the question and put the relevant expressions above
it.

Step 2: Write out the equation.


Isolate variable x

Answer: John is now 18 years old.

How to use Algebra to solve age problems?


Examples:
1. Ten years from now, Orlando will be three times older than he is today. What is his current
age?

2. In 20 years, Kayleen will be four times older than she is today. What is her current age?

 Show Step-by-step Solutions

How to solve Age Problems Involving More Than One Person?

Example:

John is twice as old as his friend Peter. Peter is 5 years older than Alice. In 5 years, John will be
three times as old as Alice. How old is Peter now?

Solution:

Step 1: Set up a table.

age now age in 5 yrs


John
Peter
Alice

Step 2: Fill in the table with information given in the question.


John is twice as old as his friend Peter. Peter is 5 years older than Alice. In 5 years, John will be
three times as old as Alice. How old is Peter now?

Let x be Peter’s age now. Add 5 to get the ages in 5 yrs.

age now age in 5 yrs


John 2x 2x + 5
Peter x x+5
Alice x–5 x–5+5

Write the new relationship in an equation using the ages in 5 yrs.

In 5 years, John will be three times as old as Alice.

2x + 5 = 3(x – 5 + 5)
2x + 5 = 3x

Isolate variable x
x = 5

Answer: Peter is now 5 years old.

Example:

John’s father is 5 times older than John and John is twice as old as his sister Alice. In two years
time, the sum of their ages will be 58. How old is John now?

Solution:

Step 1: Set up a table.

age now age in 2 yrs


John’s father
John
Alice

Step 2: Fill in the table with information given in the question.

John’s father is 5 times older than John and John is twice as old as his sister Alice. In two years
time, the sum of their ages will be 58. How old is John now?
Let x be John’s age now. Add 2 to get the ages in 2 yrs.

age now age in 2 yrs


John’s father 5x 5x + 2
John x x+2
Alice

Write the new relationship in an equation using the ages in 2 yrs.

In two years time, the sum of their ages will be 58.

Answer: John is now 8 years old.

How to solve word problems with multiple ages?


Example:
Ben is eight years older than Sarah. 10 years ago, Ben is twice as old as Sarah. Currently, how
old is Ban and Sarah?

 Show Step-by-step Solutions


Algebra word problems with multiple ages
Example:
Mary is three times as old as her son. In 12 years, Mary's age will be one year less than twice
her son's age. How old is each now?

 Show Step-by-step Solutions


Algebra word problem with past and present ages
Example:
Arun is 4 times as old as Anusha is today. Sixty years ago, Arun was 6 times as old as Anusha.
How old are they today?

 Show Step-by-step Solutions


Algebra age word problem with past, present, and future ages
How to organize the data using a table and solve using a system of linear equations?
Examples:
1. Sally is 3 times as old as John. 8 years from now, Sally will be twice as old as John. How old
is John?

2. Kim is 6 years more than twice Timothy's age. 2 years ago, Kim was three times as old as
Timothy. How old was Kim 2 years ago?

3. Leah is 2 less than 3 times Rachel's age. 3 years from now, Leah will be 7 more than twice
Rachel's age. How old will Rachel be in 3 years from now?

4. Becca is twice as old as Susan and Greg is 9 years older than Susan. 3 years ago, Becca was
9 less than 3 times Susan's age. How old is Greg now?

5. Lauren is 3 less than twice Andrew's age. 4 years from now, Sam will be 2 more than twice
Andrew's age. 5 years ago, Sam was three times Andrew's age. How old was Lauren 5 years
ago?

6. Gabby is 1 year more than twice Larry's age. 3 years from now, Megan will be 27 less than
twice Gabby's age. 4 years ago, Megan was 1 year less than 3 times Larry's age. How old will
Megan be 3 years from now?

Some solved examples on Algebra Age problems

1) The present age of Jacob’s father is three times that of Jacob. After 5 years, sum of their ages would be 70 years.
Find their present ages.

Solution :
Let Jacob’s age = x years

His fathers’s age = 3x years

After 5 years

Jacob’s age = x + 5

Father’s age = 3x + 5

Sum of their age = 70

X + 5 + 3x + 5 = 70

4x + 10 = 70

4x = 60

X = 15
Jacob’s age = 15 years and his father’s age = 3(15) = 45 years

2) After 15 years, Ariel’s age will become four times that of her present age. Find her age.

Solution :
Let Ariel’s present age = x years

After 15 years, her age = x + 15

X + 15 = 4x

X – 4x = -15
-3x = -15

X=5

Ariel’s present age = 5 years.

3) John’s mother’s age is 5 years more than the three times of John’s present age. Find John’s present age, if his
mother is 44 years old.

Solution :

Let John’s age = x year

Then 3 x + 5 = 44

Or 3 x = 44 – 5 = 39

Or x = 13 years

John’s present age = 13 years.

___________________________________________________________________
4) The present ages of Deklerk and Saniya are in the ratio 3:4. Five years from now, the ratio of their ages will be
4:5. Find their present ages.

Sponsored Content

Better Than Botox? Mother 57, Looks 35 Without SurgeryHealth News Online


Why Doctors May No Longer Prescribe Metformin (Watch)healthnewstips.today


The air conditioning companies are furious with this new Device!Cool Air Technology

Recommended by
Solution :
Let the present age of Deklerk and Saniya be 3x and 4x years respectively.

After 5 years,

Deklerk's age = 3x + 5

Saniya's age = 4x + 5

According to the question

(3x + 5) / (4x + 5) = 4 /5

⇒ 5 (3x + 5) = 4 (4x + 5) [ Cross multiplication]

⇒ 15x + 25 = 16x + 20

⇒ 15x - 16x = 20 -25

⇒ -x = -5

⇒x=5

∴ Deklerk's present age = 3x = 3(5) = 15 years

and Saniya's age = 4x = 4(5) = 20 years.

Systems of Linear Equations

A Linear Equation is an equation for a line.

A linear equation is not always in the form y = 3.5 − 0.5x,


It can also be like y = 0.5(7 − x)
Or like y + 0.5x = 3.5
Or like y + 0.5x − 3.5 = 0 and more.
(Note: those are all the same linear equation!)

A System of Linear Equations is when we have two or more linear equations working
together.
Example: Here are two linear equations:
2x + y = 5
−x + y = 2

Together they are a system of linear equations.

Can you discover the values of x and y yourself? (Just have a go, play with them a bit.)

Let's try to build and solve a real world example:

Example: You versus Horse

It's a race!

You can run 0.2 km every minute.

The Horse can run 0.5 km every minute. But it takes 6 minutes to saddle the horse.

How far can you get before the horse catches you?

We can make two equations (d=distance in km, t=time in minutes)

 You run at 0.2km every minute, so d = 0.2t


 The horse runs at 0.5 km per minute, but we take 6 off its time: d = 0.5(t−6)

So we have a system of equations (that are linear):

 d = 0.2t
 d = 0.5(t−6)

We can solve it on a graph:

Do you see how the horse starts at 6 minutes, but then runs faster?
It seems you get caught after 10 minutes ... you only got 2 km away.

Run faster next time.

So now you know what a System of Linear Equations is.

Let us continue to find out more about them ....

Solving
There can be many ways to solve linear equations!

Let us see another example:

Example: Solve these two equations:

 x+y=6
 −3x + y = 2

The two equations are shown on this graph:

Our task is to find where the two lines cross.

Well, we can see where they cross, so it is already solved graphically.

But now let's solve it using Algebra!

Hmmm ... how to solve this? There can be many ways! In this case both equations have "y"
so let's try subtracting the whole second equation from the first:

x + y − (−3x + y) = 6 − 2
Now let us simplify it:

x + y + 3x − y = 6 − 2
4x = 4
x=1

So now we know that x=1 is on both lines.

And we can find the matching value of y using either of the two original equations (because we
know they have the same value at x=1). Let's use the first one (you can try the second one
yourself):

x+y=6
1+y=6
y=5

And the solution is:

x = 1 and y = 5
And the graph shows us we are right!

Linear Equations
Only simple variables are allowed. No x2, y3, √x, etc:

Linear vs non-linear

Dimensions
A Linear Equation can be in 2 dimensions ...
(such as x and y)
... or in 3 dimensions ...
(it makes a plane)
... or 4 dimensions ...
... or more!

Common Variables
For the equations to "work together" they share one or more variables:

A System of Equations has two or more equations in one or more variables

Many Variables
So a System of Equations could have many equations and many variables.

Example: 3 equations in 3 variables


2x + y − 2z = 3
x − y − z = 0
x + y + 3z = 12

There can be any combination:

 2 equations in 3 variables,
 6 equations in 4 variables,
 9,000 equations in 567 variables,
 etc.

Solutions
When the number of equations is the same as the number of variables there is likelyto be a
solution. Not guaranteed, but likely.

In fact there are only three possible cases:

 No solution
 One solution
 Infinitely many solutions
When there is no solution the equations are called "inconsistent".
One or infinitely many solutions are called "consistent"

Here is a diagram for 2 equations in 2 variables:


Independent
"Independent" means that each equation gives new information.
Otherwise they are "Dependent".
Also called "Linear Independence" and "Linear Dependence"

Example:

 x+y=3
 2x + 2y = 6

Those equations are "Dependent", because they are really the same equation, just
multiplied by 2.

So the second equation gave no new information.

Where the Equations are True


The trick is to find where all equations are true at the same time.

True? What does that mean?

Example: You versus Horse

The "you" line is true all along its length (but nowhere else).

Anywhere on that line d is equal to 0.2t

 at t=5 and d=1, the equation is true (Is d = 0.2t? Yes, as 1 = 0.2×5 is true)
 at t=5 and d=3, the equation is not true (Is d = 0.2t? No, as 3 = 0.2×5 is not true)

Likewise the "horse" line is also true all along its length (but nowhere else).

But only at the point where they cross (at t=10, d=2) are they both true.

So they have to be true simultaneously ...


... that is why some people call them "Simultaneous Linear Equations"

Solve Using Algebra


It is common to use Algebra to solve them.

Here is the "Horse" example solved using Algebra:

Example: You versus Horse

The system of equations is:

 d = 0.2t
 d = 0.5(t−6)

In this case it seems easiest to set them equal to each other:

d = 0.2t = 0.5(t−6)

Start with:0.2t = 0.5(t − 6)


Expand 0.5(t−6):0.2t = 0.5t − 3
Subtract 0.5t from both sides:−0.3t = −3
Divide both sides by −0.3:t = −3/−0.3 = 10 minutes

Now we know when you get caught!

Knowing t we can calculate d:d = 0.2t = 0.2×10 = 2 km

And our solution is:

t = 10 minutes and d = 2 km

Algebra vs Graphs
Why use Algebra when graphs are so easy? Because:

More than 2 variables can't be solved by a simple graph.

So Algebra comes to the rescue with two popular methods:

 Solving By Substitution
 Solving By Elimination

We will see each one, with examples in 2 variables, and in 3 variables. Here goes ...
Solving By Substitution
These are the steps:

 Write one of the equations so it is in the style "variable = ..."


 Replace (i.e. substitute) that variable in the other equation(s).
 Solve the other equation(s)
 (Repeat as necessary)

Here is an example with 2 equations in 2 variables:

Example:

 3x + 2y = 19
 x+y=8

We can start with any equation and any variable.

Let's use the second equation and the variable "y" (it looks the simplest equation).

Write one of the equations so it is in the style "variable = ...":

We can subtract x from both sides of x + y = 8 to get y = 8 − x. Now our equations look like
this:

 3x + 2y = 19
 y=8−x

Now replace "y" with "8 − x" in the other equation:

 3x + 2(8 − x) = 19
 y=8−x

Solve using the usual algebra methods:

Expand 2(8−x):

 3x + 16 − 2x = 19
 y=8−x

Then 3x−2x = x:

 x + 16 = 19
 y=8−x

And lastly 19−16=3

 x=3
 y=8−x

Now we know what x is, we can put it in the y = 8 − x equation:

 x=3
 y=8−3=5

And the answer is:

x=3
y=5

Note: because there is a solution the equations are "consistent"

Check: why don't you check to see if x = 3 and y = 5 works in both equations?

Solving By Substitution: 3 equations in 3 variables


OK! Let's move to a longer example: 3 equations in 3 variables.

This is not hard to do... it just takes a long time!

Example:

 x+z=6
 z − 3y = 7
 2x + y + 3z = 15

We should line up the variables neatly, or we may lose track of what we are doing:

x + z = 6
− 3y + z = 7
2x + y + 3z = 15

WeI can start with any equation and any variable. Let's use the first equation and the variable
"x".

Write one of the equations so it is in the style "variable = ...":

x = 6−z
− 3y + z = 7
2x + y + 3z = 15

Now replace "x" with "6 − z" in the other equations:

(Luckily there is only one other equation with x in it)

x = 6−z
− 3y + z = 7
2(6−z) + y + 3z = 15

Solve using the usual algebra methods:

2(6−z) + y + 3z = 15 simplifies to y + z = 3:

x = 6−z
− 3y + z = 7
y + z = 3

Good. We have made some progress, but not there yet.

Now repeat the process, but just for the last 2 equations.
Write one of the equations so it is in the style "variable = ...":

Let's choose the last equation and the variable z:

x = 6−z
− 3y + z = 7
z = 3−y

Now replace "z" with "3 − y" in the other equation:

x = 6−z
− 3y + 3−y = 7
z = 3−y

Solve using the usual algebra methods:

−3y + (3−y) = 7 simplifies to −4y = 4, or in other words y = −1

x = 6−z
y = −1
z = 3−y

Almost Done!

Knowing that y = −1 we can calculate that z = 3−y = 4:

x = 6−z
y = −1
z = 4

And knowing that z = 4 we can calculate that x = 6−z = 2:

x = 2
y = −1
z = 4

And the answer is:


x=2
y = −1
z=4

Check: please check this yourself.

We can use this method for 4 or more equations and variables... just do the same steps again
and again until it is solved.

Conclusion: Substitution works nicely, but does take a long time to do.

Solving By Elimination
Elimination can be faster ... but needs to be kept neat.

"Eliminate" means to remove: this method works by removing variables until there is just one
left.

The idea is that we can safely:

 multiply an equation by a constant (except zero),


 add (or subtract) an equation on to another equation

Like in these examples:

WHY can we add equations to each other?

Imagine two really simple equations:

x−5=3
5=5

We can add the "5 = 5" to "x − 5 = 3":

x−5 +5 =3 +5
x=8

Try that yourself but use 5 = 3+2 as the 2nd equation

It will still work just fine, because both sides are equal (that is what the = is for!)
We can also swap equations around, so the 1st could become the 2nd, etc, if that helps.

OK, time for a full example. Let's use the 2 equations in 2 variables example from before:

Example:

 3x + 2y = 19
 x+y=8

Very important to keep things neat:

3x + 2y = 19
x + y = 8

Now ... our aim is to eliminate a variable from an equation.

First we see there is a "2y" and a "y", so let's work on that.

Multiply the second equation by 2:

3x + 2y = 19
2x + 2y = 16

Subtract the second equation from the first equation:

x = 3
2x + 2y = 16

Yay! Now we know what x is!

Next we see the 2nd equation has "2x", so let's halve it, and then subtract "x":

Multiply the second equation by ½ (i.e. divide by 2):

x = 3
x + y = 8

Subtract the first equation from the second equation:

x = 3
y = 5

Done!

And the answer is:

x = 3 and y = 5

And here is the graph:

The blue line is where 3x + 2y = 19 is true

The red line is where x + y = 8 is true

At x=3, y=5 (where the lines cross) they are both true. That is the answer.

Here is another example:

Example:

 2x − y = 4
 6x − 3y = 3

Lay it out neatly:

2x − y = 4
6x − 3y = 3

Multiply the first equation by 3:

6x − 3y = 12
6x − 3y = 3
Subtract the second equation from the first equation:

0 − 0 = 9
6x − 3y = 3

0 − 0 = 9 ???
What is going on here?

Quite simply, there is no solution.

They are actually parallel lines:

And lastly:

Example:

 2x − y = 4
 6x − 3y = 12

Neatly:

2x − y = 4
6x − 3y = 12

Multiply the first equation by 3:

6x − 3y = 12
6x − 3y = 12

Subtract the second equation from the first equation:

0 − 0 = 0
6x − 3y = 3

0−0=0
Well, that is actually TRUE! Zero does equal zero ...

... that is because they are really the same equation ...
... so there are an Infinite Number of Solutions

They are the same line:

And so now we have seen an example of each of the three possible cases:

 No solution
 One solution
 Infinitely many solutions

Solving By Elimination: 3 equations in 3 variables


Before we start on the next example, let's look at an improved way to do things.

Follow this method and we are less likely to make a mistake.

First of all, eliminate the variables in order:

 Eliminate xs first (from equation 2 and 3, in order)


 then eliminate y (from equation 3)

So this is how we eliminate them:

We then have this "triangle shape":

Now start at the bottom and work back up (called "Back-Substitution")


(put in z to find y, then z and y to find x):
And we are solved:

ALSO, we will find it is easier to do some of the calculations in our head, or on scratch paper,
rather than always working within the set of equations:

Example:

 x+y+z=6
 2y + 5z = −4
 2x + 5y − z = 27

Written neatly:

x + y + z = 6
2y + 5z = −4
2x + 5y − z = 27

First, eliminate x from 2nd and 3rd equation.

There is no x in the 2nd equation ... move on to the 3rd equation:

Subtract 2 times the 1st equation from the 3rd equation (just do this in your head or on
scratch paper):

And we get:

x + y + z = 6
2y + 5z = −4
3y − 3z = 15
Next, eliminate y from 3rd equation.

We could subtract 1½ times the 2nd equation from the 3rd equation (because 1½ times 2 is
3) ...

... but we can avoid fractions if we:

 multiply the 3rd equation by 2 and


 multiply the 2nd equation by 3

and then do the subtraction ... like this:

And we end up with:

x + y + z = 6
2y + 5z = −4
z = −2

We now have that "triangle shape"!

Now go back up again "back-substituting":

We know z, so 2y+5z=−4 becomes 2y−10=−4, then 2y=6, so y=3:

x + y + z = 6
y = 3
z = −2

Then x+y+z=6 becomes x+3−2=6, so x=6−3+2=5

x = 5
y = 3
z = −2

And the answer is:


x=5
y=3
z = −2

Check: please check for yourself.

General Advice
Once you get used to the Elimination Method it becomes easier than Substitution, because you
just follow the steps and the answers appear.

But sometimes Substitution can give a quicker result.

 Substitution is often easier for small cases (like 2 equations, or sometimes 3 equations)
 Elimination is easier for larger cases

And it always pays to look over the equations first, to see if there is an easy shortcut ... so
experience helps.

Complex Fractions (page 1 of 2)

Complex Fractions
I sometimes refer to complex fractions as "stacked" fractions, because they tend to have fractions stacked on top of each
other, like this:

 Simplify the following expression:

This fraction is formed of two fractional expressions, one on top of the other. There are two methods for simplifying complex fractions.

The first method is fairly obvious: find common denominators for the complex numerator and complex denominator, convert

the complex numerator and complex denominator to their respective common denominators, combine everything in the
complex numerator and in the complex denominator into single fractions, and then, once you've got one fraction (in the
complex numerator) divided by another fraction (in the complex denominator), you flip-n-multiply. (Remember that, when
you are dividing by a fraction, you flip the fraction and turn the division into multiplication.)

This method looks like this:

Nothing cancels at this point, so this is the final answer.

(The "for x not equal to zero" part is because, in the original expression, " x = 0" would have caused division by zero in
the complex fraction. Depending on your book and instructor, you may not need to account for this technicality . If you're
not sure, ask now, before the test.)

The other method is to find one common denominator for all the fractions in the expression, and then multiply both the complex

numerator and complex denominator by this expression. Then simplify.


This method looks like this:

Then the final answer is:

By multiplying through, top and bottom, by the same thing, I was really just multiplying by 1. This is similar to multiplying the

fraction 1/2 by 2/2 to convert it to 2/4. In my experience, books and teachers often use the first method, but students generally
prefer the second method. When I was in school, I was taught the first method. As soon as I encountered the second method,
I switched to it. In the remaining examples, I will demonstrate this second method, but you can use either method you prefer.

(If your text or instructor requires that you find the restrictions on the domains [the " x not equal to zero" part in the above example],

you might find it helpful to use the "flip-n-multiply" method covered first, since this will give you the full fraction form of the
denominator at some point in the computations.)

 Simplify the following expression:

Copyright © Elizabeth Stapel 2003-2011 All Rights Reserved

Can I start by hacking off the x's? Or lopping off the 3's? (Hint: No!) I can only cancel off factors, not terms, so I can't do
any canceling yet. The first thing I'll do is find the LCM for this expression.
The LCM (Least Common
Multiple, or, for us older
types, the LCD, Lowest
Common Denominator) of
the given denominators
within this complex fraction
is (x – 1)(x + 4), so I'll
multiply through, top and
bottom, by this expression:

(If you're not sure how I

multiplied those factors to get

the cubic results, review this

lesson on multiplying
polynomials.)

Can I now cancel off the x3's? Or cancel the 6's into the 12? Can I go inside the adding and rip out parts of some of the
terms? (Hint: No!) Nothing cancels, so this is the final answer:

Lesson 22, Section 2


Complex fractions -- Division
Back to Section 1
A complex fraction looks like this:

The numerator and/or the denominator are themselves fractions.


To simplify a complex fraction, we can immediately apply the definition of
division (Lesson 5):
a 1
= a·
b b
Any fraction is equal to the numerator times the reciprocal
of the denominator.
Therefore,

p n
= ·
q m

Problem 1. State in words how to simplify a complex fraction.


Rewrite it as the numerator times the reciprocal
of the denominator.

Example 1. Simplify

x2 − 25 x³
Solution. = ·
x 8
x−5

(x + 5)(x − 5) x³
= ·
x8 x−5

x+5
=
x5
Division -- which effectively this is -- becomes multiplication by the reciprocal.
on canceling the x + 2's.
Problem 2. Simplify.

6 x2 3
a) = · =
x 5 8 4x 3

4 1 _4_
b) = · = 2
x−1 x−1 x − 2x + 1

x−2
c) = (x + 2) · = x−2
x+2

x+2 x−2 x2 − 4
d) = · = 2
x+1 x−1 x −1

−h 1 1
e) = · = −
x(x + h) h x(x + h)
The h's cancel. And according to the Rule of Signs, the product is negative. (It's all
right to leave the product in its factored form.)

(x + 2)(x − 2) _x_
f) = ·
3x 2 (x + 2)(x + 3)

x−2 1 x−2
= · =
3x x + 3 3x(x + 3)
Example 3. If a complex fraction looks like this --

-- then we can simplify it by multiplying the numerator and denominator by c.

Problem 3. Simplify the following.

_(x + 1) − 1_ __x__
= =
(x + 1)(x − 1) x2 − 1
Example 4. Simplify

Solution. 1-over any number is its reciprocal. Therefore,


4
=3

Problem 4. Simplify the following.

x+1
a) = b) = x−1
x

WORD PROBLEMS
Examples
Problems

WORD PROBLEMS require practice in translating verbal language into algebraic


language. See Lesson 1, Problem 8. Yet, word problems fall into distinct
types. Below are some examples.
Example 1. ax ± b = c. All problems like the following lead eventually to an equation in
that simple form.
Jane spent $42 for shoes. This was $14 less than twice what she spent for a
blouse. How much was the blouse?
Solution. Every word problem has an unknown number. In this problem, it is the price of
the blouse. Always let x represent the unknown number. That is, let x answer the question.
Let x, then, be how much she spent for the blouse. The problem states that "This" --
that is, $42 -- was $14 less than two times x.
Here is the equation:
2x − 14 = 42.

2x = 42 + 14 (Lesson 9)

= 56.
56
x=
2

= 28.
The blouse cost $28.
Example 2. There are b boys in the class. This is three more than four times the number
of girls. How many girls are in the class?
Solution. Again, let x represent the unknown number that you are asked to find: Let x be
the number of girls.
(Although b is not known, it is not what you are asked to find.)
The problem states that "This" -- b -- is three more than four times x:
4x + 3 = b.
Therefore,
4x = b − 3

b−3
x= .
4
The solution here is not a number, because it will depend on the value of b. This is a
type of "literal" equation, which is very common in algebra.
Example 3. The whole is equal to the sum of the parts.
The sum of two numbers is 84, and one of them is 12 more than the other. What are
the two numbers?
Solution. In this problem, we are asked to find two numbers. Therefore, we must let x be
one of them. Let x, then, be the first number.
We are told that the other number is 12 more, x + 12.
The problem states that their sum is 84:

= 84
The line over x + 12 is a grouping symbol called a vinculum. It saves us writing
parentheses.
We have:
2x = 84 − 12
= 72.
72
x=
2
= 36.
This is the first number. Therefore the other number is
x + 12 = 36 + 12 = 48.
The sum of 36 + 48 is 84.
Example 4. The sum of two consecutive numbers is 37. What are they?
Solution. Two consecutive numbers are like 8 and 9, or 51 and 52.
Let x, then, be the first number. Then the number after it is x + 1.
The problem states that their sum is 37:

= 37
2x = 37 − 1

= 36.

36
x=
2

= 18.
The two numbers are 18 and 19.
Example 5. One number is 10 more than another. The sum of twice the smaller plus three
times the larger, is 55. What are the two numbers?
Solution. Let x be the smaller number.
Then the larger number is 10 more: x + 10.
The problem states:
2x + 3(x + 10) = 55.
That implies
2x + 3x + 30 = 55. Lesson 14.
5x = 55 − 30 = 25.
x = 5.
That's the smaller number. The larger number is 10 more: 15.
Example 6. Divide $80 among three people so that the second will have twice as much as
the first, and the third will have $5 less than the second.
Solution. Again, we are asked to find more than one number. We must begin by
letting x be how much the first person gets.
Then the second gets twice as much, 2x.
And the third gets $5 less than that, 2x − 5.
Their sum is $80:

5x = 80 + 5

85
x=
5

= 17.
This is how much the first person gets. Therefore the second gets
2x = 34.
And the third gets
2x − 5 = 29.
The sum of 17, 34, and 29 is in fact 80.
Example 7. Odd numbers. The sum of two consecutive odd numbers is 52. What are
the two odd numbers?
Solution. First, an even number is a multiple of 2: 2, 4, 6, 8, and so on. It is conventional
in algebra to represent an even number as 2n, where, by calling the variable 'n,' it is
understood that n will take whole number values: n = 0, 1, 2, 3, 4, and so on.
An odd number is 1 more (or 1 less) than an even number. And so we represent an odd
number as 2n + 1.
Let 2n + 1, then, be the first odd number. Then the next one will be 2 more -- it will be
2n + 3. The problem states that their sum is 52:
2n + 1 + 2n + 3 = 52.
We will now solve that equation for n, and then replace the solution in 2n + 1 to find
the first odd number. We have:
4n + 4 = 52
4n = 48
n = 12.
Therefore the first odd number is 2· 12 + 1 = 25. And so the next one is 27. Their
sum is 52.
Problems
Problem 1. Julie has $50, which is eight dollars more than twice what John has. How
much has John? (Compare Example 1.)
First, what will you let x represent?
To see the answer, pass your mouse over the colored area.
To cover the answer again, click "Refresh" ("Reload").
Do the problem yourself first!
The unknown number -- which is how much that John has.
What is the equation?
2x + 8 = 50.
Here is the solution:
x = $21
Problem 2. Carlotta spent $35 at the market. This was seven dollars less than three times
what she spent at the bookstore; how much did she spend there?
Here is the equation.
3x − 7 = 35
Here is the solution:
x = $14
Problem 3. There are b black marbles. This is four more than twice the number of red
marbles. How many red marbles are there? (Compare Example 2.)
Here is the equation.
2x + 4 = b
Here is the solution:
b−4
x= 2

Problem 4. Janet spent $100 on books. This was k dollars less than five times what she
spent on lunch. How much did she spend on lunch?
Here is the equation.
5x − k = 100
Here is the solution:
100 + k
x= 5
Problem 5. The whole is equal to the sum of the parts.
The sum of two numbers is 99, and one of them is 17 more than the other. What are
the two numbers? (Compare Example 3.)
Here is the equation.

Here is the solution:


x = 41
x + 17 = 58
Problem 6. A class of 50 students is divided into two groups; one group has eight less than
the other; how many are in each group?
Here is the equation.

Here is the solution:


x = 29
x − 8 = 21
Problem 7. The sum of two numbers is 72, and one of them is five times the other; what
are the two numbers?
Here is the equation.
x + 5x = 72.
Here is the solution:
x = 12. 5x = 60.
Problem 8. The sum of three consecutive numbers is 87; what are they? (Compare
Example 4.)
Here is the equation.

Here is the solution:


28, 29, 30.
Problem 9. A group of 266 persons consists of men, women, and children. There are four
times as many men as children, and twice as many women as children. How many of each
are there?
(What will you let x equal -- the number of men, women, or children?)
Let x = The number of children. Then

4x = The number of men. And


2x = The number of women.

Here is the equation:


x + 4x + 2x = 266
Here is the solution:
x = 38. 4x = 152. 2x = 76.
Problem 10. Divide $79 among three people so that the second will have three times more
than the first, and the third will have two dollars more than the second. (Compare Example
6.)
Here is the equation.

Here is the solution:


$11, $33, $35.
Problem 11. Divide $15.20 among three people so that the second will have one dollar
more than the first, and the third will have $2.70 more than the second.
Here is the equation.

Here is the solution:


$3.50, $4.50, $7.20.
Problem 12. Two consecutive odd numbers are such that three times the first is 5 more
than twice the second. What are those two odd numbers?
(See Example 7, where we represent an odd number as 2n + 1.)
Solution. Let the first odd number be 2n + 1.
Then the next one is 2n + 3 -- because it will be 2 more.
The problem states, that is, the equation is:
3(2n + 1) = 2(2n + 3) + 5.
That implies:
6n + 3 = 4n + 6 + 5.
2n = 8.
n = 4.
Therefore the first odd number is 2· 4 + 1 = 9. The next one is 11.
And that is the true solution, because according to the problem:
3· 9 = 2· 11 + 5.

INEQUALITIES
The number line
"Or" versus "and"
A continued inequality
Some theorems of inequalities
Solving inequalities

THIS SIGN < means is less than.. This sign > means is greater than. In each case,
the sign opens towards the larger number.
For example, 2 < 5 ("2 is less than 5"). Equivalently, 5 > 2 ("5 is greater than 2").
These are the two senses of an inequality: < and > .
The number line

On the number line, a < b means: a falls to the left of b.


2<5

−1 < 2
−4 < −1

1 > −4
Problem 1. Between each pair, place the correct sign of inequality.
To see the answer, pass your mouse over the colored area.
To cover the answer again, click "Refresh" ("Reload").
Do the problem yourself first!

3<7 8>5 −2 < 1 −6 < −2

−3 > −9 −10 > −12 0 > −9 −6 < 0

"Or" versus "and"


The following is called a compound inequality:
x > 1 and x ≤ 5.
It is a compound sentence whose conjunction is "and." It says that x takes on values
that are greater than 1 and less than or equal to 5.

It is within that interval that x takes its values. The endpoint 1 is not included. x is
definitely greater than 1; we indicate that by placing a parenthesis "(". The endpoint 5 is
included; we indicate that by placing a bracket "]".
Now consider this compound inequaltiy:
x < 1 or x > 5.

Here, the values of x are either less than 1 or greater than 5.


It should be clear that x could not be a number that is less than 1 andgreater than
5. There is no such number.
So, when the conjunction is and,
x > 1 and x ≤ 5.

the values of x fall inside a certain interval. But when the conjunction is or,
x < 1 or x > 5.

the values of x fall outside an interval.


Problem 2. Graph these compound inequalities.
a) x < −1 or x ≥ 3.

b) x ≥ −1 and x < 3.

A continued inequality
a<x<b
That is called a continued inequality. It means
a < x and x < b.
A continued inequality always implies the conjunction and. The sense is always < or ≤.
The continued inequality means:
x falls in the interval between a and b.
"a < x < b" illustrates that.
Note: When neither end point is included, as in this example, we call that
an open interval. When both end points are included --

-- we call that a closed interval. (Otherwise, like the proverbial glass, the interval is half-
open, or half-closed.)
Problem 3. Write as a continued inequality.
a) x > −3 and x < 1.
−3 < x < 1.
b) Graph that continued inequality.

Problem 4. 0 < x < 6. Write that continued inequality as a compound inequality.


x > 0 and x < 6.
Problem 5. Write as a continued inequality:
x < 1 or x > 5.
Not possible! The conjunction must be and.
Problem 6. Name four values that x might have.
a) 1 ≤ x < 3.
For example, 1, 1.2, 2.5, 2.999999999999999999999.
b) x < −1, or x > 1.
For example, −2, −3,456,987, 1.000005, 1023.

* * *
Problem 7. Write in symbols.
a) x is a positive number. x > 0
b) x is a negative number. x < 0
c) x is a non-negative. x ≥ 0. "x is greater than or equal to 0."
Some theorems of inequalities
To prove a statment whose predicate is "is greater than," we must have a definition of "is
greater than." We shall adopt the following. We shall define "a is greater than b" to
mean: a − b is positive. Algebraically:
a > b if and only if a − b > 0.
On the basis of this definition, we can prove various theorems about inequalities.
Theorem 1. We may add the same number to both sides of an inequality, and the sense will not
change.
If
a > b,

then

a + c > b + c.

Note: If c is a negative number, then the theorem implies that we may subtract the
same number from both sides. Just as with equalites, any theorem of addition is also true for
subtraction.
It follows, from this Theorem, that we may transpose.

x+d > e

implies x > e − d.
We could prove that by adding −d to both sides.
We can prove Theorem 1 as follows:
a>b
means
a − b > 0.
Therefore, upon adding and subtracting c :
a − b + c − c>0

(a + c) − (b + c) > 0 (Lesson 7)

Therefore,
a + c > b + c.
Which is what we wanted to prove.
Note that any theorem of inequality is true for an inequality of the opposite sense. For,
we could write: b < a implies b + c < a + c.
Theorem 2. We may multiply both sides of an inequality by the same positive number, and the
sense will not change.
If
a > b, and c > 0,

then

ca > cb.

For example:
4 > −5.

If we now multiply both sides by 3, for example, then

12 > −15.
The sense does not change.
The proof is similar to that of Theorem 1. Simply apply the Rule of Signs.
a>b
implies
a − b > 0.
Therefore, if c is positive, then
c(a − b) > 0
ca − cb > 0.
This means
ca > cb.
This theorem also allows us to divide both sides by the same positive number, because
division is multiplication by the reciprocal.
Theorem 3. If we multiply both sides of an inequality by the same negativenumber, the sense of the
inequality changes.
If
a > b, and c < 0,
then

ca < cb.

Here is an example:
−2 < 5.

If we now multiply both sides by −3, say, then


6 > −15.
The sense changes.
(As in Theorem 2, this one also implies that when we divide both sides by a negative
number, the sense changes, because division is multiplication by the reciprocal.)
Proof:
a> b
implies
a − b > 0.
Therefore, if c is negative, then according to the Rule of Signs:
c(a − b) < 0
ca − cb < 0.
This means
ca < cb.
Theorem 4. If we change the signs on both sides of an inequality,
then the sense of the inequaltiy will change.
If
−a < −b,
then

a > b.

We can see that on the number line.


We could prove it simply by transposing −a and −b.
For example, since
−1 < 2,
then on changing the signs on both sides:
1 > −2.
We may think of Theorem 3 as an instance of Theorem 4, because when we multiply or
divide both sides by the same negative number, the signs on each side necessarily change.
Example 1. If
−5x < 30,
then
x > −6.
The signs on each side have changed. Therefore the sense also must change.
The signs changed because we divided each side by negative 5.
Theorem 5. If a, b are both positive or both negative, then on taking reciprocals, the sense of the
inequality changes.
If
a > b,

then

1 1
< .
a b
We can prove that by dividing both sides by ab (which, since a and bhave the same sign,
will be positive) -- but that will depend on knowing how to reduce fractions.
In any event, since
2 < 5,
then

1 1
2
> 5

Problem 8. Apply the theorems to complete the following with an inequality.


a) If x + 2 < 8 b) If x − 2 < 8

then x<8 − 2 then x<8 + 2

x<6 x < 10

c) If 3x < 12 d) If −3x < 12

12 12
then x < 3 then x > −3

x<4 x > −4
On dividing by a negativenumber, the sense
changes.
Equivalenly, we can think of it as changing
the signs on both sides.
Problem 9. Use Theorem 2 to prove: If a is a positive number less than 1, then a2 is less
than a.
For example, (½)2 = ¼, which is less than ½.
a<1

implies a· a < 1· a

That is, a2 < a .


Problem 10. Assuming that the literals all have positive values, complete the following with
an inequality.

a c b d
If < then > , according to Theorem 5.
b d a c
Problem 11. −a > 0. How is that possible?
a is not a number. It is a variable. It takes values which are numbers. To
find the values of a such that −a > 0, applyTheorem 4, and change the signs on
both sides. But then we must change the sense.
−a > 0
implies a < −0.
That is, since −0 = 0,
a < 0.
a must be a negative number. For example, if a = −5, then −a = −(−5) =
+5, that is, −a is a positive number.
Solving inequalities
A linear inequality has this standard form:
ax + b < c.
When a is positive, then solving it is identical to solving an equation:
ax < c − b
c−b
x<
a
As with equations, the inequality is "solved" when positive x is isolated on the
left. (The above steps follow from Theorems 1 and 2.)
The only difference between solving an inequality and solving an equation, is the
following:
When when we multiply or divide by a negative number,
the sense must change. (Theorem 3.)
Equivalently, when the signs on both sides change, then the sensealso must change
(Theorem 4), as in Example 1.
Example 2.
−2x + 5 < 11
−2x < 6
x > −3.
On going to the last line, the signs on both sides changed. Therefore, the sense also
changes.
The signs changed, of course, because we divided both sides by negative 2.
Alternatively, we could immediately make 2x positive -- by changing all the signs on
both sides. But then we must also change the sense.
−2x + 5 < 11 implies 2x − 5 > −11,
and so on.
Problem 12. Solve each inequality for x. Write a logical sequence of statements.
a) 5x + 3 < 38 b) 5x − 3 < 12

5x < 35 5x < 15

x<7 x<3
c) −2x + 7 < 19 d) −2x − 1 < −9

−2x < 12 −2x < −8

x > −6 x>4
e) −3x < x − 8 f) 4x − 7 < 11x + 7

−3x − x < −8 4x − 11x < 7 + 7

−4x < −8 −7x < 14

x>2 x > −2
Problem 13. In each of the following, what can you conclude about the signs of a and b?
a) ab > 0
a and b must have the same signs. Formally,
a > 0 and b > 0, or a < 0 and b < 0.
Lesson 4: The Rule of Signs.
b) ab < 0
a > 0 and b < 0, or a < 0 and b > 0.
c) ab = 0
Either a = 0 or b = 0.
a
d) >0
b
Same as a). a, b ≠ 0.
a
e) <0
b
Same as b). a, b ≠ 0.
a
f) =0
b
The numerator must be 0. a = 0.
Lesson 5, Problem 17.
Example 3. Solve this inequality for x :
x−3
> 0.
x+5
Solution. According to d) above, the numerator and denominator must
have the same sign. Therefore, either
1) x − 3 > 0 and x + 5 > 0,
or
2) x − 3 < 0 and x + 5 < 0.
Now, 1) implies
x > 3 and x > −5.
Which numbers are these that are both greater than 3 and greater than −5?
Clearly, any number greater than 3 will also be greater than −5. Therefore, 1) has the
solution
x > 3.
Next, 2) implies
x < 3 and x < −5.
Which numbers are these that are both less than 3 and less than −5?
Clearly, any number less than −5 will also be less than 3. Therefore, 2) has the solution
x < −5.
The solution, therefore, is
x < −5 or x > 3.

MULTIPLYING BINOMIALS
Quadratic trinomials
Vocabulary
A binomial is a sum of two terms. a + b.
A trinomial is a sum of three terms, while a multinomial is more than three.
Quadratic is another name for a polynomial of the 2nd degree. For example,
2x2 − 7x + 5.
Problem 1. Which of the following is a quadratic?
To see the answer, pass your mouse over the colored area.
To cover the answer again, click "Refresh" ("Reload").
Do the problem yourself first!
a) 5x2 − 2x + 4 Yes. 2 is the highest exponent.
b) 4x − 9 No. c) x3 + x2 + 1 No.

d) y2 + 8y + 10 Yes. e) x2 + x5 No. f) z2 Yes.


Multiplying binomials
(2x + 3)(x + 5)
Multiplying binomials come up so often that the student should be able to write the product
quickly and easily. It is one of the skills of algebra. Therefore, let us multiply those
binomials and see what results. First, we will distribute 2x to (x + 5), then we will distribute
3.
(2x + 3)(x + 5) = 2x2 + 10x + 3x + 15
= 2x2 + 13x + 15.
When we multiply two such binomials, then, what form is produced?
A quadratic trinomial.
Now, the first term of the trinomial is no mystery: It is 2x· x. And the last term is no
problem either -- it is 3· 5. And so the only question is: Where does the middle
term 13x come from?
The middle term is the sum of the like terms: 10x + 3x :
(2x + 3)(x + 5) = 2x2 + 10x + 3x + 15
= 2x2 + 13x + 15.
If we say that when multiplying binomials, there are four steps --

-- then which steps produce the middle term, which is the sum of the like terms?
The 2nd plus the 3rd.
Or as we often call them, the "Outers" plus the "Inners."
It is skillful to be able to pick out the like terms quickly -- to point at them mentally
with your eyes. Because in the next Lesson we will want to factor 2x2 + 13x + 15.
Will it be factored as
(2x + 5)(x + 3) ?
Or as
(2x + 3)(x + 5) ?
The key lies in choosing the combination that correctly gives the middle term, 13x.
In the first possibility, can we make 13x by combining the Outersplus
the Inners: 6x with 5x ?
No, we cannot. But in the second possibility we can:
10x + 3x = 13x.
2x2 + 13x + 15 will therefore be correctly factored as
(2x + 3)(x + 5).
To check that, the student should practice adding the like terms mentally. That is, to
multiply
(2x + 3)(x + 5) ,
look at 2x and x and write 2x2.
Next, look at 2x· 5 and 3· x together -- "10x + 3x" -- and write 13x.
Finally, 3· 5 = 15.
(2x + 3)(x + 5) = 2x2 + 13x + 15.
With practice, your eye will get used to picking out the Outers plus the Inners. Those
are the like terms. You do algebra with your eyes.
Example 1. (3x − 1)(x + 2). Write only the sum of the like terms.
Answer. 5x. That is,

6x − x = 5x.
Example 2. Multiply (3x − 1)(x + 2).
Answer. 3x2 + 5x − 2
The first term of the trinomial is 3x· x = 3x2.
The middle term we found to be 5x.
And the third term is −1· 2 = −2.
Problem 3. Write only the sum of the like terms.
a) (2x + 1)(x + 3). 7x b) (3x − 2)(x + 4). 10x

c) (5x − 3)(2x − 1). −11x d) (x + 2)(x + 3). 5x

e) (x + 4)(x − 6). −2x f) (x − 5)(x − 3). −8x


Problem 4. Write only the trinomial product -- do not write all four terms.
a) (3x + 2)(2x + 1) = 6x2 + 7x + 2
b) (3x + 2)(2x − 1) = 6x2 + x − 2
c) (3x − 2)(2x + 1) = 6x2 − x − 2
d) (3x − 2)(2x − 1) = 6x2 − 7x + 2
Problem 5. Write only the trinomial product.
a) (7x − 2)(5x + 4) = 35x2 + 18x − 8
b) (x − 1)(3x + 8) = 3x2 + 5x − 8
c) (5x − 4)(x − 1) = 5x2 − 9x + 4
d) (2x + 3)(2x + 5) = 4x2 + 16x + 15
e) (6x + 7)(2x − 3) = 12x2 − 4x − 21
f) (4x − 3)(5x − 2) = 20x2 − 23x + 6
Example 3. 1 the coefficient of x. (x + 2)(x + 3)
When 1 is the coefficient of the x's, the multiplication is especially simple:

When we add the outers plus the inners, the coefficient of x is the sum of the two
numbers. The constant term, as always, is their product.
Example 4. (x − 1)(x + 5) = x2 + 4x − 5.
The outers plus the inners give 4x. More simply, the coefficient of xis −1 + 5. The
constant term is −1· 5.
Problem 6. Write only the trinomial product.
a) (x + 2)(x + 4) = x2 + 6x + 8
b) (x + 2)(x − 4) = x2 − 2x − 8
c) (x − 2)(x − 4) = x2 − 6x + 8
d) (x + 1)(x − 8) = x2 − 7x − 8
e) (x − 1)(x + 8) = x2 + 7x − 8
f) (x − 1)(x − 8) = x2 − 9x + 8
g) (x + 4)(x + 5) = x2 + 9x + 20
h) (x + 2)(x − 8) = x2 − 6x − 16
i) (x − 3)(x − 7) = x2 − 10x + 21
Example 5. Multiply 2(x + 3)(x − 1)
Solution. We can multiply only two factors at a time. First multiply the binomials, then
distribute 2:
2(x + 3)(x − 1) = 2(x2 + 2x − 3)
= 2x2 + 4x − 6.

Example 6. −(x + 4)(x − 5) = −(x2 − x − 20)

= −x2 + x + 20.
Multiply the binomials. Then remove the parentheses.
Example 7. (x − 4)(x + 5)(x − 2)
Multiply two of the binomials. Then multiply that product with third. (Lesson
14, Example 4.)
(x − 4)(x + 5)(x − 2) = (x − 4)(x2 + 3x − 10)
= x3 + 3x2 − 10x
− 4x2 − 12x + 40
= x3 − x2 − 22x + 40.
Problem 7. Multiply.
a) 4(x − 1)(x + 3) = 4(x2 + 2x − 3) = 4x2 + 8x − 12
b) x(x − 2)(3x + 4) = x(3x2 − 2x − 8) = 3x3 − 2x2 − 8x
c) −(x + 1)(x − 2) = −(x2 − x − 2) = −x2 + x + 2
d) (x + 1)(x + 2)(x + 3) = (x + 1)(x2 + 5x + 6)

= x 3 + 5x 2 + 6x + x 2 + 5 x + 6

= x3 + 6x2 + 11x + 6
e) (x − 2)(x + 4)(x − 5) = (x − 2)(x2 − x − 20)
= x3 − x2 − 20x − 2x2 + 2x + 40

= x3 − 3x2 − 18x + 40

FACTORING TRINOMIALS
2nd Level:
Positive leading term
Quadratics in different arguments

FACTORING IS THE REVERSE of multiplying. Skill in factoring, then, depends upon skill in
multiplying: Lesson 16. As for a quadratic trinomial --
2x2 + 9x − 5
-- it will be factored as a product of binomials:
(? ?)(? ?)
The first term of each binomial will be the factors of 2x2, and thesecond term will be
the factors of 5.
Now, how can we produce 2x2? There is only one way: 2x· x :
(2x ?)(x ?)
And how can we produce 5? Again, there is only one way: 1· 5. But does the 5 go
with 2x --
(2x 5)(x 1)
or with x --
(2x 1)(x 5) ?
Notice: We have not yet placed any signs
How shall we decide between these two possibilities? It is the combination that will
correctly give the middle term, 9x :
2x2 + 9x − 5.
Consider the first possibility:
(2x 5)(x 1)
Is it possible to produce 9x by combining the outers and the inners:
2x (that is, 2x· 1) with 5x ?
No, it is not. Therefore, we must eliminate that possibility and consider the other:
(2x 1)(x 5)
Can we produce 9x by combining 10x with 1x ?
Yes -- if we choose +5 and −1:
(2x − 1)(x + 5)
(2x − 1)(x + 5) = 2x2 + 9x − 5.
Skill in factoring depends on skill in multiplying -- particularly in picking out the middle
term
Problem 1. Place the correct signs to give the middle term.
a) 2x2 + 7x − 15 = (2x − 3)(x + 5)
b) 2x2 − 7x − 15 = (2x + 3)(x − 5)
c) 2x2 − x − 15 = (2x + 5)(x − 3)
d) 2x2 − 13x + 15 = (2x − 3)(x − 5)
Note: When the constant term is negative, as in parts a), b), c), then the signs in each
factor must be different. But when the constant term is positive, as in part d), the signs must
be the same. Usually, however, that happens by itself.
Nevertheless, can you correctly factor the following?
2x2 − 5x + 3 = (2x − 3)(x − 1)
Problem 2. Factor these trinomials.
a) 3x2 + 8x + 5 = (3x + 5)(x + 1)
b) 3x2 + 16x + 5 = (3x + 1)(x + 5)
c) 2x2 + 9x + 7 = (2x + 7)(x + 1)
d) 2x2 + 15x + 7 = (2x + 1)(x + 7)
e) 5x2 + 8x + 3 = (5x + 3)(x + 1)
f) 5x2 + 16x + 3 = (5x + 1)(x + 3)
Problem 3. Factor these trinomials.
a) 2x2 − 7x + 5 = (2x − 5)(x − 1)
b) 2x2 − 11x + 5 = (2x − 1)(x − 5)
c) 3x2 + x − 10 = (3x − 5)(x + 2 )
d) 2x2 − x − 3 = (2x − 3)(x + 1)
e) 5x2 − 13x + 6 = (5x − 3)(x − 2)
f) 5x2 − 17x + 6 = (5x − 2)(x − 3)
g) 2x2 + 5x − 3 = (2x − 1)(x + 3)
h) 2x2 − 5x − 3 = (2x + 1)(x − 3)
i) 2x2 + x − 3 = (2x + 3)(x − 1)
j) 2x2 − 13x + 21 = (2x − 7 )(x −3)
k) 5x2 − 7x − 6 = (5x + 3)(x − 2)
l) 5x2 − 22x + 21 = (5x − 7)(x − 3)
Example 1. 1 the coefficient of x2. Factor x2 + 3x − 10.
Solution. The binomial factors will look like this:
(x a)(x b)
Since the coefficient of x2 is 1, it will not matter in which binomial we put the numbers.
(x a)(x b) = (x b)(x a).
Now, what are the factors of 10? Let us try 2 and 5:
x2 + 3x − 10 = (x 2)(x 5).
We must now choose the signs so that -- as always -- the sum of the outers plus
the inners will equal the middle term, which is +3x.
Choose +5 and −2.
x2 + 3x − 10 = (x − 2)(x + 5).
When the coefficient of x2 is 1, we may simply look for two numbers whose product is
10, the numerical term, and whose algebraic sum is +3, the coefficient of x. That will ensure
that the sum of the outers plus the inners will equal the middle term.
Again, the order of the factors does not matter.
(x − 2)(x + 5) = (x + 5) (x − 2).
Example 2. Factor x2 − x − 12.

Solution. <="" td="" style="font-size: 15pt; font-family: Garamond, serif; font-weight: 500;
color: rgb(0, 0, 102); width: 85px;">
(x ?)(x ?)
There are several factors of 12. Let us try 2 and 6.
(x 2)(x 6)
But there is no way to choose signs for 6x and 2x to give the middle term, which is −x.
Let's try 3 and 4:
(x 3)(x 4)
That will work. Choose −4 and +3.
(x + 3)(x − 4) = x2 − x − 12.
Skill in factoring depends on skill in multiplying, specifically in constructing the middle
term. (Lesson 16.)
Problem 4. Factor. Again, the order of the factors does not matter.
a) x2 + 5x + 6 = (x + 2)(x + 3)
b) x2 − x − 6 = (x − 3 )(x + 2)
c) x2 + x − 6 = (x + 3 )(x − 2)
d) x2 − 5x + 6 = (x − 3)(x − 2 )
e) x2 + 7x + 6 = (x + 1)(x + 6 )
f) x2 − 7x + 6 = (x − 1)(x − 6 )
g) x2 + 5x − 6 = (x − 1)(x + 6 )
h) x2 − 5x − 6 = (x + 1)(x − 6 )
Problem 5. Factor.
a) x2 − 10x + 9 = (x − 1 )(x − 9)
b) x2 + x − 12 = (x + 4)(x − 3)
c) x2 − 6x − 16 = (x − 8)(x + 2)
d) x2 − 5x − 14 = (x − 7)(x + 2)
e) x2 − x − 2 = (x + 1)(x − 2)
f) x2 − 12x + 20 = (x − 10 )(x − 2)
g) x2 − 14x + 24 = (x − 12 )(x − 2)
Example 3. Factor completely 6x8 + 30x7 + 36x6.
Solution. To factor completely means to first remove any common factors (Lesson 15).
6x8 + 30x7 + 36x6 = 6x6(x2 + 5x + 6).
Now continue by factoring the trinomial:
= 6x6(x + 2)(x + 3).
Problem 6. Factor completely. First remove any common factors.
a) x3 + 6x2 + 5x = x(x2 + 6x + 5) = x(x + 5)(x + 1)
b) x5 + 4x4 + 3x3 = x3(x2 + 4x + 3) = x3(x + 1)(x + 3)
c) x4 + x3 − 6x2 = x2(x2 + x − 6) = x2(x + 3)(x − 2)
d) 4x2 − 4x − 24 = 4(x2 − x − 6) = 4(x + 2)(x − 3)
e) 6x3 + 10x2 − 4x = 2x(3x2 + 5x − 2) = 2x(3x − 1)(x + 2)
f) 12x10 + 42x9 + 18x8 = 6x8(2x2 + 7x + 3) = 6x8(2x + 1)(x + 3).

2nd Level
Example 4. Factor by making the leading term positive.
−x2 + 5x − 6 = −(x2 − 5x + 6) = −(x − 2)(x − 3).
Problem 7. Factor by making the leading term positive.
a) −x2 − 2x + 3 = −(x2 + 2x − 3) = −(x + 3)(x − 1)
b) −x2 + x + 6 = −(x2 − x − 6) = −(x + 2)(x − 3)
c) −2x2 − 5x + 3 = −(2x2 + 5x − 3) = −(2x − 1)(x + 3)
Quadratics in different arguments
Here is the form of a quadratic trinomial with argument x :
ax2 + bx + c.
The argument is whatever is being squared. x is being squared. x is called the
argument. The argument appears in the middle term.
a, b, c are called constants. In this quadratic,
3x2 + 2x − 1,
the constants are 3, 2, −1.
Now here is a quadratic whose argument is x3:
3x6 + 2x3 − 1.
x6 is the square of x3. (Lesson 13: Exponents.)
But that quadratic has the same constants -- 3, 2, − 1 -- as the one above. In a sense, it
is the same quadratic only with a different argument. For it is the constants that distinguish a
quadratic.
Now, since the quadratic with argument x can be factored in this way:
3x2 + 2x − 1 = (3x − 1)(x + 1),
then the quadratic with argument x3is factored the same way:
3x6 + 2x3 − 1 = (3x3 − 1)(x3 + 1).
Whenever a quadratic has constants 3, 2, −1, then for any argument, the factoring will
be
(3 times the argument − 1)(argument + 1).

Example 5. z2 − 3z − 10 = (z + 2)(z − 5).

x8 − 3x4 − 10 = (x4 + 2)(x4 − 5).


The trinomials on the left have the same constants 1, −3, −10 but different
arguments. That is the only difference between them. In the first, the argument is z. In the
second, the argument is x4.
(The square of x4 is x8.)
Each quadratic is factored as
(argument + 2)(argument − 5).
Every quadratic with constants 1, −3, −10 will be factored that way.
Problem 8.
a) Write the form of a quadratic trinomial with argument z.
az2 + bz + c
b) Write the form of a quadratic trinomial with argument x4.
ax8 + bx4 + c
c) Write the form of a quadratic trinomial with argument xn.
ax2n + bxn + c
Problem 10. Multiply out each of the following, which have the same constants, but
different argument.
a) (z + 3)(z − 1) = z2 + 2z − 3 b) (y + 3)(y − 1) = y2 + 2y − 3

c) (y6 + 3)(y6 − 1) = y12 + 2y6 − 3


d) (x5 + 3)(x5 − 1) = x10 + 2x5 − 3
Problem 11. Factor each quadratic.
a) x2 − 6x + 5 = (x − 1)(x − 5)
b) z2 − 6z + 5 = (z − 1)(z − 5)
c) x8 − 6x4 + 5 = (x4 − 1)(x4 − 5)
d) x10 − 6x5 + 5 = (x5 − 1)(x5 − 5)
e) x6y6 − 6x3y3 + 5 = (x3y3 − 1)(x3y3 − 5)
f) sin2x − 6 sin x + 5 = (sin x − 1)(sin x − 5).
sin2x -- "sine squared x" -- means (sin x)2.
Problem 12. Factor each quadratic.
a) x4 − x2 − 2 = (x2 − 2)(x2 + 1)
b) y6 + 2y3 − 8 = (y3 + 4)(y3 − 2)
c) z8 + 4z4 + 3 = (z4 + 1)(z4 + 3)
d) 2x10 + 5x5 + 3 = (2x5 + 3)(x5 + 1)
e) x4y2 − 3x2y − 10 = (x2y + 2)(x2y − 5)
f) cos2x − 5 cos x + 6 = (cos x − 3)(cos x − 2)
"cos x" is an abbreviation for the trigonometric function "cosine of angle x." It is
conventional to write the square of cos x as cos2x ("cosine squared x"). In calculus, rather
than solve triangles with cos x, we do algebra with it. The above is an example. And so while
you may think that in that example you are doing trigonometry, you are doing nothing but
algebra.
HE SQUARE OF A BINOMIAL
Perfect square trinomials
The square numbers
The square of a binomial
Geometrical algebra
2nd level
(a + b)³
The square of a trinomial
Completing the square

LET US BEGIN by learning about the square numbers. They are the numbers
1· 1 2· 2 3· 3
and so on. The following are the first ten square numbers -- and their roots.
Square numbers 1 4 9 16 25 36 49 64 81 100
Square roots 1 2 3 4 5 6 7 8 9 10
1 is the square of 1. 4 is the square of 2. 9 is the square of 3. And so on.
The square root of 1 is 1. The square root of 4 is 2. The square root of 9 is 3. And so
on.
In a multiplication table, the square numbers lie along the diagonal.

The square of a binomial


(a + b)2
The square of a binomial comes up so often that the student should be able to write the final
product immediately. It will turn out to be a very specific trinomial. To see that, let us
square (a + b):
(a + b)2 = (a + b)(a + b) = a2 + 2ab + b2.
For, the outers plus the inners will be
ab + ba = 2ab.
The order of factors does not matter.
(a + b)2 = a2 + 2ab + b2
The square of any binomial produces the following three terms:
1. The square of the first term of the binomial: a2
2. Twice the product of the two terms: 2ab
3. The square of the second term: b2
The square of every binomial has that form: a2 + 2ab + b2.
To recognize that is to know an essential product in the "multiplication table" of
algebra.
(See Lesson 8 of Arithmetic: How to square a number mentally, particularly the square
of 24, which is the "binomial" 20 + 4.)
Example 1. Square the binomial (x + 6).
Solution. (x + 6)2 = x2 + 12x + 36
x2 is the square of x.
12x is twice the product of x with 6. (x· 6 = 6x. Twice that is 12x.)
36 is the square of 6.
x2 + 12x + 36 is called a perfect square trinomial -- which is the square of a binomial.
Example 2. Square the binomial (3x − 4).
Solution. (3x − 4)2 = 9x2 − 24x + 16
9x2 is the square of 3x.
−24x is twice the product of 3x· −4. (3x· −4 = −12x. Twice that is −24x.)
16 is the square of −4.
Note: If the binomial has a minus sign, then the minus sign appears only in the middle
term of the trinomial. Therefore, using the double sign ± ("plus or minus"), we can state
the rule as follows:
(a ± b)2 = a2 ± 2ab + b2
This means: If the binomial is a + b, then the middle term will be +2ab; but if the
binomial is a − b, then the middle term will be −2ab
The square of +b or −b, of course, is always positive. It is always +b2.
Example 3. (5x3 − 1)2 = 25x6 − 10x3 + 1
25x6 is the square of 5x3. (Lesson 13: Exponents.)
−10x3 is twice the product of 5x3 and −1. (5x3 times −1 = −5x3. Twice that is
−10x3.)
1 is the square of −1.
The student should be clear that (a + b)2 is not a2 + b2, any more than
(a + b)3 is equal to a3 + b3.
An exponent may not be "distributed" over a sum.
(See Topic 25 of Precalculus: The binomial theorem.)

Example 4. Is this a perfect square trinomial: x2 + 14x + 49 ?


Answer. Yes. It is the square of (x + 7).
x2 is the square of x. 49 is the square of 7. And 14x is twice the product of x· 7.
In other words, x2 + 14x + 49 could be factored as
x2 + 14x + 49 = (x + 7)2
Note: If the coefficient of x had been any number but 14, this would not have been a
perfect square trinomial.
Example 5 Is this a perfect square trinomial: x2 + 50x + 100 ?
Answer. No, it is not. Although x2 is the square of x, and 100 is the square of 10, 50x is
not twice the product of x· 10. (Twice their product is 20x.)
Example 6 Is this a perfect square trinomial: x8 − 16x4 + 64 ?
Answer. Yes. It is the perfect square of x4 − 8.
Problem 1. Which numbers are the square numbers?
To see the answer, pass your mouse over the colored area.
To cover the answer again, click "Refresh" ("Reload").
Do the problem yourself first!
1, 4, 9, 16, 25, 36, 49, 64, etc.
They are the numbers 12, 22, 32, and so on.
Problem 2.
a) State in words the rule for squaring a binomial.
The square of the first term.
Twice the product of the two terms.
The square of the second term.
b) Write only the trinomial product: (x + 8)2 = x2 + 16x + 64
c) Write only the trinomial product: (r + s)2 = r2 + 2rs + s2
Problem 3. Write only the trinomial product.
a) (x + 1)2 = x2 + 2x + 1 b) (x − 1)2 = x2 − 2x + 1

c) (x + 2)2 = x2 + 4x + 4 d) (x − 3)2 = x2 − 6x + 9

e) (x + 4)2 = x2 + 8x + 16 f) (x − 5)2 = x2 − 10x + 25

g) (x + 6)2 = x2 + 12x + 36 h) (x − y)2 = x2 − 2xy + y2


Problem 4. Write only the trinomial product.
a) (2x + 1)2 = 4x2 + 4x + 1 b) (3x − 2)2 = 9x2 − 12x + 4
c) (4x + 3)2 = 16x2 + 24x + 9 d) (5x − 2)2 = 25x2 − 20x + 4

e) (x3 + 1)2 = x6 + 2x3 + 1 f) (x4 − 3)2 = x8 − 6x4 + 9

g) (xn + 1)2 = x2n + 2xn + 1 h) (xn − 4)2 = x2n − 8xn + 16


Problem 5. Factor: p2 + 2pq + q2.
p2 + 2pq + q2 = (p + q)2
The left-hand side is a perfect square trinomial.
Problem 6. Factor as a perfect square trinomial -- if possible.
a) x2 − 4x + 4 = (x − 2)2 b) x2 + 6x + 9 = (x + 3)2

c) x2 − 18x + 36 Not possible. d) x2 − 12x + 36 = (x − 6)2

e) x2 − 3x + 9 Not possible. f) x2 + 10x + 25 = (x + 5)2


Problem 7. Factor as a perfect square trinomial, if possible.
a) 25x2 + 30x + 9 = (5x + 3)2
b) 4x2 − 28x + 49 = (2x − 7)2
c) 25x2 − 10x + 4 Not possible.
d) 25x2 − 20x + 4 = (5x − 2)2
e) 1 − 16y + 64y2 = (1 − 8y )2
f) 16m2 − 40mn+ 25n2 = (4m − 5n)2
g) x4 + 2x2y2 + y4 = (x2 + y2)2
h) 4x6 − 10x3y4 + 25y8 Not possible.
i) x12 + 8x6 + 16 = (x6 + 4)2
j) x2n + 8xn + 16 = (xn + 4)2
Geometrical algebra
Here is a square whose side is a + b.
It is composed of
a square whose side is a,
a square whose side is b,
and two rectangles ab.
That is,
(a + b)2 = a2 + 2ab + b2.

2nd Level

THE DIFFERENCE
OF TWO SQUARES
Geometrical algebra
Summary of Multiplying/Factoring
2nd level:
The form (a + b)(a − b)
Factoring by grouping
The sum and difference of odd powers
The difference of even powers

WHEN THE SUM of two numbers multiplies their difference --


(a + b)(a − b)
-- then the product is the difference of their squares:
(a + b)(a − b) = a2 − b2
For, the like terms will cancel. (Lesson 16.)
Symmetrically, the difference of two squares can be factored:
x2 − 25 = (x + 5)(x − 5)
x2 is the square of x. 25 is the square of 5.
The sum of two squares -- a2 + b2 -- cannot be factored. See Section 2.

Example 1. Multiply (x3 + 2)(x3 − 2).


Solution. Recognize the form:
(a + b)(a − b)
The product will be the difference of two squares:
(x3 + 2)(x3 − 2) = x6 − 4.
x6 is the square of x3. 4 is the square of 2.
Upon seeing the form (a + b)(a − b), the student should not do the FOIL method. The
student should recognize immediately that the product will be a2 − b2.
That is skill in algebra.
And the order of factors never matters:
(a + b)(a − b) = (a − b)(a + b) = a2 − b2.
Problem 1. Write only final product..
a) (x + 9)(x − 9) = x2 − 81
b) (y + z)(y − z) = y2 − z2
c) (6x − 1)(6x + 1) = 36x2 − 1
d) (3y + 7)(3y − 7) = 9y2 − 49
e) (x3 − 8)(x3 + 8) = x6 − 64
f) (xy + 10)(xy − 10) = x2y2 − 100
g) (xy2 − z3)(xy2 + z3) = x2y4 − z6
h) (xn + ym)(xn − ym) = x2n − y2m
Problem 2. Factor.
a) x2 − 100 = (x + 10)(x − 10)
b) y2 − 1 = (y + 1)(y − 1)
c) 1 − 4z2 = (1 + 2z)(1 − 2z)
d) 25m2 − 9n2 = (5m + 3n)(5m − 3n)
e) x6 − 36 = (x3 + 6)(x3 − 6)
f) y4 − 144 = (y2 + 12)(y2 − 12)
g) x8 − y10 = (x4 + y5)(x4 − y5)
h) x2n − 1 = (xn + 1)(xn − 1)
Problem 3. Factor completely.
a) x4 − y4 = (x2 + y2)(x2 − y2)

= (x2 + y2)(x + y)(x − y)

b) 1 − z8 = (1 + z4)(1 − z4)

= (1 + z4)(1 + z2)(1 − z2)

= (1 + z4)(1 + z2)(1 + z)(1 − z)


Problem 4. Completely factor each of the following. First remove a common
factor. Then factor the difference of two squares.
a) xy2 − xz2 = x(y2 − z2) = x(y + z)(y − z)
b) 8x2 − 72 = 8(x2 − 9) = 8(x + 3)(x − 3)
c) 64z − z3 = z(64 − z2) = z(8 + z)(8 − z)
d) rs3 − r3s = rs(s2 − r2) = rs(s + r)(s − r)
e) 32m2n − 50n3 = 2n(16m2 − 25n2) = 2n(4m + 5n)(4m − 5n)
f) 5x4y5 − 5y5 =
f) 5y5(x4 − 1) = 5y5(x2 + 1)(x + 1)(x − 1)
Geometrical algebra

The entire figure on the left is a square on side a. The square b2 has been inserted in the
upper left corner, so that the shaded area is the difference of the two squares, a2 − b2.
Now, in the figure on the right, we have moved the rectangle (a − b)b to the side. The
shaded area is now equal to the rectangle
(a + b)(a − b).
That is,
a2 − b2 = (a + b)(a − b).
*
The Difference of Two Squares completes our study of products of binomials. Those
products come up so often that the student should be able to recognize and apply each
form.
Summary of Multiplying/Factoring
In summary, here are the four forms of Multiplying/Factoring that characterize algebra.

1. Common Factor 2(a + b) = 2a + 2b

2. Quadratic Trinomial (x + 2)(x + 3) = x2 + 5x + 6

3. Perfect Square Trinomial (x − 5)2 = x2 − 10x + 25

4. The Difference of Two Squares (x + 5)(x − 5) = x2 − 25

Problem 5. Distinguish each form, and write only the final product.
a) (x − 3)2 = x2 − 6x + 9. Perfect square trinomial.
b) (x + 3)(x − 3) = x2 − 9. The difference of two squares.
c) (x − 3)(x + 5) = x2 + 2x − 15. Quadratic trinomial.
d) (2x − 5)(2x + 5) = 4x2 − 25. The difference of two squares.
e) (2x − 5)2 = 4x2 − 20x + 25. Perfect square trinomial.
f) (2x − 5)(2x + 1) = 4x2 − 8x − 5. Quadratic trinomial.
Problem 6. Factor. (What form is it? Is there a common factor? Is it the difference of
two squares? . . . )
a) 6x − 18 = 6(x − 3). Common factor.
b) x6 + x5 + x4 + x3 = x3(x3 + x2 + x + 1). Common factor.
c) x2 − 36 = (x + 6)(x − 6). The difference of two squares.
d) x2 − 12x + 36 = (x − 6)2. Perfect square trinomial.
e) x2 − 6x + 5 = (x − 5)(x − 1). Quadratic trinomial.
f) x2 − x − 12 = (x − 4)(x + 3)
g) 64x2 − 1 = (8x + 1)(8x − 1)
h) 5x2 − 7x − 6 = (5x + 3)(x − 2)
i) 4x5 + 20x4 + 24x3 = 4x3(x2 + 5x + 6) = 4x3(x + 3)(x + 2)

2nd Level
ALGEBRAIC FRACTIONS
The principle of equivalent fractions
Reducing to lowest terms
2nd Level

FRACTIONS IN ALGEBRA are often called rational expressions. (See Topic 18


of Precalculus.) We begin with the principle of equivalent fractions, which
appears as follows:
x ax
=
y ay
"Both the numerator and denominator may be multiplied
by the same factor."
x ax
Both x and y have been multiplied by the factor a. and are
y ay
called equivalent fractions. That means that, in a calculation, we could replace either one with
the other.
It is the same rule as in arithmetic:
2 10
=
3 15
on multiplying both 2 and 3 by 5.
Problem 1. Write the missing numerator.
To see the answer, pass your mouse over the colored area.
To cover the answer again, click "Refresh" ("Reload").
Do the problem yourself first!

6 18
=
n 3n
The denominator has been multiplied by 3; therefore the numerator will
also be multiplied by 3.
Problem 2. Write the missing numerator.
4 4x
=
x x2
The denominator has been multiplied by x; therefore the numerator will
also be multiplied by x.
Problem 3. Write the missing numerator.
m 8x2m
=
x 8x3
The denominator has been multiplied by 8x2; therefore the numerator will
also be multiplied by 8x2.
The student should expect that the original denominator on the left will be a factor of
the new denominator on the right. It must be a factor because to produce the new
denominator, the original denominator was multiplied
Problem 4. Write the missing numerator.
("The denominator has been multiplied by _____. Therefore the numerator will also be
multiplied by ____.")
a 5a 3 6 5 5y
a) = b) = c) = 2
b 5b x 2x y y

8 8y a 2x2a b bx2y
d) = e) = f) = 22
x xy x 2x3 y xy

p prs 2 2ac 4 4(x + 1)


g) = h) = i) =
q qrs b abc x x(x + 1)
?
Example 1. a=
b
a
Solution. To explain the solution, we will write a as .
1
a ab
=
1 b
Since 1 has been multiplied by b, then so will a.
The numerator ab however is simply the product of a times b. It is a kind of cross-
multiplying, and the student should not have to write the denominator 1.
ab
a=
b
You do algebra with your eyes.
Problem 5. Write the missing numerator.
3x 2ab x³
a) x = b) 2 = c) x =
3 ab x2
x 2x + 2 x2 − 1
d) 1 = e) 2 = f) x + 1 =
x x+1 x−1
Part f) is The Difference of Two Squares.
There will be more problems of this type at the 2nd Level.
Reducing to lowest terms
The numerator and denominator of a fraction are called its terms. Since we
may multiply both terms, then, symmetrically, we may divide both terms.
ax x
=
ay y
"Both the numerator and denominator may be divided
by a common factor."
When we do that, we say that we have reduced the fraction to its lowest terms.
Again, this is the same as in arithmetic.
5x
Example 2. Reduce .
5y
5x x
Answer. = .
5y y
5 is a common factor of the numerator and denominator. Therefore we may divide
each of them by 5.
One often hears that we have "canceled" the 5's. But that can be very dangerous
expression, as the following examples will show.

5+x
Example 3. Reduce .
5+y
Answer. This can not be reduced. We cannot "cancel" the 5's, because 5 is not a factor of
either the numerator or the denominator. In both of them, 5 is a term.
We cannot cancel terms.
The word term does double duty in algebra. We speak of the terms of a
sum and also the terms of a fraction, which are the numerator and
denominator. A fraction is in its lowest terms when the terms -- the
numerator and denominator -- have no common factors.
3a + 6b + 9c
Example 4. Reduce .
12d
Answer. When the numerator or denominator is made up of a sum, then if every term has a
common factor, we may divide every term by it.
In this example, every term in both the numerator and denominator has a factor
3. Therefore, upon dividing every term by 3, we can write immediately:
3a + 6b + 9c a + 2b + 3c
=
12d 4d
There is no more reducing. The numerator and denominator no longer have a
common factor.
We could show the common factor explicitly, by writing
3a + 6b + 9c 3(a + 2b + 3c)
12d
= 3·4d
But to actually write that is not required.

This example illustrates the following:


To divide a sum -- 3a + 6b + 9c -- by a number,
we must be able to divide every term by that number.

3a + 6b + 8c
Example 5. Reduce .
12a
Answer. Not possible. The numerator and denominator have no common factor. 3 is not
a common factor, because 3 is not a factor of 8. 2 is not a common factor, because 2 is not a
factor of 3. And a is not a common factor. That fraction is in its lowest terms.
Again, to divide a sum, every term must have a common factor, as in Example 4.

8x
Example 6. Reduce .
8x + 10
Answer. 2 is a factor of every term in both the numerator and denominator. Therefore we
may divide every term by 2.
8x 4x
= .
8x + 10 4x + 5
There is no more dividing. We cannot "cancel" the 4x's, because 4xis not a factor of
the denominator. 4x is not a factor of 5.
Problem 6. Reduce to lowest terms.
3a a 8xy 2y 56y 8
a) = b) = c) =
3b b 12x 3 77xy 11x
2x + 6 x + 3 , on dividing every term in both the numerator
d) =
4x + 8 2x + 4 and denominator by 2.
2x + 3 Not possible. The terms of the numerator and denominator have no
e) =
4x + 9 common factor.

x
Example 7. Reduce .
4x
x 1
Answer. = ,
4x 4
on dividing both the numerator and denominator by x.
Note that we must write 1 in the numerator, for x = 1· x.

4x
Example 8. Reduce .
x
4x
Answer. = 4.
x
It is not the style in algebra to write 1 as a denominator.

x−3
Example 9. Reduce .
6(x − 3)
x−3 1
Answer. = .
6(x − 3) 6

We can view x − 3 as a factor of the numerator, because


x − 3 = (x − 3)· 1
Again, we must write 1 in the numerator.
Problem 7. Reduce.
2a a 1 2x 1
a) =2 b) = c) =
a ab b 8xy 4y
5(x − 2) x+1 1 3(x + 2)x 1
d) =5 e) = f) =
x−2 2(x + 1) 2 6(x + 2)xy 2y

15x
Example 10. Reduce .
5x − 3
Answer. Not possible. The numerator and denominator have no common factor.
x2 − x − 6
Example 11. Reduce .
x2 − 4x + 3
Answer. In its present form, there is no reducing -- because there are no factors. But we
can make factors:
x2 − x − 6 (x − 3)(x + 2) x + 2
= =
x − 4x + 3 (x − 3)(x − 1) x − 1
2

(x −3) is now seen to be a common factor. We can divide by it. And when we do, the
numerator and denominator no longer have a common factor. The end.

4x³ − 9x2
Example 12. Reduce: .
4x³ + 6x2
Answer. The only common factor is x2. And we could display it by factoring both the
numerator and denominator:
4x³ − 9x2 x2(4x − 9) 4x − 9
= 2 =
4x³ + 6x 2x (2x + 3) 2(2x + 3)
2

The fraction is now in its lowest terms. No common factors.


Problem 8. Reduce.
5x 5x x
a) = =
10x + 15 5(2x + 3) 2x + 3

3x − 12 3(x − 4) x − 4
b) = =
3x 3x x

12x − 18y + 21z 4x − 6y + 7z


c) = ,
6y 2y
upon dividing every term by their common factor, 3.

2m 2m 2
d) = =
m2 − 2m m(m − 2) m − 2

x2 − x x(x − 1)
e) = = x−1
x x

12x2 12x2 3
f) = = 3
16x − 20x
5 2 4x (4x − 5) 4x − 5
2 3

x+3 x+3 1
g) = =
4x + 12 4(x + 3) 4
2x − 8 2(x − 4)
h) = =2
x−4 x−4

2x − 2y 2(x − y) 2
i) = =
3x − 3y 3(x − y) 3
Problem 9. Make factors, and reduce.
x2 − 2x − 3 (x + 1)(x − 3) x − 3
a) = =
x2 − x − 2 (x + 1)(x − 2) x − 2

x2 + x − 2 (x + 2)(x − 1) x − 1
b) = =
x2 − x − 6 (x + 2)(x − 3) x − 3

x2 − 2x + 1 (x − 1)2 x−1
c) = =
x2 − 1 (x + 1)(x − 1) x + 1

x2 − 100 (x + 10)(x − 10)


d) = = x − 10
x + 10 x + 10

x+3 x+3 1
e) = =
x + 6x + 9
2 (x + 3)2 x+3

x³ + 4x2 _ x2(x + 4) x2
f) = =
x2 + x − 12 (x − 3)(x + 4) x − 3
Problem 10. Reduce to lowest terms -- if possible.

3+x
a) Not possible. The numerator and denominator have no common factors.
3x

8a + b
b) Not possible. Again, no common factors.
2ab

8a + 2b 2(4a + b) 4a + b
c) = =
2ab 2ab ab

6a + b Not possible. The numerator and denominator have no common


d) 3a + b factors. 3 is not a factor of either the numerator or denominator. It is a
factor only of the first term in each.

6(a + b)
e) =2
3(a + b)
2x + 4y + 6z x + 2y + 3z
f) = Divide every term by 2.
10 5

2x + 4y + 5z Not possible. The numerator and denominator have no common


g)
10 factor.

(x + 1) + (x + 2)
h) Not possible. The numerator is not made up of factors.
(x + 1)(x + 3)

(x + 1)(x + 2) x + 2
i) =
(x + 1)(x + 3) x + 3

ab + c
j) = Not possible. The numerator and denominator have no common factors.
abc

ab + ac a(b + c) b + c
k) = =
abc abc bc

x2 − x − 12 (x + 3)(x − 4) x − 4
l) = =
x2 + x − 6 (x + 3)(x − 2) x − 2

2nd Level

MULTIPLYING AND DIVIDING


ALGEBRAIC FRACTIONS
The rule
Reducing
Section 2
Complex fractions -- Division

TO MULTIPLY FRACTIONS, multiply the numerators and multiply the denominators,


as in arithmetic.

Problem 1. Multiply.
To see the answer, pass your mouse over the colored area.
To cover the answer again, click "Refresh" ("Reload").
Do the problem yourself first!
2 5 10 3ab 4a2b 3a³b2
a) · = 2 b) · = The 4's cancel.
x x x 4c 5d 5cd
3x 6x2 18x³
c) · = 2 The Difference of Two Squares
x+1 x−1 x −1
x−3 x−2 x 2 − 5x + 6
d) · = 2
x+1 x+1 x + 2x + 1

b b
If a multiplication looks like this: a· or · a, multiply only
c c
the numerator.
b ab
a· =
c c
Problem 2. Multiply.
2x 2x 2 3x2 21x5
a) x · = b) · 7x3 =
3 3 4 4
x−3 x2 − 9
c) (x + 3)· =
x+6 x+6

x2 − 2x + 5
d) · 2x3 = No canceling!
6x2 − 4x + 1 6 x 2 − 4x + 1
Reducing
If any numerator has a divisor in common with any denominator,
they may be canceled.
a c e ce
· · =
b d a bd
The a's cancel.
For if we took the trouble to multiply, and write
ace
bda
then it's obvious that we could divide both the numerator and denominator by a. It is more
skillful, then, to reduce before multiplying.
Problem 3. Multiply. Reduce first.
ab ed hcf bh
a) · · =
cd fg ake gk
(x − 2)(x + 2) __2x__ x−2
b) · =
8x (x + 2)(x − 1) 4(x − 1)

__x³__ x+3 __1_


c) · 7 =
(x + 2)(x + 3) x (x + 2)x4

x(x + 1) 2 x(x + 1) __2__ _x_


d) · 2 = · =
6 x −1 6 (x + 1)(x − 1) 3(x − 1)

b ab x+2
e) aq· = f) 10· = 5(x + 2) = 5x + 10
cq c 2

5x 5x 2
g) 3x· =
6 2

−a 1 1
h) · = −
b a b
The a's cancel as −1, which on multiplication with 1 makes the fraction
itself negative (Lesson 4).

x2 − 4x − 5 x2 − 5x + 6
Example 1. Multiply · 2
x2 − x − 6 x − 6x + 5
Solution. Although the problem says "Multiply," that is the last thing to do in
algebra. First factor. Then reduce. Finally, multiply.
And remember: Only factors can be divided.

x2 − 4x − 5 x2 − 5x + 6 (x + 1)(x − 5) (x − 3)(x − 2)
· = ·
x2 − x − 6 x2 − 6x + 5 (x + 2)(x − 3) (x − 1)(x − 5)

x+1 x−2
= ·
x+2 x−1

x2 − x − 2
=
x2 + x − 2
Problem 4. Multiply.

__x2__ x2 − 9 __x2__ (x − 3)(x + 3)


a) · = ·
x + x − 12 2x
2 6
(x + 4)(x − 3) 2x6
1 x+3
= ·
x+4 2x 4

_x+3_
=
2 x 5 + 8x 4
x2 − 2x + 1 x2 + x − 6 __(x − 1)2__ (x + 3)(x − 2)
b) · = ·
x − x − 12 x − 6x + 5
2 2
(x − 4)(x + 3) (x − 1)(x − 5)

x−1 x−2
= ·
x−4 x−5

x 2 − 3x + 2
=
x2 − 9x + 20
x2 + 3x − 10 x2 + 5x − 6 (x + 5)(x − 2) (x − 1)(x + 6)
c) · = ·
x2 + 4x − 12 x + 4x − 5
2
(x + 6)(x − 2) (x − 1)(x + 5)

= 1
_x³_ x2 + x − 2 __x2__
d) · · 2
x2 − 1 x4 x + 4x + 4
___x³___ (x − 1)(x + 2) __x2__
= · ·
(x + 1)(x − 1) x4 (x + 2)2

1
= ·
x+1 x+2

_ _ x_ _
=
x 2 + 3x + 2

ADDING ALGEBRAIC FRACTIONS


Different denominators -- The LCM
2nd level

THERE IS ONE RULE for adding or subtracting fractions: The denominators must
be the same -- just as in arithmetic.
a b a+b
+ =
c c c
Add the numerators, and place their sum
over the common denominator.

6x + 3 4x − 1 10x + 2
Example 1. + =
5 5 5
The denominators are the same. Add the numerators as like terms.

6x + 3 4x − 1
Example 2. −
5 5
To subtract, change the signs of the subtrahend, and add.

6x + 3 4x − 1 6x + 3 − 4x + 1 2x + 4
− = =
5 5 5 5
Problem 1.
To see the answer, pass your mouse over the colored area.
To cover the answer again, click "Refresh" ("Reload").
Do the problem yourself first!

x y x+y 5 2 3
a) + = b) − =
3 3 3 x x x

x x + 1 2x + 1 3x − 4 x−5 4x − 9
c) + = d) + =
x−1 x−1 x−1 x−4 x−4 x−4

6x + 1 4x + 5 6x + 1 − 4x − 5 2x − 4
e) − = =
x−3 x−3 x−3 x−3

2x − 3 x−4 2x − 3 − x + 4 x+1
f) − = =
x−2 x−2 x−2 x−2
Different denominators -- The LCM
To add fractions with different denominators, we must learn how to construct the Lowest
Common Multiple of a series of terms.
The Lowest Common Multiple (LCM) of a series of terms
is the smallest product that contains every factor from every term.
For example, consider this series of three terms:
pq pr ps
We will now construct their LCM -- factor by factor.
To begin, it will have the factors of the first term:
LCM = pq
Moving on to the second term, the LCM must have the factors pr. But it already has the
factor p -- therefore, we need add only the factor r:
LCM = pqr
Finally, moving on to the last term, the LCM must contain the factors ps. But again it
has the factor p, so we need add only the factor s:
LCM = pqrs.
That product is the Lowest Common Multiple of pq, pr, ps. It is the smallest product
that contains each of them as factors.
Example 3. Construct the LCM of these three terms: x, x2, x3.
Solution. The LCM must have the factor x.
LCM = x
But it also must have the factors of x2 -- which are x ·x. Therefore, we must add one
more factor of x :
LCM = x2
Finally, the LCM must have the factors of x3, which are x· x· x. Therefore,
LCM = x3.
x3 is the smallest product that contains x, x2, and x3 as factors.
We see that when the terms are powers of a variable -- x, x2, x3 -- then their LCM is the
highest power.
Problem 2. Construct the LCM of each series of terms.
a) ab, bc, cd. abcd b) pqr, qrs, rst. pqrst

c) a, a2, a3, a4. a4 d) a2b, ab2. a2b2


e) ab, cd. abcd
We will now see what this has to do with adding fractions.

3 4 5
Example 4. Add: + +
ab bc cd
Solution. To add fractions, the denominators must be the same. Therefore, as a common
denominator choose the LCM of the original denominators. Choose abcd. Then, convert
each fraction to an equivalent fraction with denominator abcd.
It is necessary to write the common denominator only once:
3 4 5 3cd + 4ad + 5ab
+ + =
ab bc cd abcd
3
To change into an equivalent fraction with denominator abcd,
ab
simply multiply ab by the factors it is missing, namely cd. Therefore, we must also multiply 3
by cd. That accounts for the first term in the numerator.
4
To change into an equivalent fraction with denominator abcd,
bc
multiply bc by the factors it is missing, namely ad. Therefore, we must also multiply 4
by ad. That accounts for the second term in the numerator.
5
To change into an equivalent fraction with denominator abcd,
cd
multiply cd by the factors it is missing, namely ab. Therefore, we must also multiply 5
by ab. That accounts for the last term in the numerator.
That is how to add fractions with different denominators.
Each factor of the original denominators must be a factor
of the common denominator.
Problem 3. Add.

5 6 5c + 6b
a) + =
ab ac abc

2 3 4 2rs + 3ps + 4pq


b) + + =
pq qr rs pqrs

7 8 9 7c + 8a + 9
c) + + =
ab bc abc abc
1 2 3 a2 + 2a + 3
d) + 2 + 3 =
a a a a3

3 4 3b + 4a
e) + 2=
ab
2 ab a2b2

5 6 5cd + 6ab
f) + =
ab cd abcd

_2_ __3__ 2(x − 3) + 3x


g) + =
x(x + 2) (x + 2)(x − 3) x(x + 2)(x − 3)

_ 2x − 6 + 3x_
=
x(x + 2)(x − 3)

_5x − 6_
=
x(x + 2)(x − 3)

At the 2nd Level we will see a similar problem, but the denominators will not
be factored.

1 c+1
Problem 4. Add: 1 − + . But write the answer as
a ab
1 − A fraction.

1 c+1 1 c+1 b − (c + 1) b−c−1


1− + =1−( − )=1− =1−
a ab a ab ab ab
Example 5. Denominators with no common factors.
a b
+
m n
When the denominators have no common factors, their LCM is simply their
product, mn.
a b an + bm
+ =
m n mn
The numerator then appears as the result of "cross-multiplying" :
an + bm
However, that technique will work only when adding two fractions, and the
denominators have no common factors.

2 1
Example 6. −
x−1 x
Solution. These denominators have no common factors -- x is not a factor of x − 1. It is a
term. Therefore, the LCM of denominators is their product.

2 1 2x − (x − 1) 2x − x + 1 _x + 1_
− = = =
x−1 x (x − 1)x (x − 1)x (x − 1)x

Note: The entire x − 1 is being subtracted. Therefore, we write it in parentheses --


and its signs change.
Problem 5.
x y xb + ya x 3x 2x + 15x 17x
a) + = b) + = =
a b ab 5 2 10 10

6 3 6(x + 1) + 3(x − 1)
c) + =
x−1 x+1 (x + 1)(x − 1)

6x + 6 + 3x − 3
=
(x + 1)(x − 1)

_9x + 3_
=
(x + 1)(x − 1)

6 3 6(x + 1) − 3(x − 1)
d) − =
x−1 x+1 (x + 1)(x − 1)

6x + 6 − 3x + 3
=
(x + 1)(x − 1)
_3x + 9_
=
(x + 1)(x − 1)

3 2 3x − 2(x − 3)
e) − =
x−3 x (x − 3)x

3x − 2x + 6
=
(x − 3)x

x+6
=
(x − 3)x

3 1 3x − (x − 3)
f) − =
x−3 x (x − 3)x

3x − x + 3
=
(x − 3)x

2x + 3
=
(x − 3)x

1 2 3 yz + 2xz + 3xy
g) + + =
x y z xyz

b
Example 7. Add: a + .
c
Solution. We have to express a with denominator c.
ac
a= (Lesson 20)
c
Therefore,
b ac + b
a+ = .
c c
Problem 6.

p p + qr 1 1−x
a) + r = b) − 1 =
q q x x

1 x2 − 1 1 x2 − 1
c) x − = d) 1 − =
x x x2 x2

1 x+1−1 x
e) 1 − = =
x+1 x+1 x+1

2 3x + 3 + 2 3x + 5
f) 3 + = =
x+1 x+1 x+1

1 1
Problem 7. Write the reciprocal of 2 + 3 .

a b
[Hint: Only a single fraction has a reciprocal; it is .]
b a

1 1 3+2 5
+ = =
2 3 6 6

6
Therefore, the reciprocal is 5 .

2nd Level

EQUATIONS
WITH FRACTIONS
Clearing of fractions
2nd Level

TO SOLVE AN EQUATION WITH fractions, we transform it into an equation without


fractions -- which we know how to solve. The technique is called clearing of fractions.
Example 1. Solve for x:
x x−2
+ = 6.
3 5
Solution. Clear of fractions as follows:
Multiply both sides of the equation -- every term -- by the LCM of
denominators. Each denominator will then divideinto its multiple. We will then
have an equation without fractions.
The LCM of 3 and 5 is 15. Therefore, multiply both sides of the equation by 15.
x x−2
15· + 15· = 15· 6
3 5
On the left, distribute 15 to each term. Each denominator will now divide into 15 --
that is the point -- and we have the following simple equation that has been "cleared" of
fractions:
5x + 3(x − 2) = 90.
It is easily solved as follows:
5x + 3x − 6 = 90
8x = 90 + 6
96
x=
8
= 12.
We say "multiply" both sides of the equation, yet we take advantage of the
fact that the order in which we multiply or divide does not matter. (Lesson
1.) Therefore we divide the LCM by each denominator first, and in that way
clear of fractions.
We choose a multiple of each denominator, because each denominator will
then be a divisor of it.

Example 2. Clear of fractions and solve for x:


x 5x 1
− =
2 6 9
Solution. The LCM of 2, 6, and 9 is 18. (Lesson 23 of Arithmetic.) Multiply both sides by
18 -- and cancel.
9x − 15x = 2.
It should not be necessary to actually write 18. The student should
simply look at and see that 2 will go into 18 nine (9) times. That term therefore becomes
9x.
Next, look at , and see that 6 will to into 18 three (3) times. That term therefore
becomes 3· −5x = −15x.
Finally, look at , and see that 9 will to into 18 two (2) times. That term therefore
becomes 2 · 1 = 2.
Here is the cleared equation, followed by its solution:
9x − 15x = 2

−6x = 2

2
x=
−6

1
x=−
3
Example 3. Solve for x:
½(5x − 2) = 2x + 4.
Solution. This is an equation with a fraction. Clear of fractions by mutiplying both sides by
2:
5x − 2 = 4x + 8

5x − 4x = 8 + 2

x = 10.

In the following problems, clear of fractions and solve for x:


To see each answer, pass your mouse over the colored area.
To cover the answer again, click "Refresh" ("Reload").
Do the problem yourself first!

x x
Problem 1. − =3
2 5

The LCM is 10. Here is the cleared equation and its solution:

5x − 2x = 30

3x = 30

x = 10.
On solving any equation with fractions, the very next line you write --
5x − 2x = 30
-- should have no fractions.

x 1 x
Problem 2. = +
6 12 8

The LCM is 24. Here is the cleared equation and its solution:

4x = 2 + 3 x

4 x − 3x = 2

x= 2

x−2 x x
Problem 3. + =
5 3 2

The LCM is 30. Here is the cleared equation and its solution:

6(x − 2) + 10x = 15x

6x − 12 + 10x = 15x

16x − 15x = 12

x = 12.
Problem 4. A fraction equal to a fraction.
x−1 x
=
4 7

The LCM is 28. Here is the cleared equation and its solution:

7(x − 1) = 4x

7x − 7 = 4x

7 x − 4x = 7

3x = 7

7
x= 3
We see that when a single fraction is equal to a single fraction, then the equation can be
cleared by "cross-multiplying."
If
a c
= ,
b d
then
ad = bc.

x−3 x−5
Problem 5. =
3 2

Here is the cleared equation and its solution:

2(x − 3) = 3(x − 5)

2x − 6 = 3x − 15
2x − 3x = − 15 + 6

−x = −9

x= 9
x−3 x+1
Problem 6. =
x−1 x+2

Here is the cleared equation and its solution:

(x − 3)(x + 2) = (x − 1)(x + 1)

x² −x − 6 = x² − 1

−x = −1 + 6

−x = 5

x = −5.

2x − 3 x+1
Problem 7. + = x−4
9 2

The LCM is 18. Here is the cleared equation and its solution:

4x − 6 + 9x + 9 = 18x − 72
13x + 3 = 18x − 72

13x − 18x = − 72 − 3

−5x = −75

x = 15.
2 3 1
Problem 8. − =
x 8x 4

The LCM is 8x. Here is the cleared equation and its solution:

16 − 3 = 2x

2x = 13

13
x= 2

2nd Level

WORD PROBLEMS
THAT LEAD TO
EQUATIONS WITH FRACTIONS
2nd Level:
Same time problem
Total time problem
Job problem

HERE ARE SOME WORD PROBLEMS that lead to equations with fractions.
Example 1. Half of a number, added to a fifth of three less than the number, is equal to
two thirds the number. What is the number?
Solution. As in Lesson 10, always let x represent the unknown number that we are asked to
find. Let x answer the question.
x
In algebra, we express "half of a number" as 2 . Similarly, a third of a
x
number will be 3
. And so on.
Here is the equation:
x x−3 2
+ = 3x
2 5
The LCM of denominators is 30. And upon clearing of fractions, we have
15x + 6(x − 3) = 20x

15x + 6x − 18 = 20x

21x − 20x = 18

x = 18.

Example 2. The whole is equal to the sum of the parts.


This problem is from a classical Hindu text on algebra from the 9th century.
During an amorous struggle, the lady's pearls broke. Half of the pearls fell onto the
floor; a fourth rolled under a chair; a sixth fell into her lap; and three pearls remained on the
strand. How many were there originally on the strand?
Solution. Let x be that total number. And let us account for them all.
First, we will clear the equation of fractions. The LCM of 2, 4, and 6 is 12. We will
multiply every term by 12, and cancel the denominators:
x x x
x= + + + 3
2 4 6

12x = 6x + 3x + 2x + 36

12x = 11x + 36

12x − 11x = 36

x = 36
There were originally 36 pearls on the strand.
Example 3. One number is three times another. The sum of their reciprocals is 4. What
are the two numbers?
Solution. Let x be the first number. Then the other is three times that: 3x. The problem
states that the sum of their reciprocals is 4:
1 1
+ = 4
x 3x

Clear of fractions. The LCM is 3x:

3 + 1 = 3x· 4

That is,
12x = 4

4
x= 12

1
= 3
1 1
3
is the first number. The other number is 3· 3 = 1.
(Note that the sum of their reciprocals -- 3 + 1 -- does equal 4.)
Problem 1. Half of a number added to its third part, is eight less than the number. What is
the number?
To see the equation, pass your mouse over the colored area.
To cover the answer again, click "Refresh" ("Reload").
But do the problem yourself first!
In every case, let x represent the unknown number.
The equation is:
x x
+ = x−8
2 3

The LCM is 6. Here is the cleared equation and its solution:

3x + 2x = 6x − 48

5x − 6x = −48

−x = −48

x = 48
Problem 2. Three fifths of a number, plus 8, is equal to the number. What is the number?
Here is the equation:
3
5
x+ 8 = x

The LCM is 5. Here is the cleared equation and its solution:

3x + 40 = 5x

3x − 5x = −40
−2x = −40

x = 20
Problem 3. Half of a number, plus a fifth of two less than the number, is four less than the
number. What is the number?
Here is the equation:
x x−2
+ = x−4
2 5

The LCM is 10. Here is the cleared equation and its solution:

5x + 2x − 4 = 10x − 40

7x − 10x = −40 + 4

−3x = −36

x = 12
Problem 4. Melissa went shopping and spent half of her money on shoes, a third on a
blouse, a tenth to take her boyfriend to lunch, and she came home with $12. How much did
she start out with?
Here is the equation:

x = 2x + x3 + 10
x
+ 12

The LCM is 30. Here is the cleared equation and its solution:

30x = 15x + 10x + 3x + 360


30x − 28x = 360

2x = 360

x = 180
Problem 5. One number is four times another. The sum of their
5
reciprocals is 12 . What are the two numbers?

Let x be the first number. Then the other is 4x. Here is the equation:

1 1 5
+ = 12
x 4x

The LCM is 12x. Here is the cleared equation and its solution:

12 + 3 = 5x

5x = 15

x = 3.

4x = 12.
These are the two numbers.
Problem 6 If the same number is added to the numerator and
7 5
denominator of 9 the result is 6 . What is that number?
Here is the equation:
7+x 5
=
9+x 6

On cross multiplying, here is the cleared equation and its solution:


(7 + x)6 = (9 + x)5

42 + 6x = 45 + 5x

6x − 5x = 45 − 42

x=3

2nd Level:

Radicals: Rational and Irrational Numbers

The square numbers


The radical sign and the radicand
Rational and irrational numbers
Which square roots are rational?
An equation x² = a, and the principal square root
2nd level:
Equations (x + a)² = b
The definition of the square root radical
Rationalizing a denominator
Real numbers

HERE ARE THE FIRST TEN square numbers and their roots:
Square numbers 1 4 9 16 25 36 49 64 81 100
Square roots 1 2 3 4 5 6 7 8 9 10
We write, for example,
= 5.
"The square root of 25 is 5."
This mark is called the radical sign (after the Latin radix = root). The number under
the radical sign is called the radicand. In the example, 25 is the radicand.
Problem 1. Evaluate the following.
To see the answer, pass your mouse over the colored area.
To cover the answer again, click "Refresh" ("Reload").
Do the problem yourself first!

a) = 8 b) = 12 c) = 20

7
d) = 17 e) = 1 f) = 9

Example 1. Evaluate .
Solution. = 13.
For, 13· 13 is a square number. And the square root of 13· 13 is 13
If a is any whole number, then a· a is a square number, and

Problem 2. Evaluate the following.


a) = 28. b) = 135.

c) = 2· 3· 5 = 30.
We can state the following theorem:
A square number times a square number is itself a square number.
For example,
36· 81 = 6· 6· 9· 9 = 6· 9· 6· 9 = 54· 54
Problem 3. Without multiplying the given square numbers, each product of square
numbers is equal to what square number?
a) 25· 64 = 5· 8· 5· 8 = 40· 40
b) 16· 49 = 4· 7· 4· 7 = 28· 28
c) 4· 9· 25 = 2· 3· 5· 2· 3· 5 = 30· 30
Rational and irrational numbers
The rational numbers are the everyday numbers of arithmetic: the whole numbers, fractions,
mixed numbers, and decimals; together with their negative images. A rational number has
the same ratio to 1 as two natural numbers.
That is what a rational number is. As for what it looks like, it can take the
form , where a and b are integers (b ≠ 0).

Problem 4. Which of the following numbers are rational?


4 13
1 −6 3½ 5
− 5 0 7.38609
All of them.
At this point, the student might wonder, What is a number that is not rational?
An example of such a number is ("Square root of 2"). is not a number of
arithmetic. We cannot name any whole number, any fraction
7
or any decimal whose square is 2. 5 is close, because
7 7 49
·
5 5
= 25
-- which is almost 2.
But to prove that there is no rational number whose square is 2, then suppose there
were. Then we could express it as a fraction in lowest terms. That is, suppose
· = 2.
But that is impossible. Since is in lowest terms, then m and n have no common
divisors except 1. Therefore, m· m and n· n also have no common divisors -- they
are relatively prime -- and it will be impossible to divide n· n into m· m and get 2.
There is no rational number whose square is 2. Therefore we call
an irrational number.
Question. The square roots of which natural numbers are rational?
Answer. Only the square roots of square numbers.
=1 Rational

Irrational

Irrational

=2 Rational

, , , Irrational
= 3 Rational
And so on.
The square roots of the square numbers are the only square roots that we can name.
The existence of irrationals was first realized by Pythagoras in the 6th century B.C. He
called them "without a name." For if we ask, " How much is ? -- we cannot say. We can
only call it, "Square root of 2."
Problem 5. Say the name of each number.
a) Square root of 3. b) Square root of 8. c) 3.
2
d) 5
e) Square root of 10

As for the decimal representation of both irrational and rational numbers, see Topic 2
of Precalculus.
An equation x² = a, and the principal square root
Example 2. Solve this equation:
x² = 25.

Solution. x = 5 or −5, because (−5)² = 25, also.

In other words,
x= or − .
We say however that the positive value 5 is the principal square root. That is, we say that
"the square root of 25" is 5.
= 5.
As for −5, it is "the negative of the square root of 25."
− = −5.
Thus the symbol refers to one non-negative number.
Example 3. Solve this equation:
x² = 10.

Solution. x= or − .

Always, if an equation looks like this,


x² = a,

then the solution will look like this:

x= or − .

We often use the double sign ± ("plus or minus") and write:


x=± .
Problem 6. Solve for x.

a) x² = 9 implies x = ±3 b) x² = 144 implies x = ±12

c) x² = 5 implies x = ± d) x² = 3 implies x = ±

e) x² = a − b implies x = ±

2nd Level

SIMPLIFYING RADICALS
Simplest form
Similar radicals
2nd level

Simplifying powers
Factors of the radicand
Fractional radicand

WE SAY THAT A SQUARE ROOT RADICAL is simplified, or in its simplest form, when
the radicand has no square factors.
A radical is also in simplest form when the radicand is not a fraction.

Example 1. 33, for example, has no square factors. Its factors are 3· 11, neither of which
is a square number. Therefore, is in its simplest form.
Example 2. Extracting the square root. 18 has the square factor 9.
18 = 9· 2.
Therefore, is not in its simplest form. We have,
=
We may now extract, or take out, the square root of 9:
= =3 .
is now simplified. The radicand no longer has any square factors.
The justification for taking out the square root of 9, is this theorem:

The square root of a product


is equal to the product of the square roots
of each factor.
(We will prove that when we come to rational exponents, Lesson 29.
Here is a simple illustration: )
As for , then, it is equal to the square root of 9 times the square root of 2, which
is irrational. 3 .

Example 3 Simplify .
Solution. = =5 .
75 has the square factor 25. And the square root of 25 times 3
is equal to the square root of 25 times the square root of 3.
is now simplified.

Example 4. Simplify .
Solution. We have to factor 42 and see if it has any square factors. We can begin the
factoring in any way. For example,
42 = 6· 7
We can continue to factor 6 as 2· 3, but we cannot continue to factor 7, because 7 is
a prime number (Lesson 32 of Arithmetic). Therefore,
42 = 2· 3· 7
We now see that 42 has no square factors -- because no factor is repeated. Compare
Example 1 and Problem 2 of the previous Lesson.
therefore is in its simplest form.

Example 5. Simplify .
Solution. We must look for square factors, which will be factors that are repeated.
180 = 2· 90 = 2· 2· 45 = 2· 2· 9· 5 = 2· 2· 3· 3· 5
Therefore,
= 2· 3 = 6 .
Problem 1. To simplify a radical, why do we look for square factors?
To see the answer, pass your mouse over the colored area.
To cover the answer again, click "Refresh" ("Reload").
Do the problem yourself first!
In order to take its square root out of the radical.

Problem 2. Which is correct?

Problem 3. Simplify the following. Do that by inspecting each radicand for a square
factor: 4, 9, 16, 25, and so on.
a) =
b) = = =5
c) = = =3
d) = =7
e) = =4
f) = = 10
g) = =5
h) = =4
Problem 4. Reduce to lowest terms.

a) = = =
2 2 2

b) = = =2
3 3 3

c) = The radical is in its simplest form. The fraction cannot be reduced.


2
Similar radicals
Similar radicals have the same radicand. We add them as like terms.
7+2 +5 +6 − =7 + 2 +6 +5 −

=7 + 8 +4 .
2 and 6 are similar, as are 5 and − . We combine them by adding their
coefficients.
In practice, it is not necessary to change the order of the terms. The student should
simply see which radicals have the same radicand.
As for 7, it does not "belong" to any radical.
Problem 5. Simplify each radical, then add the similar radicals.
a) + =3 +2 =5

b) 4 −2 + =4 −2 +

= 4· 5 − 2· 7 +

= 20 − 14 +

=7

c) 3 + −2 =3 + −2

= 3· 2 +2 − 2· 4

=6 +2 −8

=2 −2

d) 3 + + =3 + +

=3 + 2 +3

=3 + 5

e) 1 − + =1 − +

=1 − 8 +3

=1 − 5
Problem 6. Simplify the following.
a) = =2 − ,
2 2 on dividing every term in the numerator by 2.
Compare Example 4 here.
To see that 2 was a factor of the radical, you first have to simplify the
radical. Compare Problem 4.

b) = = 2+
5 5

c) = = on dividing every term by 2.


6 6 3

2nd Level

MULTIPLYING AND DIVIDING


RADICALS
Conjugate pairs

HERE IS THE RULE for multiplying radicals:

It is the symmetrical version of the rule for simplifying radicals. It is valid for a and b greater
than or equal to 0.
Problem 1. Multiply.
To see the answer, pass your mouse over the colored area.
To cover the answer again, click "Refresh" ("Reload").
Do the problem yourself first!

a) · = b) 2 ·3 =6

c) · = =6 d) (2 )2 = 4· 5 = 20
e) =
The difference of two squares
Problem 2. Multiply, then simplify:
Example 1. Multiply ( + )( − ).
Solution. The student should recognize the form those factors will produce:
The difference of two squares
( + )( − )=( )2 − ( )2

=6 − 2

= 4.
Problem 3. Multiply.
a) ( + )( − ) = 5−3=2
b) (2 + )(2 − ) = 4· 3 − 6 = 12 − 6 = 6
c) (1 + )(1 − ) = 1 − (x + 1) = 1 − x − 1 = −x
d) ( + )( − ) = a−b
Problem 4. (x − 1 − )(x − 1 + )
a) What form does that produce?
The difference of two squares. x − 1 is "a." is "b.">
b) Multiply out.
(x − 1 − )(x − 1 + ) = (x − 1)2 − 2

= x2 − 2x + 1 − 2, on squaring the binomial,

= x 2 − 2x − 1
Problem 5. Multiply out.
(x + 3 + )(x + 3 − ) = (x + 3)2 − 3

= x 2 + 6x + 9 − 3
= x 2 + 6x + 6
Dividing radicals

For example,
= =

Problem 6. Simplify the following.


3
a) = b) = c) = a = a· a = a2
8 4

Conjugate pairs
The conjugate of a + is a − . They are a conjugate pair.
Example 2. Multiply 6 − with its conjugate.
Solution. The product of a conjugate pair --
(6 − )(6 + )
-- is the difference of two squares. Therefore,
(6 − )(6 + ) = 36 − 2 = 34.
When we multiply a conjugate pair, the radical vanishes and we obtain
a rational number.
Problem 7. Multiply each number with its conjugate.
a) x + = x2 − y
b) 2 − (2 − )(2 + )=4−3=1

c) + You should be able to write the product immediately: 6 − 2 = 4.


d) 4 − 16 − 5 = 11
Example 3. Rationalize the denominator:
1

Solution. Multiply both the denominator and the numerator by the conjugate of the
denominator; that is, multiply them by 3 − .
1
= =
9−2 7
The numerator becomes 3 − . The denominator becomes the difference of the two
squares.

Example 4. = =
3−4 −1

= −(3 − 2 )

=2 − 3.
Problem 8. Write out the steps that show the following.
1
a) = ½( )

= = = ½( − )
5−3 2

The definition of division


2
b) = ½(3 − )
3+

2
= = = ½(3 − )
3+ 9−5 4

_7_
c) =
3 + 6

_7_
= = =
3 + 9· 5 − 3 42 6

d) = 3+2
−1

= = 2+2 + 1, Perfect square trinomial


2−1

= 3+2
e) =
1+ x

=
1+ 1 − (x + 1)

= , Perfect square trinomial


1−x−1

=
−x

= on changing all the signs.


x

Example 5. Simplify

Solution. = on adding those fractions,

= on taking the reciprocal,

= on multiplying by the conjugate,


6−5

=6 −5 on multiplying out.

Problem 9. Simplify
= on adding those fractions,

= on taking the reciprocal,

= on multiplying by the conjugate,


3−2

=3 +2 on multiplying out.
Problem 10. Here is a problem that comes up in Calculus. Write out the steps that show:

____1____
= −

In this case, you will have to rationalize the numerator.

1
= ·
h

1 _____x − (x + h)_____
= ·
h

1 ____x − x − h_____
= ·
h

1 _______−h_______
= ·
h
_______ 1_______
= −

COMPLEX
OR IMAGINARY
NUMBERS
The defining property of i
The square root of a negative number
Powers of i
Algebra with complex numbers
The real and imaginary components
Complex conjugates

IN ALGEBRA, we want to be able to say that every polynomial equation has a solution;
specifically, this one:
x2 + 1 = 0.
That implies,
x2 = −1.
But there is no real number whose square is negative. (Lesson 13.) Therefore, we invent
a number -- a complex or imaginary number -- and we call it i.
i2 = −1.
That is the defining property of the complex unit i. Its square is negative.
In other words,
i = .
The complex number i is purely algebraic. That is, we call it a "number"
because it will obey all the rules we normally associate with a number. We
may add it, subtract it, multiply it, and so on. The complex number iturns
out to be extremely useful in mathematics and physics.

Example 1. 3i· 4i = 12i2 = 12(−1) = −12.


Example 2. −5i· 6i = −30i2 = 30.
We see, then, that the factor i2 changes the sign of a product.
Problem 1. Evaluate the following.
To see the answer, pass your mouse over the colored area.
To cover the answer again, click "Refresh" ("Reload").
Do the problem yourself first!

a) i2 = −1 b) i· 2i = 2i2 = 2(−1) = −2

c) (3i)2 = 32i2 = −9 d) −5i· 4i = −20i2 = 20


The square root of a negative number
If a radicand is negative --
, where a > 0,
-- then we can simplify it as follows:
= = = i .
In other words:
The square root of −a is equal to i times the square root of a.

Examples 3. =i

=i = 2i

=i = 2i
Problem 2. Express each of the following in terms of i.
a) = i b) = 3i c) = 7i

d) = i e) = i f) = 2i

g) = 3i h) = 5i i) = 7i
Powers of i
Let us begin with i0, which is 1. (Any number with exponent 0 is 1.) Each power of i can be
obtained from the previous power by multiplying it by i. We have:
i0 = 1
i1 = i
i2 = −1
i3 = −1· i = −i
i4 = −i· i = −i2 = −(−1) = 1
And we are back at 1 -- the cycle of powers will repeat. Any power of i will be either
1, i, −1, or −i
-- according to the remainder upon dividing the exponent n by 4.

Examples 4.

i9 = i, because on dividing 9 by 4, the remainder is 1.

i9 = i1 .

i18 = −1, because on dividing 18 by 4, the remainder is 2.

i18 = i2.

i35 = −i, because on dividing 35 by 4, the remainder is 3.

i35 = i3.

i40 = 1, because on dividing 40 by 4, the remainder is 0.

i40 = i0.
Note: Even powers of i will be either 1 or −1, according as the exponent is a multiple
of 4, or 2 more than a multiple of 4. While odd powers will be either i or −i.
Problem 3. Evaluate each power of i.
a) i3 = −i b) i4 = 1 c) i6 = i2 = −1

d) i9 = i1 = i e) i12 = i0 = 1 f) i17 = i1 = i

g) i27 = i3 = −i h) i30 = i2 = −1 i) i100 = i0 = 1


Problem 4.

Algebra with complex numbers


Complex numbers follow the same rules as real numbers. For example, to multiply
(2 + 3i)(2 − 3i)
the student should recognize the form (a + b)(a − b) -- which will produce the difference of
two squares. Therefore,
(2 + 3i)(2 − 3i) = 4 − 9i2

= 4 − 9(−1)

=4 + 9

= 13.
Again, the factor i2 changes the sign of the term.
Problem 5. Multiply.
a) (1 + i )(1 − i ) = 1 − 2i2 = 1 + 2 = 3
b) (3 − i )2 = 9 − 6i + 2i2, upon squaring the binomial,

= 9 − 6i −2

= 7 − 6i
c) (2 + 3i)(4 − 5i) = 8 − 10i + 12i − 15i2

= 8 + 2i + 15

= 23 + 2i
Problem 6. (x + 1 + 3i)(x + 1 − 3i)
a) What form will that produce? The difference of two squares.
Lesson 19.
b) Multiply out.
(x + 1 + 3i)(x + 1 − 3i) = (x + 1)2 − 9i2
= x 2 + 2x + 1 + 9
= x2 + 2x + 10

c) (x − 2 − i )(x − 2 + i ) = (x − 2)2 − 2i2

= x 2 − 4x + 4 + 2

= x 2 − 4x + 6
The real and imaginary components
Here is the standard form of a complex number:
a + bi.
Both a and b are real. For example,
3 + 2i.
a -- that is, 3 in the example -- is called the real component (or the real part). b (2 in the
example) is called the imaginary component (or the imaginary part). Again, the components
are real.
Problem 7. Name the real component a and the imaginary component b.
a) 3 − 5i a = 3, b = −5. b) 1 + i a = 1, b = .

c) i a = 0, b = 1. d) −6 a = −6, b = 0.
Complex conjugates
The complex conjugate of a + bi is a − bi. The main point about a conjugate pair is that
when they are multiplied --
(a + bi)(a − bi)
-- the product is a positive real number. For, that form is the difference of two squares:
(a + bi)(a − bi) = a2 − b2i2 = a2 + b2
The product of a complex conjugate pair
is equal to the sum of the squares of the components.
Problem 8. Calculate the positive real number that results from multiplying each number
with its complex conjugate.
a) 2 + 3i
(2 + 3i)(2 − 3i) = 22 + 32 = 4 + 9 = 13.
b) 3 − i .
(3 − i )(3 + i ) = = 32 + ( )2= 9 + 2 = 11.
c) u + iv. (u + iv)(u − iv) = u2 + v2.
d) 1 + i. (1 + i)(1 − i) = 12 + 12 = 2.
e) −i. (−i)(i) = −i2 = 1.

RECTANGULAR COÖRDINATES
Straight lines

WE WILL BEGIN with vocabulary.


First, a coördinate. A coördinate is a number. It labels a point on a line.

The coördinate 0 is called the origin of coördinates. Distances to the right of 0 are
labeled with positive coördinates: 1, 2, 3, etc. Distances to the left are labeled with negative
numbers: −1, −2, −3, etc. Each coördinate is the "address" of a distance and direction from
0.
A coördinate axis is a line with coördinates.
Now, to label a point in a plane (a flat surface), we will need more than one coördinate
axis, and so we place a second at right angles to the first.
Distances above the origin will have positive coördinates; distances below, negative
coördinates.
Those axes are called rectangular coördinate axes, because they are at right angles to
one another. The coördinates on them are called rectangular coördinates. They are also
called Cartesian coördinates, after the 17th century philosopher and mathematician René
Descartes; for he was one of the first to realize the possibility of solving problems of
geometry by means of algebra with the coördinates. Hence we have the name coördinate
geometry or, as it is often called, analytic geometry.
Rectangular coördinates are an ordered pair, (x, y).

The pair (2, 3)—over 2 and up 3—labels a different point than (3, 2): over 3 and up
2. The horizontal coördinate—Right or left—is always entered first. The vertical
coördinate—Up or down—is always entered second. For that reason, (3, 2) is called
an ordered pair.
The coördinates of the origin are (0, 0). We don't move right or left and we don't move
up or down. We will see that 0 is an extremely important coördinate. It means that the
point is on one of the axes.
Now the horizontal axis is always called the x-axis, and the vertical axis is always called
the y-axis. The first coördinate, then, is called the x-coördinate; the second is called the y-
coördinate. We always write (x, y).
Finally, the coördinate axes divide the plane into four quadrants:

The first, the second, the third, and the fourth. We label the quadrants counter-clockwise.
Problem 1. Name the coördinates of each point.
To see the answer, pass your mouse over the colored area.
To cover the answer again, click "Refresh" ("Reload").
Do the problem yourself first!

a) b) c)

(2, 3) (3, 2) (−1, 3)

d) e) f)

(3, −1) (−2, −1) (0, 3)


g) h) i)

(3, 0) (−2, 0) (0, −2)


Problem 2. Coördinate 0.
a) On the x-axis, what is the value of every y-coördinate? 0

On the x-axis, we don't move up or down. At every point, y = 0.


b) On the y-axis, what is the value of every x-coördinate? 0

On the y-axis, we don't move right or left. At every point, x = 0.


c) Where is the y-coördinate always 0? On the x-axis.
d) Where is the x-coördinate always 0? On the y-axis.
Problem 3.
a) Where is the x-coördinate always 2?
On the vertical line 2 units to the right of the origin.
In fact, we say that that vertical line is the graph of the equation
x = 2.
That is the equation, because at every point of that line, the x-coördinate is 2.
b) Where is the y-coördinate always −3?
On the horizontal line 3 units below the origin.

That line is called the graph of y = −3. And y = −3 is called the equation of that line.
Problem 4. In which quadrant does each point lie? Or is it on an axis; if so, which axis?
a) (2, −3) Fourth b) (−4, 2) Second

c) (0, −5) On the y-axis. d) (−3, −1) Third

e) (5, 0) On the x-axis. f) (−6, 9) Second

g) (0, −4) On the y-axis. h) (−4, 0) On the x-axis.

i) (−1, −1) Third j) (0, 6) On the y-axis.


k) (−1, 0) On the x-axis. l) (0, 1) On the y-axis.

m) (5, −2) Fourth n) (−5, 0) On the x-axis.

The extremities of a straight line AB have coördinates (4, 3) and (15, 8), and that line is
the hypotenuse of a right triangle ACB, whose sides are parallel to the axes.
Name the coördinates of the right angle at C.
C has the same x-coördinate as B. Therefore its x-coördinate is 15. And C
has the same y-coördinate as A. Therefore its y-coördinate is 3. The coördinates
at C are (15, 3).
Straight lines
An actually infinite straight line is a line that has no extremities—no endpoints.

Obviously, such a line is a mental object---an idea---only. It is not possible to witness


one or draw one.
(Similarly, it is not possible to produce an actually infinite sequence of digits, as this
symbol, "0.36363636. . . ," is meant to signify. That, too, is a purely mental object; it is an
idea.)
And so the student should be advised that when writers use the expression "straight
line" these days, they invariably mean an actually infinite line. Hence they refer to any finite
line, with its two extremities, as a line segment. They imagine that every finite straight line—
even the side of a square you might draw—is a segment of an actually infinite line.
That is what we are taught, and so we think that's mathematics—that's the way things
are. We do not realize that it is a human invention.
In classical plane geometry a straight line has two extremities.

It is potentially infinite in the sense that we may extend it in either direction for as far as
we please. Which in practice is all we ever require.
A line—length with no width—is an idea in the first place. As an idea, it clearly exists.
But should that be sufficient for mathematics? Should we not be able to make an image of it
in the physical world?
Euclid, in his first Postulate, has answered that. He has asked us to grant that we can do
the following:
To draw a straight line from any point to any point.

In other words, he asks us to regard what we have drawn to be a straight line. By means
of that Postulate, a line will then exist logicallly. We will have brought a representative of that
idea into this world.
See First Principles of Euclidean Geometry, Commentary on the Definitions.
A ray is also a mental object. It is the idea of a straight line with one extremity.

THE PYTHAGOREAN DISTANCE FORMULA


The distance of a point from the origin
The distance between any two points
A proof of the Pythagorean theorem

BASIC TO TRIGONOMETRY and calculus is the theorem that relates the squares
drawn on the sides of a right-angled triangle. Credit for the proving the
theorem goes to the Greek philosopher Pythagoras, who lived in the 6th
century B. C.
Here is the statement of the theorem:
In a right triangle the square drawn on the side opposite the right angle
is equal to the squares drawn on the sides that make the right angle.

That means that if ABC is a right triangle with the right angle at A, then the square
drawn on BC opposite the right angle, is equal to the two squares together on CA, AB.
In other words, if it takes one can of paint to paint the square on BC, then it will also
take exactly one can to paint the other two squares.
The side opposite the right angle is called the hypotenuse ("hy-POT'n-yoos"; which
literally means stretching under).
Algebraically, if the hypotenuse is c, and the sides are a, b:

a2 + b2 = c2.
For a proof, see below.
Problem 1. State the Pythagorean theorem in words.
In a right triangle the square on the side opposite the right angle will equal
the squares on the sides that make the right angle.
Problem 2. Calculate the length of the hypotenuse c when the sides are as follows.
To see the answer, pass your mouse over the colored area.
To cover the answer again, click "Refresh" ("Reload").
Do the problem yourself first!
a) a = 5 cm, b = 12 cm.
c2 = 52 + 122

= 25 + 144

= 169.

Therefore, c =

= 13 cm,
on simplifying the radical.
b) a = 3 cm, b = 6 cm.
c 2 = 2 + 62
= 9 + 36

= 9(1 + 4)

= 9· 5.

Therefore, c =

= cm.
Since 9 is a square number, and a common factor of 9 and 36, then we may anticipate
simplifying the radical by writing 9 + 36 = 9(1 + 4) = 9· 5.
We could, of course, have written 9 + 36 = 45 = 9· 5. But that first wipes out the
square number 9. We then have to bring it back.
The distance d of a point (x, y) from the origin

According to the Pythagorean theorem, and the meaning of the rectangular


coördinates (x, y),
d 2 = x2 + y2.
Therefore,

"The distance of a point from the origin


is equal to the square root of the
sum of the squares of the coördinates."
Example 1. How far from the origin is the point (4, −5)?

Problem 3. How far from the origin is the point (−5, −12)?
The distance between any two points

How far is it from (4, 3) to (15, 8)?

Consider the distance d as the hypotenuse of a right triangle. Then according to Lesson
31, Problem 5, the coördinates at the right angle are (15, 3).
Therefore, the horizontal leg of that triangle is simply the distance from 4 to 15: 15 −
4 = 11.

The vertical leg is the distance from 3 to 8: 8 − 3 = 5.


Therefore,
To find a formula, let us use subscripts and label the two points as
(x1, y1) ("x-sub-1, y-sub-1") and (x2, y2) ("x-sub-2, y-sub-2") .

The subscript 1 labels the coördinates of the first point; the subscript 2 labels the
coördinates of the second. We write the absolute value because distance is never negative.
Here then is the Pythagorean distance formula between any two points:

It is conventional to denote the difference of x-coördinates by the symbol Δx ("delta-


x"):
Δx = x2 − x1
Similarly,
Δy = y2 − y1
Therefore,

Example 2. Calculate the distance between the points (1, 3) and (4, 8).
Solution. Δx = 4 − 1 = 3.

Δy = 8 − 3 = 5.
Therefore,
Note: It does not matter which point we call the first and which the
second. Alternatively,
Δx = 1 − 4 = −3.
Δy = 3 − 8 = −5.
But (−3)2 = 9, and (−5)2 = 25. The distance between the two points is the same.
Example 3. Calculate the distance between the points (−8, −4) and (1, 2).
Solution. Δx = 1 − (−8) = 1 + 8 = 9.

Δy = 2 − (−4) = 2 + 4 = 6.
Therefore,

Problem 4. Calculate the distance between (2, 5) and (8, 1)

Problem 5. Calculate the distance between (−11, −6) and (−16, −1)
A proof of the Pythagorean theorem

Let a right triangle have sides a, b, and hypotenuse c. And let us arrange four of those triangles to form a
square whose side is a + b. (Fig. 1)
Now, the area of that square is equal to the sum of the four triangles, plus the interior square whose
side is c.
Two of those triangles taken together, however, are equal to a rectangle whose sides are a, b. The
area of such a rectangle is atimes b: ab. Therefore the four triangles together are equal to two such
rectangles. Their area is 2ab.
As for the square whose side is c, its area is simply c2. Therefore, the area of the entire square is
c2 + 2ab. . . . . . . (1)
At the same time, an equal square with side a + b (Fig. 2)
is made up of a square whose side is a, a square whose side is b, and two rectangles whose sides
are a, b. Therefore the area of that square is
a2 + b2 + 2ab.
But this is equal to the square formed by the triangles, line (1):
a2 + b2 + 2ab = c2 + 2ab.
Therefore, on subtracting the two rectangles 2ab from each square, we are left with
a2 + b2 = c2.
That is the Pythagorean theorem.

THE EQUATION AND GRAPH


OF A STRAIGHT LINE
Solutions to an equation of the first degree
The graph of a first degree equation
Constants versus variables
Section 2: Drawing the graph
The x- and y-intercepts of a graph
The form y = ax.
Vertical and horizontal lines
y = 2x + 6.
That is called an equation of the first degree. It is called that because the highest exponent is
1.
A solution to that equation will be any values of x and y that will make the equation --
that statement -- true.
To find a solution, simply let x have any value you please. The equation will then
determine the value of y.
For example, if we let x = 0, then
y = 2· 0 + 6 = 0 + 6 = 6.
The pair (0, 6) solves that equation.
Or, if we choose x = 3, then
y = 2· 3 + 6 = 6 + 6 = 12.
The pair (3, 12) also solves that equation. In fact, when there are two
unknowns, x and y, but only one equation that relates them, then there is no limit to the
number of solutions.
Since we typically first choose the value of x, we call x the independent variable. y will
be the dependent variable, because its value will depend on the value we have chosen for x.
Problem 1. Find three solutions to the first degree equation y = x + 5.
To see the answer, pass your mouse over the colored area.
To cover the answer again, click "Refresh" ("Reload").
Do the problem yourself first!
For example: (0, 5), (1, 6), (2, 7).
Problem 2. Which of the following ordered pairs solve this equation:
y = 3x − 4 ?
(0, −4) (1, 2) (1, −1) (2, −3)
(0, −4) and (1, −1). Because when x and y have those values, the equation is
true.
The graph of a first degree equation
Consider the equation
y = 2x + 1.
Since there are two variables, x and y, then will it be possible, on the x-y plane, to draw
a "picture" of all the solutions to that equation?
First, to find a few solutions, complete this table. That is, calculate the value of y that
corresponds to each value of x:
x y = 2x + 1
01
13
25
−1 −1
Now plot those ordered pairs as coördinates on the plane:
We see that all those solutions lie on a straight line. In fact, every pair (x, y) that solves
that equation will be the coördinates of a point on that line. On that line, every coördinate
pair is
(x, 2x + 1).
y = 2x + 1.
That line, therefore, is called the graph of the equation y = 2x + 1. And y = 2x + 1 is
called the equation of that line.
The graph of an equation, in other words, is the graph of its solutions. It is the picture
of those values of (x, y) that make the equation -- that statement -- true.
Every first degree equation -- where 1 is the highest exponent -- has as its graph a
straight line. (We prove that in Topics in Precalculus .)
For that reason, an equation of the first degree is called a linear equation.
Problem 3.
a) An equation of the form y = ax + b has what graph?
A straight line. This is a linear equation.
b) An equation of the form Ax + By + C = 0 has what graph?
A straight line. This is a linear equation. The capital letters are a
convention for indicating integer coefficients.
Problem 4. What characterizes a linear equation?
1 is the highest exponent.
Problem 5. Which of the following are linear equations?
a) y = 4x − 5 b) 2x − 3y + 8 = 0 c) y = x² − 2x + 1

d) 3x + 1 = 0 e) y = 6x + x3 f) y = 2
a), b), d), f).
Problem 6.
a) Name the coördinates of any three points on the line whose equation
a) is
y = 2x − 1.
(Choose any number for x; the equation will then determine the
a) value of y.)
For example, (0 −1), (1, 1), (−1, −3).
Problem 7.
a) Which of these ordered pairs solves the equation y = 5x − 6 ?
(You have to test each pair!)
(1, −2) (1, −1) (2, 3) (2, 4)
(1, −1) and (2, 4)
b) Which of those are points on the graph of y = 5x − 6 ?
(1, −1) and (2, 4)
Problem 8. True or false?
a) (−2, −3) is on the line whose equation is x + y = 5.
False.
b) (2, 3) is on the line whose equation is x + y = 5.
True.
Constants versus variables
A constant is a symbol whose value does not change. The symbols 2, 5, , are constants.
The beginning letters of the alphabet a, b, c, are typically used as arbitrary constants, to
which numerical values may be assigned; while the letters x, y, z, are typically used to denote
variables. For example, if we write
y = ax² + bx + c,
we mean that a, b, c are numbers, and that x and y are variables.
Problem 9. The arbitrary constants a and b. Each of the following has
the form y = ax + b. What number is a and what number is b?
a) y = 2x + 3. a = 2, b = 3. b) y = x − 4. a = 1, b = −4.

c) y = −x + 1. a = −1, b = 1. d) y = 5x. a = 5, b = 0.

e) y = −2. a = 0, b = −2. f) y = −4x − 5. a = −4, b = −5.


The equation and graph of a straight line: Section 2
Drawing the graph
Back to Section 1
The x- and y-intercepts of a graph
Drawing the graph
The form y = ax.
Vertical and horizontal lines

The x- and y-intercepts of a graph

The x-intercept of a graph is that value of x where the graph crosses the x-axis. The y-
intercept is the value of y where it crosses the y-axis.
To find the x-intercept, we have to find the value of x where y = 0 -- because
at every point on the x-axis, y = 0.
Example 1. Calculate the value of x when y = 0, that is, find the x-intercept of y = 2x +
10.
Solution. On putting y = 0, we have to solve the equation,
2x + 10 = 0.
We have:
2x = −10
x = −5.
The x-intercept is −5.
Problem 10. Calculate the value of x when y = 0.
a) y = 2x + 4 b) y = 3x − 12

2x + 4 = 0 3x − 12 = 0
2x = −4 3x = 12

x = −2 x=4
c) y = 4x + 1

4x + 1 = 0

4x = −1

x = −¼
To find the y-intercept of a graph, we must find the value of y when x = 0 -- because at
every point on the y-axis, x = 0. But when the equation has the form
y = ax + b,
the y-intercept is simply b. When x = 0,
y = 0 + b = b.
Drawing the graph
Two points determine a straight line. And the two most important points are the x- and y-
intercepts. Therefore whenever we draw a graph, we always mark those points.
Problem 11.
a) When we have the equation of a straight line, how do we find the
a) x−intercept?
Put y =0 and solve for x.
b) How do we find the y−intercept?
Put x =0 and solve for y.
Problem 12. Mark the x- and y-intercepts, and draw the graph of
y = 2x + 6.
The x-intercept −3 is the solution to 2x + 6 = 0. The y-intercept is the
constant term, 6. The graph is the straight line that passes through those two
intercepts.
Example 2. Mark the x- and y-intercepts, and draw the graph of
5x − 2y = 10.
Solution. Although this does not have the form y = ax + b, the strategy is the same. Find
the intercepts by putting x -- then y -- equal to 0.
xy
y-intercept 0 −5
20 x-intercept

When we put x = 0, we have


−2y = 10.
This implies
y = −5.
When we put y = 0, we have
5x = 10.
This implies
x = 2.
Here is the graph:

Problem 13.
a) In an equation y = ax + b , where on the graph do we find b ?
As the y-intercept.
b) Where do we find the solution to ax + b = 0 ?
As the x-intercept.
Problem 14. Mark the x− and y−intercepts, and draw the graph.
a) y = 2x − 6 b) y = −3x + 3

c) y = 4x + 2 d) x − y = 3

See Example 2.

e) x − 2y + 2 = 0 f) 2x − 3y − 6 = 0

g) y = −x + 1 h) y = 6x − 3
Problem 15. The form y = ax.
a) When an equation has the form y = ax (for example, y = 2x), what
a) number is b ? 0
b) Therefore, what number is the y-intercept? 0
c) Through which point does the line pass?
The origin. In other words the x- and y- intercepts coincide.
d) How do we find another point on the line?
Choose any value for x. The equation will then determine the value of y.
(In the next Lesson, we will see that a is the slope of the line.
Example 3. draw the graph of y = 2x.
Solution. Since b = 0, the graph passes through the origin. To find another point, let x =
1, for example. Then y = 2· 1 = 2. The point (1, 2) is on the graph:

2
Example 4. draw the graph of y = 3 x.

Solution. Again, since b = 0, the graph passes through the origin. To find another point,
choose an integer value for x in such a way that y also will be an integer. Choose x equal to
the denominator, 3. The point (3, 2) then is on the graph.

Problem 16. draw each graph.


a) y = 4x b) y = −x

3 2
c) y = 5x d) y = −5x

Example 6. Vertical and horizontal lines.


An equation of the form
x = A number
is the equation of a vertical line. For example,
x = 3.

x = 3 -- the x-coördinate is 3 -- is true of every point on that line.


An equation of the form
y = A number
is the equation of a horizontal line. For example,
y = −4.
The y-coördinate is −4 at every point on that line.
Problem 17. draw each graph.
a) x=5 b) x = −2

c) y=3 d) y = −1

Problem 18.
a) What is the equation of the x-axis? y = 0
b) What is the equation of the y-axis? x = 0

THE SLOPE OF A
STRAIGHT LINE
Definition of the slope
"Up" or "down"?
Horizontal and vertical lines
The slope-intercept form
The general form
Parallel and perpendicular lines
2nd level:
The point-slope formula
The two-point-formula

IN THE PREVIOUS LESSON, we discussed the equation of a straight line. Sketching


the graph of the equation of a line should be a basic skill.

Consider this straight line. The (x, y) coördinates at B have changed from the
coördinates at A. By the symbol Δx ("delta x") we mean the change in the x-
coördinate. That is,
Δx = x2 − x1.
(As for using the subscripts 1 and 2, see Lesson 32, the section, The distance bewteen
any two points.)
Similarly, Δy ("delta y") signifies the resulting change in the y-coördinates.
Δy = y2 − y1.
Δx is the horizontal leg of that right triangle; Δy is the vertical leg.
By the slope of a straight line, then, we mean this number:

_Vertical leg_ Δy
Slope = = =
Horizontal leg Δx

For example,
If the value of y changes by 2 units when the value of x changes by 3,
2
then the slope of that line is 3 .
2
What does slope 3 mean? It indicates the rate at which a change
in the value of x produces a change in the value of y. 2 units of y per -- for every -- 3 units
of x.
For every 3 units that line moves to the right, it will move up 2. That will be true
between any two points on that line. Over 6 and up 4, over 15 and up 10. Because a straight
line has one and only one slope. (Theorem 8.1 of Precalculus.)

In each line above, the x-coördinate has increased by 1 unit. In the line on the left,
however, the value of y has increased much more than in the line on the right. The line on
the left has a greater slope than the line on the right. The value of y has changed at a much
greater rate.
If the x-axis represents time and the y-axis distance, as is the case in many
applications, then the rate of change of y with respect to x -- of distance
with respect to time -- is called speed or velocity. So many miles per hour,
or meters per second.

Up or down?
Which line do we say is sloping "up"? And which is sloping "down"?
Since we imagine moving along the x-axis from left to right, we say that the line on the
left is sloping up, and the line on the right, down.
What is more, a line that slopes up has a positive slope. While a line that slopes down
has a negative slope.
For, both the x- and y-coördinates of B are greater than the coördinates of
A, so that both Δx and Δy are positive. Therefore their quotient, which is
the slope, is positive.
But while the x-coördinate of D is greater than the x-coördinate of C, so
that Δx is positive, the y-coördinate of D is less than the y-coördinate of C,
so that Δy, 5 − 8, is negative. Therefore that quotient is negative.

Problem 1. What number is the slope of each line?


To see the answer, pass your mouse over the colored area.
To cover the answer again, click "Refresh" ("Reload").
Do the problem yourself first!

a) b)

3 4
4 3

c) d)
4 3
−3 −4

Horizontal and vertical lines


What number is the slope of a horizontal line -- that is, a line parallel to the x-axis? And
what is the slope of a vertical line?

A horizontal line has slope 0, because even though the value of xchanges, the value
of y does not. Δy = 0.
Δy 0
= = 0. (Lesson 5)
Δx Δx
For a vertical line, however, the slope is not defined. The slope tells how the y-
coördinate changes when the x-coördinate changes. But the x-coördinate does not change --
Δx = 0. A vertical line does not have a slope.
Δy Δy
= = No value. (Lesson 5)
Δx 0
Problem 2.

a) Which numbered lines have a positive slope? 2 and 4.


b) Which numbered lines have a negative slope? 1 and 3.
c) What slope has the horizontal line 5? 0.
c) What slope has the vertical line 6? It does not have a slope.
Example 1. Calculate the slope of the line that passes through the points (3, 6) and (1, 2)
Solution. To do this problem, here again is the definition of the slope. It is the number
Δy Difference of y-coördinates
= =
Δx Difference of x-coördinates
Therefore, the slope of the line passing through (3, 6) and (1, 2) is:

Δy 6−2 4 2
= = = = 2.
Δx 3−1 2 1
Note: It does not matter which point we call the first and which the second. But if we
calculate Δy starting with (3, 6), then we must calculate Δx also starting with (3, 6).
As for the meaning of slope 2: On the straight line that joins those two points, for
every 1 unit the value of x changes, the value of y will change by 2 units. That is the rate of
change of y with respect to x. 2 for every 1.
Problem 3. Calculate the slope of the line that joins these points.
a) (1, 5) and (4, 17) b) (−3, 10) and (−2, 7)
17 − 5 12 _ 10 − 7 _ 3
4−1 3
= =4 = = −3
−3 − (−2) −1

c) (1, −1) and (−7, −5) d) (2, −9) and (−2, −5)
−5 − (−1) −4 −9 − (−5) −4
−7 − 1
= −8 = ½ 2 − (−2)
= 4 = −1

The slope-intercept form


This linear form
y = ax + b
is called the slope-intercept form of the equation of a straight line. Because, as we
can prove (Topic 9 of Precalculus): a is the slope of the line, and b is the y-intercept.
Problem 4. What number is the slope of each line, and what is the meaning of each slope?
a) y = 5x − 2
The slope is 5. This means that y increases 5 units for every 1 unit x increases.
That is the rate of change of y with respect to x.

2
b) y = − 3 x + 4
2
The slope is − 3 . This means that y decreses 2 units for every
3 units of x..
Problem 5.
a) Write the equation of the straight line whose slope is 3 and whose
a) y-intercept is 1.
y = 3x + 1
b) Write the equation of the straight line whose slope is −1 and whose
a) y-intercept is −2.
y = −x − 2
2
c) Write the equation of the straight line whose slope is 3 and which

passes through the origin.

y = 23 x. The y-intercept b is 0.

Problem 6. Sketch the graph of y = −2x.

This is a straight line of slope −2 -- over 1 and down 2 -- that passes through the
origin: b = 0.
(Compare Lesson 33, Problem 15.)
The general form
This linear form
Ax + By + C = 0
where A, B, C are integers (Lesson 2), is called the general form of the equation of a straight
line.
Problem 7. What number is the slope of each line, and what is the meaning of each slope?
a) x + y − 5 = 0
This line is in the general form. It is only when the line is in the slope-
intercept form, y = ax + b, that the slope is a. Therefore, on solving for y: y =
−x + 5. The slope therefore is −1. This means that the value of y decreases 1
unit for every unit that the value of x increases.
5, incidentally, is b, the y-intercept.
b) 2x − 3y + 6 = 0
−3y = −2x − 6

y = 23 x + 2, on dividing every term by −3.

2
The slope therefore is 3 . This means that for every 3 units
the line goes over, it goes up 2.
c) Ax + By + C = 0

By = −Ax − C

y = −A
B
x −
C
B
, on dividing every term by B.

A
The slope is − B .
We can view that as a formula for the slope when the equation is in the general
form. For example, if the equation is
4x − 5y + 2 = 0,
then the slope is
4 4
− = .
−5 5
Parallel and perpendicular lines
Straight lines will be parallel if they have the same slope. The following are equations
of parallel lines:
y = 3x + 1 and y = 3x − 8.
They have the same slope 3.
Straight lines will be perpendicular if
their slopes have opposite signs -- one positive and one negative,
1)
and

2) they are reciprocals of one another.


That is:
1
If m is the slope of one line, then a perpendicular line has slope − .
m
To be specific, if a line has slope 4, then every line that is perpendicular to it has slope
−¼.
(We will prove that below.)
Problem 8. Which of these lines are parallel and which are perpendicular?
a) y = 2x + 3 b) y = −2x + 3 c) y = ½x + 3 d) y = 2x − 3
a) and d) are parallel. b) and c) are perpendicular.
Problem 9. If a line has slope 5, then what is the slope of a line that is
1
perpendicular to it? −5

2
Problem 10. If a line has slope − 3 , then what is the slope of a

3
perpendicular line? 2

Problem 11. If a line has equation y = 6x − 5, then what is the slope


1
of a perpendicular line? − 6

Theorem: The slopes of perpendicular lines


If two straight lines are perpendicular to one another, then the product of their slopes is −1.
That is: If the slope of one line is m, then the slope of the perpendicular line
1
is − .
m
Let L1 be a straight line, and let the perpendicular straight line L2cross L1 at the point A.
Let L1 have slope m1, and let L2 have slope m2. Assume that m1 is positive. Then m2, as we will
see, must be negative.
Draw a straight line AB of length 1 parallel to the x-axis, and draw BC at right angles
to AB equal in length to m1.
Extend CB in a straight line to join L2 at D.
Now, since the straight line L2 has one slope m2 (Theorem 8.1 of Precalculus), the
length of BD will be |m2|. For in going from A to Don L2, we go over 1 and down |m2|.
That is, m2 is a negative number.

(Same figure.)
Angle CAD is a right angle. Therefore angle  is the complement of angle ß.
But triangle ABD is right-angled, and therefore the angle at D is also the complement of
angle ß;
therefore the angle at D is equal to angle .
The right triangles ABC, ABD therefore are similar (Topic 5 of Trigonometry),
and the sides opposite the equal angles are proportional:

This implies
m1m2| = 1.
But m2 is negative. Therefore,
m1m2 = −1.
Which is what we wanted to prove.

How to find the Domain and Range of a relation given by


its graphs?

Example 1
a) Find the domain and b) the range of the relation given by its graph shown below and c)
state whether the relation is a function or not.

Solution:
a) Domain: We first find the 2 points on the graph of the given relation with the smallest and
the largest x-coordinate. In this example the 2 points are A(-2,-4) and B(4,-6) (see graph
above). The domain is the set of all x values from the smallest x-coordinate (that of A) to the
largest x-coordinate (that of B) and is written as:
-2 ≤ x ≤ 4
The double inequality above has the inequality symbol ≤ at both sides because the closed
circles at points A and B indicate that the relation is defined at these values of x.
b) Range: We need to find the coordinates of the 2 points on the graph with the lowest and
the largest values of the y coordinate. In this example, these points are B(4,-6) and C(2,2).
The range is the set of all y values between the smallest and the largest y coordinates and
given by the double inequality:
-6 ≤ y ≤ 2
The inequality symbol ≤ is used at both sides because the closed circles at points B and C
indicates the relation is defined at these values.
c) The relation graphed above is a function because no vertical line can intersect the given
graph at more than one point.

Example 2
Find the a) domain and a) range of the relation given by its graph shown below and c) state
whether the relation is a function or not.

Solution:
a) Domain: In this example points A(-3,-5) and B(8,4) have the smallest and the largest x-
coordinates respectively, hence the domain is given by:
-3 ≤ x ≤ 8
The use of the symbol ≤ at both sides is due to the fact that the relation is defined at points A
and B (closed circles at both points).

Sponsored Content

The air conditioning companies are furious with this new Device!Cool Air Technology


Say Goodbye to Botox. Grandma's ₱1,755 Trick Erases WrinklesHealth News Online

Recommended by
b) Range: Points A and B have the smallest and the largest values of the y-coordinate
respectively. The range is given by the inequality:
- 5≤ y ≤ 4
The use of the symbol ≤ at both sides is due to the fact that the relation is defined at points A
and B.
c) No vertical line can cut the given graph at more than one point and therefore the relation
graphed above is a function.

Example 3
Find the domain and range of the relation given by its graph shown below and state whether
the relation is a function or not.

Solution:
a) Domain: Points A(-3,-2) and B(1,-2) have the smallest and the largest x-coordinates
respectively, hence the domain:
-3 ≤ x ≤ 1
The use of the symbol ≤ at both sides is due to the fact that the relation is defined at points A
and B (closed circles at both points).
b) Range: Points C(-1,-5) and D(-1,1) have the smallest and the largest y-coordinate
respectively. The range is given by the double inequality:
- 5≤ y ≤ 1
The relation is defined at points C and D (closed circles), hence the use of the inequality
symbol ≤.
c) There is at least one vertical line that cuts the given graph at two points (see graph below)
and therefore the relation graphed above is NOT a function.
Example 4
Find the domain and range of the relation given by its graph shown below and state whether
the relation is a function or not.

Solution:
a) Domain: Points A(-3,0) has the smallest x-coordinate. The arrow at the top right of the
graph indicates that the graph continues to the left as x increases. Hence there is no limit to
the largest x-coordinate of points on the graph. The domain is given by all values greater
than or equal to the smallest values x = -3 and is written as:
x ≥ -3
The use of the symbol ≥ at because the relation is defined at points A (closed circle at point
A).
b) Range: Points B and C have equal and smallest y-coordinates equal to -2. The arrow at
the top right of the graph indicates that the y coordinate increases as x increases. Hence
there is no limit to the y-coordinate and therefore the range is given by all values greater
than or equal to the smallest value y = -2 and is written as:
y ≥ -2
The use of the inequality symbol ≥ is due to the fact that the relation is defined at y = -2
(closed circle at B and C).
c) There is no vertical line that cuts the given graph at more than one point (see graph
below) and therefore the relation graphed above is a function.

Example 5
Find the domain and range of the relation given by its graph shown below and state whether
the relation is a function or not.

Solution:
a) Domain: Points A(-2,-3) has the smallest x-coordinate. The arrow at the top right of the
graph indicates that the graph continues to the left as x increases. Hence there is no limit to
the largest x-coordinate of points on the graph. The domain is given by all values greater
than the smallest values x = - 2 and is written as:
x > -2
We use of the inequality symbol > (with no equal) because the relation is not defined at
points A (open circle at point A).
b) Range: Points A(-2,-3) has the smallest y-coordinate equal to - 3. The arrow at the top
right of the graph indicates that the y coordinate increases as x increases. Therefore there is
no limit to the y-coordinate. Hence the range is given by all values greater than the smallest
value y = - 3 and is written as:
y>-3
The inequality symbol > is used because the relation is not defined at y = - 3 (open circle at
point A).
c) The graph represents a function because there is no vertical line that cuts the given graph
at more than one point.

Arithmetic sequences and series


An arithmetic sequence is a sequence of numbers such that the difference of any two successive members of the sequence is a constant.
Example

2,4,6,8,10….is an arithmetic sequence with the common difference 2.

If the first term of an arithmetic sequence is a1 and the common difference is d, then the nth term of the sequence is given by:

an=a1+(n−1)dan=a1+(n−1)d
An arithmetic series is the sum of an arithmetic sequence. We find the sum by adding the first, a1 and last term, an, divide by 2 in order to get the mean of the two
values and then multiply by the number of values, n:

Sn=n2(a1+an)Sn=n2(a1+an)

Example

Find the sum of the following arithmetic series 1,2,3…..99,100

We have a total of 100 values, hence n=100. Our first value is 1 and our last is 100. We plug these values into our formula and get:

S100=1002(1+100)=5050

Arithmetic Sequences and Series


Home > Lessons > Arithmetic Sequences and Series Updated March 22nd, 2018
Introduction

This page will teach you about arithmetic sequences and series. Here are the sections
within this page:

 Identifying Arithmetic Sequences


 Calculating the nth Term in Arithmetic Sequences
 Finding the Number of Terms in an Arithmetic Sequence
 Finding the Sum of Arithmetic Series
 Instructional Videos
 Interactive Quizmasters
 Related Lessons and Quizmasters

Identifying an Arithmetic Sequence

Sequences of numbers that follow a pattern of adding a fixed number from one term to
the next are called arithmetic sequences. The following sequences are arithmetic
sequences:
Sequence A: 5 , 8 , 11 , 14 , 17 , ...
Sequence B: 26 , 31 , 36 , 41 , 46 , ...
Sequence C: 20 , 18 , 16 , 14 , 12 , ...

For sequence A, if we add 3 to the first number we will get the second number. This
works for any pair of consecutive numbers. The second number plus 3 is the third number:
8 + 3 = 11, and so on.
For sequence B, if we add 5 to the first number we will get the second number. This also
works for any pair of consecutive numbers. The third number plus 5 is the fourth number:
36 + 5 = 41, which will work throughout the entire sequence.

Sequence C is a little different because we need to add -2 to the first number to get the
second number. This too works for any pair of consecutive numbers. The fourth number
plus -2 is the fifth number: 14 + (-2) = 12.

Because these sequences behave according to this simple rule of addiing a constant
number to one term to get to another, they are called arithmetic sequences. So that we can
examine these sequences to greater depth, we must know that the fixed numbers that bind
each sequence together are called the common differences. Sometimes mathematicians
use the letter d when referring to these types of sequences.

Mathematicians also refer to generic sequences using the letter a along with subscripts
that correspond to the term numbers as follows:

Generic Sequence: a1, a2, a3, a4, ...

This means that if we refer to the fifth term of a certain sequence, we will label it a5. a17is
the 17th term. This notation is necessary for calculating nth terms, or an, of sequences.

The d-value can be calculated by subtracting any two consecutive terms in an arithmetic
sequence.

d = an - an - 1

...where n is any positive integer greater than 1.

Calculating the nth Term

In order for us to know how to obtain terms that are far down these lists of numbers, we
need to develop a formula that can be used to calculate these terms. If we were to try and
find the 20th term, or worse to 2000th term, it would take a long time if we were to simply
add a number -- one at a time -- to find our terms.

If a 5-year-old was asked what the 301st number is in the set of counting numbers, we
would have to wait for the answer while the 5-year-old counted it out using unnecessary
detail. We already know the number is 301 because the set is extremely simple; so,
predicting terms is easy. Upon examining arithmetic sequences in greater detail, we will find
a formula for each sequence to find terms.

1. Let's examine sequence A so that we can find a formula to express its nth term.

If we match each term with it's corresponding term number, we get:


n 1 2 3 4 5 ...

Term 5 8 11 14 17 . . .

The fixed number, called the common difference (d), is 3; so, the formula will be an =
dn + c or an = 3n + c, where c is some number that must be found.

For sequence A above, the rule an = 3n + c would give the values...


3×1 + c = 3 + c
3×2 + c = 6 + c
3×3 + c = 9 + c
3×4 + c = 12 + c
3×5 + c = 15 + c

If we compare these values with the ones in the actual sequence, it should be clear
that the value of c is 2. Therefore the formula for the nth term is...

an = 3n + 2.

Now if we were asked to find the 37th term in this sequence, we would calculate for
a37 or 3(37) + 2 which is equal to 111 + 2 = 113. So, a37 = 113, or the 37th term is
113. Likewise, the 435th term would be a435 = 3(435) + 2 = 1307.

2. Let's take a look at sequence B.

n 1 2 3 4 5 ...

Term 26 31 36 41 46 . . .

3. The fixed number, d, is 5. So the formula will be an = dn + c or an = 5n + c .


4. For the sequence above, the rule an = 5n + c would give the values...
5. 5×1 + c = 5 + c
5×2 + c = 10 + c
5×3 + c = 15 + c
5×4 + c = 20 + c
5×5 + c = 25 + c
6. If we compare these values with the numbers in the actual sequence, it should be
clear that the value of c is 21. Therefore, the formula for the nth term is...
7. an = 5n + 21.
8. If we wanted to calculate the 14th term, we would calculate for
9. a14 = 5(14) + 21 = 70 + 21 = 91. If we needed the 40th term, we would calculate a40=
5(40) + 21 = 200 + 21 = 221. The general formula is very handy.
10. Now let's do the third and final example....

n 1 2 3 4 5 ...

Term 20 18 16 14 12 . . .

11. The common difference is -2. So the formula will be -2n + c, where c is a number
that must be found.
12. For sequence C, the rule -2n + c would give the values...
13. -2×1 + c = -2 + c
-2×2 + c = -4 + c
-2×3 + c = -6 + c
-2×4 + c = -8 + c
-2×5 + c = -10 + c
14. If we compare these values with the numbers in the actual sequence, it should be
clear that the value of c is 22. Therefore, the formula for the nth term is...
15. an = -2n + 22.
16. If for some reason we needed the 42nd term, we would calculate for a42 = -2(42) +
22 = -84 + 22 = -62. Similarly, a90 = -2(90) + 22 = -180 + 22 = -158.

ideo: Finding the nth Term of an Arithmetic Sequence


uizmaster: Finding Formula for General Term
uizmaster: Finding the nth Term

Finding the Number of Terms

It may be necessary to calculate the number of terms in a certain arithmetic sequence.


To do so, we would need to know two things.

We would need to know a few terms so that we could calculate the common difference
and ultimately the formula for the general term. We would also need to know the last
number in the sequence.

Once we know the formula for the general term in a sequence and the last term, the
procedure is relatively uncomplicated. Set them equal to each other. Since the formula uses
the variable n to calculate terms, we can also use it to determine the term number for any
given term.

i. If we again look at sequence A above, let's use the formula that was found to
calculate term values, an = 3n + 2. If we knew that 47 was a number in the sequence
-- 5, 8, 11, 14, 17, ..., 47 -- we would set the number 47 equal to the formula an = 3n
+ 2, we would get 47 = 3n + 2. Solving this equation yields n = 15. This means that
there are 15 terms in the sequence and that the 15th term, a15, is equal to 47.
ii. Let's look at a portion of sequence C. If the sequence went from 20 to -26, we would
have: 20, 18, 16, 14, 12, ...,-26. We would use the formula for the general term, an =
-2n + 22, and set it equal to the last term, -26. We would get -26 = -2n + 22 and
algebra would allow us to arrive at n = 24. This means that there are 24 terms in the
sequence and that a24 = -26.

ideo: Finding the Number of Terms in a Finite Arithmetic Sequence


uizmaster: Finding Formula for General Term
uizmaster: Finding the nth Term

Finding the Sum of a Series

Given our generic arithmetic sequence...


a1, a2, a3, a4, ...

...we can add the terms, called a series, as follows.

a1 + a2 + a3 + a4 + ... + an.
Given the formula for the general term an = dn + c, there exists a formula that can add
such a finite list of these numbers. It requires three pieces of information. The formula is...

Sn = ½n(a1 + an)

...where Sn is the sum of the first n numbers, a1 is the first number in the sequence and anis
the nth number in the sequence.

If you would like to see a derivation of this arithmetic series sum formula, watch this
video.

ideo: Arithmetic Series: Deriving the Sum Formula

Usually problems present themselves in either of two ways. Either the first number and
the last number of the sequence are known or the first number in the sequence and the
number of terms are known.

i. Starting with the easier of the two cases, let's take a portion of sequence A. If we
were dealing with 5, 8, 11, 14, 17, ... , 128, then we would know that a1 = 5 and an =
128. If we knew the number of terms in this sequence, we would be able to use the
formula. Finding n becomes our next task. Since we know the formula for the
general term, an = 3n + 2, we can use it to find the number of terms in this
sequence. We set the last term equal to the formula and solve for n. We get 128 =
3n + 2, which means that n = 42 and a42 = 128. Now we can plug the information
into the sum formula and get S42 = ½(42)(5 + 128) = (21)(133) = 2793, which must
be the sum of the first 42 terms in the sequence.
ii. For the more difficult situation, let's take a finite portion of sequence B. If we had 26,
31, 36, 41, 46, ... and knew that there were 50 terms in the sequence, then we have
a1 = 26 and n = 50. We would have to develop a formula for the nth term so we
could calculate a50, the last term in the sequence. Since we already calculated the
formula above, we can use it to calculate a50. It is an = 5n + 21 is the formula so
a50 = 5(50) + 21 = 250 + 21 = 271. Now we can plug the numbers into the formula
and gain a solution. S50 = ½(50)(26 + 271) = 25(297) = 7425. This means that the
sum of the first 50 terms is 7425.

Algebra Questions with Answers and Solutions


for Grade 8
Grade 8 algebra questions with solutions are presented. Questions on solving equations,
simplifying expressions including expressions with fractions are included.

NOTE: In what follows, mixed numbers are written in the form a b/c. For example 2 1/3
means the mixed number 2 + 1/3.

1. Simplify the following algebraic expressions.


A) -2x + 5 + 10x - 9
B) 3(x + 7) + 2(-x + 4) + 5x
2. Simplify the expressions.
A) (2x - 6) / 2
B) (-x - 2) / (x + 2)
C) (5x - 5)/10
3. Solve for x the following equations.
A) -x = 6
B) 2x - 8 = -x + 4
C) 2x + 1/2 = 2/3
D) x/3 + 2 = 5
E) -5/x = 2
4. Evaluate for the given values of

x and y.
A) x2 - y2 , for x = 4 and y = 5
B) |4x - 2y| , for x = -2 and y = 3
C) 3x3 - 4y4 , for x = -1 and y = -2

5. Solve the following inequalities.


A) x + 6 < 0
B) x + 1 > 5
C) 2(x - 2) < 12
6. What is the reciprocal of each of the following numbers?
A) -1
B) 0
C) 3/4
D) 2 5/7
E) 0.02
7. Evaluate the following expressions involving mixed numbers.
A) 3 3/4 + 6 1/7
B) (1 3/5) × (3 1/3) - 2 1/2
C) (5 2/3) ÷ (4 1/5)
D) (3 4/7 - 1 1/2) ÷ (2 3/8 + 2 1/4)
8. Evaluate the folwoing exponential expressions.
A) -4
2

B) (-2)3
C) (-2)4
D) 10000
E) 5661

9. Convert to fractions and write in simplest form.


A) 0.02
B) 12%
C) 0.5%
D) 1.12
10. Convert to decimals.
A) 1/5
B) 120%
C) 0.2%
D) 4 8/5
11. Convert to percent.
A) 3/10
B) 1.4
C) 123.45
D) 2 4/5
12. Which of these numbers is divisible by 3?
A) 156312
B) 176314
13. Which of these numbers is divisible by 4?
A) 3432
B) 1257
14. Which of these numbers is divisible by 6?
A) 1233
B) 3432
15. Which of these numbers is divisible by 9?
A) 2538
B) 1451
16. Evaluate 8x + 7 given that x - 3 = 10.

Sponsored Content

The air conditioning companies are furious with this new Device!Cool Air Technology


Better Than Botox? Mother 57, Looks 35 Without SurgeryHealth News Online

Recommended by

Solutions and Answers to the Above Questions

1.
A) -2x + 5 + 10x - 9 : given
= (10x - 2x) + (5 - 9) : put like terms together
= 8x - 4 : group

B) 3(x + 7) + 2(-x + 4) + 5x : given


= 3x + 21 - 2x + 8 + 5x : expand
= (3x - 2x + 5x) + (21 + 8) : put like terms together
= 6x + 29 : group
2.
A) (2x - 6) / 2 : given
= 2(x - 3) / 2 : factor 2 in numerator
= x - 3 : divide numerator and denominator by 2 to simplify

B) (-x - 2) / (x + 2) : given
= -1(x + 2) / (x + 2) : factor -1 in numerator
= -1 : divide numerator and denominator by x + 2 to simplify

C) (5x - 5)/10 : given


= 5(x - 1) / 10 : factor 5 in numerator
= (x - 1) / 2 : divide numerator and denominator by 5 to simplify
3.
A) -x = 6 : given
x = -6 : multiply both sides of the equation by -1

B) 2x - 8 = -x + 4 : given
2x - 8 + 8 = -x + 4 + 8 : add +8 to both sides of the equation
2x = -x + 12 : group like terms
2x + x = -x + 12 + x : add +x to both sides
3x = 12 : group like terms
x = 4 : mutliply both sides by 1/3

C) 2x + 1/2 = 2/3 : given


2x + 1/2 - 1/2 = 2/3 - 1/2 : subtract 1/2 from both sides
2x = 1/6 : group like terms
x = 1/12 : multiply both sides by 1/2

D) x/3 + 2 = 5 : given
x/3 + 2 - 2 = 5 - 2 : subtract 2 from both sides
x/3 = 3 : group like terms
x = 9 : multiply both sides by 1/2

E) -5/x = 2 : given
-5 = 2x : multiply both sides by x and simplify
-5/2 = x : : multiply both sides by 1/2
4.
A) x2 - y2 , x = 4 , y = 5 : given
42 - 52 : substitute x and y by the given values
=16 - 25 = -9

B) |4x - 2y| , x = -2 , y = 3 : given


|4(-2) - 2(3)| : substitute x and y by the given values
= |-14| = 14 : evaluate

C) 3x3 - 4y4 , x = -1 , y = -2 : given


3(-1)3 - 4(-2)4 : substitute x and y by the given values
= -3 - 64 = -67 : evaluate
5.
A) x + 6 < 0 : given
x + 6 - 6 < -6 : subtract 6 from both sides
x < -6 : group like terms

B) x + 1 > 5 : given
x + 1 - 1 > 5 - 1 : subtract 1 from both sides
x > 4 : group like terms

C) 2(x - 2) < 12 : given


x - 2 < 6 : mutliply both sides by 1/2
x - 2 + 2 < 6 + 2 : add 2 to both sides
x < 8 : group like terms
6.
A) (-1) a = 1 : definition: a is the reciprocal of -1
a = 1/-1 = -1 : solve for a; -1 is the reciprocal of -1

B) (0) b = 1 : definition: b is the reciprocal of 0


b = undefined : no value of b satisfies the above equation

C) (3/4) c = 1 : definition: c is the reciprocal of 3/4


c = 4/3 : solve for c; c = 4/3 is the reciprocal of 3/4

D) (2 5/7) d = 1 : definition: d is the reciprocal of 2 5/7.


(19/7) d = 1 : convert the mixed number 2 5/7 into a fraction.
d = 7/19 : : solve for d; d = 7/19 is the reciprocal of 2(5/7)

E) 0.02 d = 1 : definition: d is the reciprocal of 0.02.


d = 1/0.02 : solve for d; d = 50 is the reciprocal of 0.02
7.
A) 3 3/4 + 6 1/7 : given
= (3 + 6) + (3/4 + 1/7) : put the whole parts together and the fractional parts together.
= 9 + (21/28 + 4/28) : add.
= 9 25/28

B) (1 3/5) × (3 1/3) - 2 1/2 : given


= (8/5) × (10/3) - 2 1/2 : convert mixed numbers to mupliply into fractions.
= 80/15 - 2 1/2 = 5 1/3 - 2 1/2 = 4 4/3 - 2 1/2: mupliply and write as mixed number if
possible
= (4 - 2) + (4/3 - 1/2) : subtract
= 2 5/6
C) (5 2/3) ÷ (4 1/5) : given
= (17/3) ÷ (21/5) : convert mixed numbers to fractions.
= 85 / 63 : divide fractions
= 1 22/63 : write as mixed number

D) (3 4/7 - 1 1/2) ÷ (2 3/8 + 2 1/4) : given


= [(3 - 1) + (4/7 - 1/2)] ÷ [(2 + 2) + (3/8 + 1/4)]: evaluate numerator and denominator as
fractions.
= (2 1/14) ÷ (4 5/8)
= (29/14) ÷ (37/8)
= 116/259
8.
A) - 42 = - (4 × 4) = -16 : expand and calculate

B) (-2)3 = (-2)×(-2)×(-2) = -8 : expand and calculate

C) 10000 = 1 : definition: any nonzero number to the power zero gives 1

D) 5661 = 566
9.
A) 0.02 = 1/50
B) 12% = 3/25
C) 0.5% = 1/200
D) 1.12 = 28/25
10.
A) 1/5 = 0.2
B) 120% = 1.2
C) 0.2% = 0.002
D) 4 8/5 = 5.6
11.
A) 3/10 = 30%
B) 1.4 = 140%
C) 123.45 = 12345%
D) 2 4/5 = 280%
12.
A) 156312 , is divisible by 3
B) 176314 , is not divisible by 3
13.
A) 3432 , is divisible by 4
B) 1257 , is not divisible by 4
14.
A) 1233 , is not divisible by 6
B) 3432 , is divisible by 6
15.
A) 2538 , is divisible by 9
B) 1451 , is not divisible by 9
16.
Evaluate 8x + 7 given that x - 3 = 10.
x - 3 = 10: given equation
x = 10 + 3 = 13: solve the given equation.
8(13) + 7 = 111 substitute x by 3 in the given expression and evaluate.

10 tricks for doing fast math


Here are 10 fast math strategies students (and adults!) can use to do math in their heads.
Once these strategies are mastered, students should be able to accurately and
confidently solve math problems that they once feared solving.

1. Adding large numbers


Adding large numbers just in your head can be difficult. This method shows how to simplify
this process by making all the numbers a multiple of 10. Here is an example:

644 + 238

While these numbers are hard to contend with, rounding them up will make them more
manageable. So, 644 becomes 650 and 238 becomes 240.

Now, add 650 and 240 together. The total is 890. To find the answer to the original equation,
it must be determined how much we added to the numbers to round them up.

650 – 644 = 6 and 240 – 238 = 2

Now, add 6 and 2 together for a total of 8

To find the answer to the original equation, 8 must be subtracted from the 890.

890 – 8 = 882

So the answer to 644 +238 is 882.

2. Subtracting from 1,000


Here’s a basic rule to subtract a large number from 1,000: Subtract every number except the
last from 9 and subtract the final number from 10

For example:

1,000 – 556

Step 1: Subtract 5 from 9 = 4

Step 2: Subtract 5 from 9 = 4

Step 3: Subtract 6 from 10 = 4

The answer is 444.

3. Multiplying 5 times any number


When multiplying the number 5 by an even number, there is a quick way to find the answer.

For example, 5 x 4 =

 Step 1: Take the number being multiplied by 5 and cut it in half, this makes the number 4
become the number 2.
 Step 2: Add a zero to the number to find the answer. In this case, the answer is 20.
5 x 4 = 20
When multiplying an odd number times 5, the formula is a bit different.

For instance, consider 5 x 3.

 Step 1: Subtract one from the number being multiplied by 5, in this instance the number 3
becomes the number 2.
 Step 2: Now halve the number 2, which makes it the number 1. Make 5 the last digit. The
number produced is 15, which is the answer.
5 x 3 = 15

4. Division tricks
Here’s a quick way to know when a number can be evenly divided by these certain numbers:

 10 if the number ends in 0


 9 when the digits are added together and the total is evenly divisible by 9
 8 if the last three digits are evenly divisible by 8 or are 000
 6 if it is an even number and when the digits are added together the answer is evenly
divisible by 3
 5 if it ends in a 0 or 5
 4 if it ends in 00 or a two digit number that is evenly divisible by 4
 3 when the digits are added together and the result is evenly divisible by the number 3
 2 if it ends in 0, 2, 4, 6, or 8
5. Multiplying by 9
This is an easy method that is helpful for multiplying any number by 9. Here is how it works:

Let’s use the example of 9 x 3.

Step 1: Subtract 1 from the number that is being multiplied by 9.


3–1=2

The number 2 is the first number in the answer to the equation.

Step 2: Subtract that number from the number 9.


9–2=7

The number 7 is the second number in the answer to the equation.

So, 9 x 3 = 27

6. 10 and 11 times tricks


The trick to multiplying any number by 10 is to add a zero to the end of the number. For
example, 62 x 10 = 620.

There is also an easy trick for multiplying any two-digit number by 11. Here it is:

11 x 25
Take the original two-digit number and put a space between the digits. In this example, that
number is 25.

2_5

Now add those two numbers together and put the result in the center:

2_(2 + 5)_5

2_7_5

The answer to 11 x 25 is 275.

If the numbers in the center add up to a number with two digits, insert the second number
and add 1 to the first one. Here is an example for the equation 11 x 88

8_(8 +8)_8

(8 + 1)_6_8

9_6_8

There is the answer to 11 x 88: 968

7. Percentage
Finding a percentage of a number can be somewhat tricky, but thinking about it in the right
terms makes it much easier to understand. For instance, to find out what 5% of 235 is, follow
this method:

 Step 1: Move the decimal point over by one place, 235 becomes 23.5.
 Step 2: Divide 23.5 by the number 2, the answer is 11.75. That is also the answer to the
original equation.
8. Quickly square a two-digit number that ends in 5
Let’s use the number 35 as an example.

 Step 1: Multiply the first digit by itself plus 1.


 Step 2: Put a 25 at the end.
35 squared = [3 x (3 + 1)] & 25

[3 x (3 + 1)] = 12

12 & 25 = 1225

35 squared = 1225

9. Tough multiplication
When multiplying large numbers, if one of the numbers is even, divide the first number in
half, and then double the second number. This method will solve the problem quickly. For
instance, consider

20 x 120

Step 1: Divide the 20 by 2, which equals 10. Double 120, which equals 240.

Then multiply your two answers together.

10 x 240 = 2400

The answer to 20 x 120 is 2,400.

10. Multiplying numbers that end in zero


Multiplying numbers that end in zero is actually quite simple. It involves multiplying the other
numbers together and then adding the zeros at the end. For instance, consider:

200 x 400

Step 1: Multiply the 2 times the 4

2x4=8

Step 2: Put all four of the zeros after the 8

80,000

200 x 400= 80,000

Practicing these fast math tricks can help both students and teachers improve their math
skills and become secure in their knowledge of mathematics—and unafraid to work with
numbers in the future.

How to Solve Math Problems Faster: 15 Techniques to Show


Students
April 6, 2017 Marcus Guido Teaching Strategies

“Test time. No calculators.”


You’ll intimidate many students by saying this, but teaching techniques to solve math problems with ease and
speed can make it less daunting.
This can also make math more rewarding. Instead of relying on calculators, students learn strategies that can
improve their concentration and estimation skills while building number sense. And, while there are educators
who oppose math “tricks” for valid reasons, proponents point to benefits such as increased confidence to handle
difficult problems.
Here are 15 techniques to show students, helping them solve math problems faster:

Addition and Subtraction


1. Two-Step Addition

Many students struggle when learning to add integers of three digits or higher together, but changing the process’s
steps can make it easier.
The first step is to add what’s easy. The second step is to add the rest.
Let’s say students must find the sum of 393 and 89. They should quickly see that adding 7 onto 393 will equal 400
— an easier number to work with. To balance the equation, they can then subtract 7 from 89.
Broken down, the process is:

 393 + 89
 (393 + 7) + (89 – 7)
 400 + 82
 482

With this fast technique, big numbers won’t look as scary now.

2. Two-Step Subtraction
There’s a similar method for subtraction.
Remove what’s easy. Then remove what’s left.
Suppose students must find the difference of 567 and 153. Most will feel that 500 is a simpler number than 567.
So, they just have to take away 67 from the minuend — 567 — and the subtrahend — 153 — before solving the
equation.
Here’s the process:

 567 – 153
 (567 – 67) – (153 – 67)
 500 – 86
 414

Instead of two complex numbers, students will only have to tackle one.

By making
math engaging, students that use Prodigy consistently outperform those that don’t on standardized assessments

3. Subtracting from 1,000


You can give students confidence to handle four-digit integers with this fast technique.
To subtract a number from 1,000, subtract that number’s first two digits from 9. Then, subtract the final digit from
10.
Let’s say students must solve 1,000 – 438. Here are the steps:

 9–4=5
 9–3=6
 10 – 8 = 2
 562

This also applies to 10,000, 100,000 and other integers that follow this pattern.

Multiplication and Division


4. Doubling and Halving

When students have to multiply two integers, they can speed up the process when one is an even number. They just
need to halve the even number and double the other number.
Students can stop the process when they can no longer halve the even integer, or when the equation becomes
manageable.
Using 33 x 48 as an example, here’s the process:

 66 x 24
 132 x 12
 264 x 6
 528 x 3
 1,584

The only prerequisite is understanding the 2 times table.

5. Multiplying by Powers of 2
This tactic is a speedy variation of doubling and halving.
It simplifies multiplication if a number in the equation is a power of 2, meaning it works for 2, 4, 8, 16 and so on.
Here’s what to do: For each power of 2 that makes up that number, double the other number.
For example, 9 x 16 is the same thing as 9 x (2 x 2 x 2 x 2) or 9 x 24. Students can therefore double 9 four times to
reach the answer:

 9 x 24
 18 x 23
 36 x 22
 72 x 2
 144

Unlike doubling and halving, this technique demands an understanding of exponents along with a strong command
of the 2 times table.

6. Multiplying by 9
For most students, multiplying by 9 — or 99, 999 and any number that follows this pattern — is difficult compared
with multiplying by a power of 10.
But there’s an easy tactic to solve this issue, and it has two parts.
First, students round up the 9 to 10. Second, after solving the new equation, they subtract the number they just
multiplied by 10 from the answer.
For example, 67 x 9 will lead to the same answer as 67 x 10 – 67. Following the order of operations will give a
result of 603. Similarly, 67 x 99 is the same as 67 x 100 – 67.
Despite more steps, altering the equation this way is usually faster.

7. Multiplying by 11
There’s an easier way for multiplying two-digit integers by 11.
Let’s say students must find the product of 11 x 34.
The idea is to put a space between the digits, making it 3_4. Then, add the two digits together and put the sum in
the space.
The answer is 374.
What happens if the sum is two digits? Students would put the second digit in the space and add 1 to the digit to the
left of the space. For example:

 11 x 77
 7_(7+7)_7
 7_(14)_7
 (7+1)_4_7
 847

It’s multiplication without having to multiply.

8. Multiplying Even Numbers by 5


This technique only requires basic division skills.
There are two steps, and 5 x 6 serves as an example. First, divide the number being multiplied by 5 — which is 6
— in half. Second, add 0 to the right of number.
The result is 30, which is the correct answer.
It’s an ideal, easy technique for students mastering the 5 times table.

9. Multiplying Odd Numbers by 5


This is another time-saving tactic that works well when teaching students the 5 times table.
This one has three steps, which 5 x 7 exemplifies.
First, subtract 1 from the number being multiplied by 5, making it an even number. Second, cut that number in half
— from 6 to 3 in this instance. Third, add 5 to the right of the number.
The answer is 35.
Who needs a calculator?

10. Squaring a Two-Digit Number that Ends with 1

Squaring a high two-digit number can be tedious, but there’s a shortcut if 1 is the second digit.
There are four steps to this shortcut, which 812 exemplifies:

 Subtract 1 from the integer: 81 – 1 = 80


 Square the integer, which is now an easier number: 80 x 80 = 6,400
 Add the integer with the resulting square twice: 6,400 + 80 + 80 = 6,560
 Add 1: 6,560 + 1 = 6,561
This work-around eliminates the difficulty surrounding the second digit, allowing students to work with multiples
of 10.

11. Squaring a Two-Digit Numbers that Ends with 5


Squaring numbers ending in 5 is easier, as there are only two parts of the process.
First, students will always make 25 the product’s last digits.
Second, to determine the product’s first digits, students must multiply the number’s first digit — 9, for example —
by the integer that’s one higher — 10, in this case.
So, students would solve 952 by designating 25 as the last two digits. They would then multiply 9 x 10 to receive
90. Putting these numbers together, the result is 9,025.
Just like that, a hard problem becomes easy multiplication for many students.

12. Calculating Percentages


Cross-multiplication is an important skill to develop, but there’s an easier way to calculate percentages.
For example, if students want to know what 65% of 175 is, they can multiply the numbers together and move the
decimal place two digits to the left.
The result is 113.75, which is indeed the correct answer.
This shortcut is a useful timesaver on tests and quizzes.

13. Balancing Averages

To determine the average among a set of numbers, students can balance them instead of using a complex formula.
Suppose a student wants to volunteer for an average of 10 hours a week over a period of four weeks. In the first
three weeks, the student worked for 10, 12 and 14 hours.
To determine the number of hours required in the fourth week, the student must add how much he or she
surpassed or missed the target average in the other weeks:

 14 hours – 10 hours = 4 hours


 12 – 10 = 2
 10 – 10 = 0
 4 hours + 2 hours + 0 hours = 6 hours

To learn the number of hours for the final week, the student must subtract the sum from the target average:

 10 hours – 6 hours = 4 hours

With practice, this method may not even require pencil and paper. That’s how easy it is.

Word Problems
14. Identifying Buzzwords
Students who struggle to translate word problems into equations will benefit from learning how to spot buzzwords
— phrases that indicate specific actions.
This isn’t a trick. It’s a tactic.
Teach students to look for these buzzwords, and what skill they align with in most contexts:

Addition Increased by; more than; combined; total of; sum


Subtraction Less than; fewer than; reduced by; difference of

Multiplication Times; the product of

Division Per; out of; ratio of; quotient of

Be sure to include buzzwords that typically appear in their textbooks (or other classroom math books), as well as
ones you use on tests and assignments.
As a result, they should have an easier time processing word problems.

15. Creating Sub-Questions

For complex word problems, show students how to dissect the question by answering three specific sub-questions.
Each student should ask him or herself:

 What am I looking for? — Students should read the question over and over, looking for buzzwords and
identifying important details.

 What information do I need? — Students should determine which facts, figures and variables they need
to solve the question. For example, if they determine the question is rooted in subtraction, they need the
minuend and subtrahend.

 What information do I have? — Students should be able to create the core equation using the information
in the word problem, after determining which details are important.

These sub-questions help students avoid overload.


Instead of writing and analyzing each detail of the question, they’ll be able to identify key information. If you
identify students who are struggling with these, you can use peer learning as needed.
For more fresh approaches to teaching math in your classroom, consider treating your students to a range of fun
math activities.

Final Thoughts About these Ways to Solve Math Problems Faster


Showing these 15 techniques to students can give them the confidence to tackle tough questions.
They’re also mental math exercises, helping them build skills related to focus, logic and critical thinking.
A rewarding class equals an engaging class. That’s an easy equation to remember.

Calculators are awesome, but they’re not always handy. More to the point, no one wants to be seen
reaching for the calculator on their mobile phone when it’s time to figure out a 15 percent gratuity. Here
are ten tips to help you crunch numbers in your head.

Mental math isn’t as difficult as it might sound, and you may be surprised at how easy it is to make
seemingly impossible calculations using nothing but your beautiful brain. You just need to remember a
few simple rules.

Add and Subtract From Left to Right

Remember how you were taught in school to add and subtract numbers from right to left (don’t forget to
carry the one!)? That’s all fine and well when doing math with pencil and paper, but when performing
mental math it’s better to do it moving from left to right. Switching the order so that you start with the
largest values makes it a bit more intuitive and easier to figure out. So when adding 58 to 26, start with
the first column and calculate 50+20=70, then 8+6=14, which added together is 84. Easy, peasy.

Make It Easy on Yourself

When confronted with a difficult calculation, try to find a way of simplifying the problem by temporarily
shifting the values around. When calculating 593+680, for example, add 7 to 593 to get 600 (more
manageable). Calculate 600+680, which is 1280, and then take away that additional 7 to get the correct
answer, 1273.

You can do a similar thing with multiplication. For 89x6, calculate 90x6 instead, and then subtract that
additional 6, so 540-6=534.

Memorize Building Blocks


Examples of “buildings blocks.” See more here.

Memorizing multiplication tables is an important aspect of mental math, and it shouldn’t be discounted.

Spencer Greenberg, a mathematician and founder of ClearerThinking.org, says that by memorizing these
basic “building blocks” of math, we can instantly get answers to simple problems that are embedded
within more difficult ones. So if you’ve forgotten these tables, it would do you well to quickly brush up.
While you’re at it, memorize your 1/n tables so you can quickly recall that 1/6 is 0.166, 1/3 is 0.333, and
3/4 is 0.75.

Remember Cool Multiplication Tricks

To help you do simple multiplication, it’s important to remember some nifty tricks. One of the most
obvious rules is that any number that’s multiplied by 10 just needs to have a zero placed at the end When
multiplying by 5, your answer will always end in either a 0 or 5.

Also, when multiplying a number by 12, it’s always 10 times plus two times that number. For example,
when calculating 12x4, do 4x10=40, and 4x2=8, and then 40+8=48. One of my favorites is multiplying
by 15: just multiply your number by 10, and then add half to the answer (e.g. 4x15 = 4x10=40, plus half
that answer, 20, giving you 60).

There’s also a neat trick for multiplying by 16. First, multiply the number in question by 10, and then
multiply half the number by 10. Then add those two results together with the number itself to get your
final answer. So to calculate 16 x 24, first calculate 10 x 24 = 240, then figure out half of 24, which is 12,
and multiply by 10, giving you 120. Simple math finishes it up: 240+120+24=384.

Similar tricks exist for other numbers, which you can read about here.

Squares Are Your Friends

These simple tricks are all fine and well, but large numbers present a different challenge. For that, a
physicist from askamathematician.com says it’s a good idea is use the difference of squares (a square
being a number multiplied by itself).
“Take the two numbers you’re multiplying and think of them as their average, x, plus and minus the
difference between each and their average, ±y,” he says. “These two numbers are squared, so rather than
memorizing entire multiplication tables you only memorize squares.”

Recent Video from GizmodoVIEW MORE >


You Can Finally Buy a Robot That Will Be Your Friend
Monday 2:43PM

It may seem like a daunting task, but memorizing all the squares from 1 to 20 isn’t as bad as it sounds.
It’s just 20 numbers, after all. Armed with this prior knowledge, you can perform some pretty incredible
calculations.

Here’s how it works, starting with a simple example. Let’s assume for a moment that we don’t know the
answer to 10x4. The first step is to figure out the average number between these two numbers, which is 7
(i.e. 10-3=7, and 4+3=7). Next, determine the square of 7, which is 49. We now have a number that’s
close, but not close enough. To get the correct answer, we have to square the difference between the
average (in this case 3) providing us with 9. The last step is to do some simple subtraction, 49-9=40, and
wouldn’t you know it you have the correct answer.

That might seem like a roundabout way to calculate 10x4 (it is), but this same technique works for bigger
numbers. Take 15x11 for example. Once again, we have to find the average number between these two,
which is 13. The square of 13 is 169. The square of the difference in the average (2) is 4. Finally, 169-
4=165, the correct answer.

It’s Okay to Approximate

When doing mental math, particularly for large numbers, it’s often a good idea to make an informed
estimate, and not worry about getting a perfect answer. Back during the Manhattan Project, for example,
physicist Enrico Fermi wanted a rough estimate of the atomic blast’s power before the diagnostic data
came in. To that end, he dropped pieces of paper when the blast wave hit him (from a safe distance, of
course). By measuring the distance the paper traveled, he estimated the blast strength to be about 10
kilotons of TNT. This estimate was fairly accurate, as the true answer was 20 kilotons of TNT.

This technique, now known as a “Fermi Estimate,” works by estimating numbers in powers of ten (see
TED-Ed video above for more). So when trying to come up with a seemingly impossible solution, it helps
to chunk items in this way and then break them down. For example, when trying to estimate the number
of piano tuners in your city, first estimate the population of your city (e.g. 1,000,000), then estimate the
number pianos (10,000), and then the number of piano tuners (e.g. 100). You won’t get the actual
answer, but you’ll get an answer quickly, and one that’s often close enough.

When in Doubt, Rearrange

It’s a good idea to use the rules of math to rearrange complex problems into a simpler form. For
instance, computing the problem 5x(14+43) is a daunting task on it’s own, but it can be broken down
into three fairly manageable calculations. Remembering your order of operations, this problem can be
rephrased as (5x14) + (5x40) + (5x3) = 285.

Turn a Big Problem Into a Bunch of Small Ones

When in doubt, decompose. “For many problems, the way to do them fast is to break them into
subproblems and solve those,” says Greenberg. “When you get a problem that sounds hard, it’s often
fruitful to look for ways that it can be broken apart into easier problems that you already know how to
solve.”

For instance, you can multiply by 8 by doubling three times. So instead of trying to figure out 12x8, just
double 12 three times: 24, 48, 96. Or when multiplying by 5, I start by multiply by 10 since it’s easy, then
divide by 2 since that’s also usually pretty easy. For example, for 5x18, calculate 10x18 instead, and
divide by 2, where 180/2=90.

Use Scientific Notation For Unreasonably Large Numbers

When calculating large numbers in your head, remember that you can convert them into scientific
notation first. What’s 44 billion divided by 400,000? A simple way to deal with this is to convert 4 billion
to 109, and 400,000 to 105. We can now express this as 44/4 and 109/105. As Greenberg points out, the
rule for dividing exponents requires us to subtract them (easy!), so we get 11 x 10(9-5)= 11 x 104 = 110,000.

The Simplest Way to Calculate the Tip

Finally, some advice on how to calculate a tip in your head. If you can calculate a 10 percent tip in your
head (easy), then you can calculate both a 20 percent tip and a 15 percent tip.

When calculating a 10 percent tip for a meal that cost $112.23, just move the decimal point one space to
the left, giving you $11.22. When calculating a 20 percent tip, do the same thing, but simply double the
answer (a 20 percent tip is twice as much as a 10 percent tip), which in this case is $22.44.
For a 15 percent tip, once again calculate the 10 percent tip, and then add half (the additional 5 percent is
just half of the 10 percent amount). So $11.22+(11.22/2). Don’t worry if you can’t get the exact answer. If
we don’t fuss too much with the decimal points, we can quickly calculate that a 15 percent tip of $112.23
is $11 + 5.50, which is $16.50. Close enough. Add a quarter or two if you’re worried about lowballing the
server.

You might also like